You are on page 1of 87

|Pag e1

1. Skin layer which prevent body from water loss is o M phase


 Stratum corneum o None all the above
o Stratum granulosum 12. Separation of reticular and papillary dermis is seen by
o Stratum spinosum arrangement of
o Stratum basale o Ground substance
o All of the above o Collagen
2. Stratum lucidum mostly apparent in : o Reticulum
o Scalp o Elastin
o Back  B&d
 Palm 13. In normal skin , transient-amplifying cells are found
o Face in the :
3. Mucous membrane generally is more permeable than  Basal layer
skin because it lacks o Suprabasal layers
o Keratin o Basal and suprabasal lauers
o A surface lipid film o Dermis
 A stratum corneum 14. About nodule all are false except:-
o Pilosebaceous follicles o Differ from papule only in size
o All of the above o Heals with scarring
4. Time which based cell needs to reach stratum corneum  Palpable circumscribed solid lesion
is:- o Heals rapidly by intralesional steroids
o 2 weeks 15. Whish of these keratinis is characteristic of the
 28 days cornea ?
o 35 days o 16
o 10 days o 14
5. The weakest part of BMZ is:- o 7
 Lamina lucida  3,12
o Lamina densa 16. Which of these keratin pairs are characteristic of
o Anchoring fibrils differentiated epidermal keratinocytes ?
o Hemidesmosomes o 8 and 18
6. Papillary dermis differs from reticular in o 3 and 12
o Reticular fibers o 6 and 16
o Collagen fibers  1 and 10
o Elastic fibers 17. Collagen of human mostly
 Collagen and elastic fiber  Type1
7. Normally the human dermis is thickest on the o Type 2
o Scalp o Type3
 Back o Type4
o Thigh 18. ↑↑Proliferation of keratinocytes by
o Palm  IL-1
o Sole o IL-5
8. Mechanical strength o Catecholamine
 Dermis o Acetylcholine
o Epidermal 19. Description of cell binuclear rounded cytoplasmic
9. Causes of mechanical traction of the skin o Granular
o Epidermis o Merkel cell
 Dermis o Melanocyt
o Subcutaneous o Ls
o Fat  Mast cell
10. Basal cell layer one layer normally but in glabrous skin 20. Cells has a halo surronded it
and hyperproliferative lesion may reach o Mast
o 2 layer o Merkelcell
 3 layer  Mellanocyte cell
o 4 layer o Langrhans
o 5 layer 21. Cells has a halo surrounds it in H/P
11. Normally most of the epidermal cell replication cycle is o Mast cells
occupied by  Merckle cells
 GI phase o Melanocyte cells
o S phase o Langerhans cells
o G2 phase
|Pag e2

22. The definitive organelle of the melanocyte is the o Acantholysis


melanosome, and the definitive organelle in the 32. Vesicule occurs histopathology by –
melanophages is  Loss of cohesion of desmosomes leading to
o Premelanosome acantholysis
o Melanophore o Defect in basement member- bullae
o Mitochondrion 33. A typical early (about 8 hours) response to epidermal
 Lysosome injury is
o None of the above o Glycogen depletion
23. Melanocyte destruction due to cooling occurs at o Glycogen accumulation
following dergrees o Shrinkage of basal cells
 -4 _ -7  Increased numbers of mitoses
o -1 _ -5 34. Langerhans's cells all except
24. Melanocytes in the oral mucosa:- o Play an important role in antigen presentation
o Can seldom be found,  By Hx & E stain they appear as high-level brown cells
o Are normally inactive, o They express class II MHC antigens
o Lack dendrites, o They circulate through the epidermis, the dermis,
o Occur only in laminal propria. lymphatics and lymph nodes
 None of above o Corticosteroids decrease their surface markers
25. Parakeratosis is not typical of 35. One of the following vehicles doesn’t suit an acute skin
o Psoriasis condition:-
o Pityriasis rosea o Powders
o Subacute eczema o Solutions
 Lichen planus  Ointments
o Seborrehic dermatitis o Lotions
26. Fluid accumulation within epidermal spaces due to 36. Least pressure ulcer point:
rupture & death of epithelial cells is called:-  Scapula
o Cyclosis o Heel
o Spongiosis o Ischeal tuberosities
o Acantholysis o Greater trochanter
 None of the above o Secrum
27. Which of the following lack desmosomes ? 37. Type IV hypersensitivity is mediated by:-
o Keratynocytes of the granular  T lymphocyte
o Langerhans cells o Lymphokines
o Melanocytes o Langerhans cell
o Keratinocytes of basal cell layer o IgA
 B and C 38. Epidermis of the facial skin in aged persons is
28. Histological epidermal structure which responsible for characterized by all except
prevention of cohesion of epidermal cells and o Thinning of the malpighian layer
acantholysis and blister formation in vesico- bulious o Reduced number of functioning melanocytes
dermatosis is : o Glycogen accumulation
o Epidermal cell collisions o More marked involutional changes than are seen in
o Intercellular substances non-light exposed skin
 Desmosomes  Absence of stratum granulosum
29. Histological epidermal structure which is responsible 39. Heat receptor can tolerate temperature at what level
for prevention of loss of coherence between epidermal after that pain
cells:- o 42 o C degrees
o Intercellular substance  45 o C degrees
 Desmosomes o 52 o C degrees
o Lamina lucida 40. Normal human epidermal glucose levels in vivo are
o Collagen fibers o About equal to blood levels
30. The basic morphologic patterns of intraepidermal  About 30% to 60% of the blood level
vesicle formation are:- o About 10% of the blood level
o Spongiosis o Zero
o Acantholysis o Not exactly known
o Cytosis 41. Concerning lichenification all are true EXCEPT:-
 All of the above o Thickening of epidermis & accentuation of skin
31. Spongiosis in dermatitis markings
 Intact desmosomes but look stretched o Due to rubbing or scratching
o Intact desmosomes that do not seem stretched  Occurs in acute dermatitis
|Pag e3

42. Pin point bleeding due to thinning of the epidermis is 53. Epidermal infiltration by polymorph nuclear
called:- lymphocyte
o Nicolesky's sign  Psoriasis
o Pathergy reaction o Lichen planus
 Auspitz sign o PRP
o Koebner phenomenon 54. Sebaceous gland activity and vaginal discharge in
43. Auspitz sign is caused by infancy usually is due to
o Test tube rete ridges  Maternal sex hormones
 Thinning of the epidermis overlying the rete ridges o Adrenal hyperplasia
44. Auspitz sign seen in:- o Maternal diabetes mellitus
o Pityriasis rosea o Maternal prenatal vitamin therapy
o LP o None of the above
 Psoriasis 55. The physiologic function of human sebum is
o Allergic contact dermatitis o A barrier layer
45. Sign that occurs by applying force to cause shearing of o A natural antiseptic
the epidermis from the dermis is o A buffer
o Darier's sign o A water retainer
 Nickolsky's sign  Undetermined
o Pathergy reaction 56. Eccrine gland developed intrauterine during
o Auspitz sign o 2th month intrauterine
46. Appearance of the lesion at the same site of the 1ry  4th month intrauterine4-6
lesion in some dermatosis on trauma, scratching or on o 6th month intrauterine
wound scar or sun burn is called:- 57. Development of eccrine glands in sole (palm &
o Pathergy reaction planter)
o Auspitz sign  2-3 months
o Nicolesky's sign o 4-5 months
 Koebner phenomenon o 6-7 months
47. Koebner phenomenon occurs in all EXCEPT:- o 7-8 months
 Pityriasis rosea 58. Normal human eccrine function is
o Pityriasis rubra pilaris o Sympathetic adrenergic
o Wart  Sympathetic cholinergic
o Psoraisiform dermatitis o Parasympathetic adrenergic
48. Kobner’sphenomenon seen in EXCEPT o Parasympathetic cholinergic
o Psoriasis , o Androgen-dependent
o Lp, 59. Apocrine glands all EXCEPT
o Lichen nitidus o Develop from follicular epithelial cells
 Bullous Pemphigoid and keratosis follicularis o Are located in the axillae, perianal region and areole
49. Each of the following are the nikolesky sign except:  Secret an oily malodorous material
o Pemphigus o Have no well-determined function
 Pemphigoid o Are influenced by sex hormones
o H. Simplex 60. Stain of epithelial of eccrine hidrocystoma
o Insect bite  S100 eccrine hidrocystoma (langerhans cells, lands,
50. Positive nikolysky sign found in: schwann cells, chondrocytes , adipose tissue and
o B.pemphegoid. melanoma) (clear cell acanthoma)
 P .vulgaris. o PASS (for granules)
51. Each of the following are +ve Nicolesky's sign o Zehl nelson
EXCEPT:- o Tolidin blue
o Pemphigus vulgaris 61. Apocrine hidrocystoma best investigation diagnostic
o Herpes simplex  PAS
 Pemphigoid o S100 ptn
o S.S.S.S o Toluidine blue
52. Epidermal infiltration by pustule of pmls (Munro 62. The tip of the nose is innervated by:-
microabsceses)occurs in:-  Ophthalmic nerve
o Pustular psoriasis o Occulomotor nerve
o Reiter's syndrome o Trigeminal nerve
o Impetigo herpetiform 63. Nerve supply of the alae nasi by
o Acrodermatitis continua (Dermatitis Repens) o Branch of supratrochlear nerve
 All of the above o Branch of infratrochlear nerve
|Pag e4

 Branch of the infraorbital nerve(from the maxillary o 290-320


division of the trigeminal nerve) 74. Immediate tanning:
o External nasal nerve o 8-12 hours
64. Which of the following criteria will usually help  1-6 hours
differentiate autosomal dominant and recessive o 12-24 hours
inheritance all except o 24-36 hours
o Frequency of phenotypic involvement of parents 75. Sun burn peaks
o Consanguinity o 4-6 hours after exposure
 Frequency of involvement of male vs females  24 hours after
o Presence or absence of the phenotype in succeeding 76. Psoralene reaches the skin after oral administration
generations within:-
o The severity of the disorder  1-2 hours
65. Each of the following is transmitted by autosomal o 3-4 hours
dominant inheritance EXCEPT: o 10-12 hours
o Gardner’s syndrome o 20-24 hours
 Cronkhite-Canada syndrome 77. Patient , whight 70 kg , for PUVA treatment what the
o Peutz-Jeghers syndrome dose of psoralin :
o Cowden’s disease o 20mg befor1h
o Muir-Torre syndrome  40mg befor2h
66. Each of the following is transmitted by autosomal o 20mg befor2h
dominant inheritance EXCEPT : o 40mg befor1h
o Gardner’s syndrome 78. Skin diseases responsive to PUVA therapy include all
 Cronkhite-Canada syndrome except
o Peutz-Jeghers syndrome o Chronic hand eczema
o Cowden’s disease o Sarcoidosis
o Muir-Torre syndrome o Urticaria pigmentosa
67. All of the following are A.D EXCEPT:- o Solar urticaria
o Peutz-Jergers syndrome  Actinic keratosis
 Xeroderma pigmentosa 79. The following dermatosis can be aggravated by
o Darier's disease exposure to UVR all EXCEPT
o Neurofibromatosis o Acne vulgaris
68. The following are autosomal dominant EXCEPT: o Atopic dermatitis
o Neurofibromatosis o Lichen planus
 Xeroderma pigmentosa o Pemphigus foliaceous
o Ataxia  Lichen sclerosus et atrophicus
o Pentz – jeugers. 80. UVB tanning appear after :
69. Patient treated with PUVA should be examine by: o 1 houre
o Neurologist. o 24 hours
 Ophthalmologist. o 48 hours
o Git specialist.  72 hours
70. The earth rays contains most concentration of UVA 81. UVB tanning appear after early hours delayed UVB
o 10am – 2pm tanning appears after -
o 11am – 1pm o 36 hours
 10am – 3pm o 24 hours
o 11am – 4pm  72 hours
o 11am – 2pm o 48 hours
71. Immediate pigment darkening (IPD) is produced and 82. Patient with psoriasis take UVB and not respond which
maintained by:- following drugs :
o Visible light  Acitritin
o UVB o Methotraxate
 UVA o Cyclosporine
o UVC 83. UVB alleviates:-
o All o Pruritis with biliary cirrhosis
72. Immediate tanning reaction occurs with – o Pruritis with diabetes
 UVA  Pruritis of hemodialysis
o UVB 84. The highest SPF available at this time is
73. Immediate tanning: o 15
o 320-340 o 19
 320-400 o 30
|Pag e5

o 39 96. Dermatologic side effects of radiotherapy include all


 50 except
85. The best sun cream (sunscreen ) o Pigmentary changes
o Paba (para-aminobenzoic acid) o Permanent alopecia
 Titanium o Fibrosis
86. For prophylactic from infection , before procedure , the o Radionecrosis
antibiotic should be given  Acneiform eruption
 6 hours 97. To complete depigmentation, the drug of choice is:-
o 14 hours o 0.1% methoxsalen
o 24 hours o 0.6 mg/kg trioxalen
87. High risk patient for cardiac diseases scheduled for  20% monobenzone
operation (cosmotic) should recive antibiotic o 0.1% monobenzone
(prphylactic).To avoid skin infection:- 98. Hydroquinone compounds cause depigmentation by
 1 hour before surgery their action on
o At time of surgery o Melanin
o 6 hours after surgery o Dopa
o 1 day after surgery o Tyrosine
88. Infection in cutaneous surgery usually occurs after  Tyrosinase
o 24 hrs o Melanocytes
o 24 – 48 hrs 99. Contraindications to oral retinoids include all except
o 48 – 72 hrs o Pregnancy
 4-8 days o Patients at risk for liver toxicity
89. The appearance of infection after surgery o Chronic use in children
o After 12-24 h o Concomitant use of methotrexate and oral retinoids
o After 24-48 h  Aphakia
o After 48-72 h 100. The difference of side effect of etretinate and
 4-8 days isotretinoin is that
90. To prevent cutaneous complications after surgery , the o Isotretinoin procedure
antibiotics is given:  Facial dryness + redness of lips
 Before surgery o Alopecia
o Day of surgery 101. Use of minoxidil for 96 day will result in:
o 6 hrs after surgery  Increase hair thickness &weight
o 24 hrs after surgery o Increase hair length & decrease weight
91. Sutures near the eye can be removed after o Increse hair thickend only
 5-7 d 102. Acyclovir-resistant herpes virus infection is best
o 1w treated by
o 7-9d o Cidofovir
o 10-14d o Indinaivir
92. Single or multiple bands in the o Granciclovir
o Vicryl  Foscarnet
o Prolein o Lamivudine
o Silk 103. Penicillamine induce :
 Nylon o Sle
93. Monofilament or single sture o Dermatomyositis
o Vicryl o Pimphigus
o Dexon o Polymyositis
o Chromic  All above
 Prolene 104. Patient who is patch test +ve to ethylendiamine group
94. Type of suture most suitable for cutaneous surgery of antihistamines should avoid
 Single nylon suture  Hydroxyzine (Atarax)
o Double nylon suture o Chlorophenramine
o Braided silk o Terfenadine
o Unbraided silk o Diphenhydramine
95. The most prominent side effects of cryosurgery is:- 105. Dose of nifedipine in treatment of Raynaud
 Hypopigmentation phenomena
o Hyperpigmentation o 10 mg twice daily
o Scarring o 20-30 daily
o Pain  30-40 daily
o Never damage o 10-80 daily
|Pag e6

106. The administration of erythromycin can even the o Skin discoloration


blood level of:-  Skin atrophy
 Theophylline 117. There is little evidence of a need to taper off a course
o Oral contraceptives of corticosteroid therapy (single morning dose) of
o Hydrochlorothiazide duration less than:-
o Insulin o One week
o 13-cls retinoic acid  Two weeks
107. Erythromycin toxicity if take with o One month
o Clindamycin o Six weeks
o Lincomycin o Two months
o Aspirin 118. Cause of hypertension
o NSAID o Dexamethasone
 Warfarin o Prednisolone
108. Lindane is no longer the treatment of choice for  Hydrocortisone
scabies because of o Triamcilone
o Cardiotoxic effects 119. Drug which is most likely to cause hypertension in
o Nephrotoxic effects equivalent dose
 Neurotoxic effects o Prednisolone
o Hepatotoxic effects o Dexamethasone
o All of the above  Hydrocortisone
109. Phenol peeling causes: o Betamethasone
 Cardiotoxicity(cardiac arrythmia) o Triamcinolone
o Renal toxicity 120. 1 gm of dexamethasone equal to how many .gm of
o Neural toxicity triamcinolone:
110. The most safe form of procaine is:- o 1
o Novocain o 2
o Lindocain  5
 Lidocain o 10
111. Patient who is sensitive to procaine can be given 121. The following drugs should not be used concomitantly
 Lidocain with dapsone all EXCEPT
o Benzocaine  Methotraxate
o Novicain o Primaquine
112. Best topical anesthesia with: o Zidovudine
o 1%lidocaine+adrenaline o Chloroquine
 2%lidocaine o Oral contraceptives
113. The best for nerve block of mohs micro surgery 122. Dapsone is most useful in patients with:-
o Procaine alone o Guttate Psoriasis
o Lidocain alone o Psoriatic Erythroderma
 Lidocain + epinephrin o Psoriasis Follicularis
o Bupivacine + epinephrin o Psoriasis rupiodes
114. Alternating days of prednisone eliminates the  Palmoplantar Pustular psoriasis
increased risk of corticosteroid induced : 123. Patient took dapsone after 2 days developed bluish
 Osteoporosis lip &hand what to give
o Posterior subcapsular cataracts o Acetylcystein
o Avascular necrosis of the hip d. o Ethanol
o Myopathy  Methelyne blue
o Growth retardation o Chrcool
115. The following tests can assess the efficacy and 124. The most prominent side effects of Dapsone is:-
strength of topical corticosteroids except  Hemolysis
o The vasoconstriction test o Liver damage
 The repeated open application test o Renal damage
o The atrophy test o Bone marrow depression
o The erythema test 125. Methotrexate before use
o Pyrexal erythema test  Liver enzyme/function
116. The most frequent adverse effect of topical o Liver biopsy
corticosteroids in children is 126. Current recommendation for monitoring
o Scarring methotrexate hepatoxicity in patients without the
o Local irritation risk factors for liver disease is
o Contact sensitization o Liver biopsy before starting methotrexate
|Pag e7

o Liver biopsy after the first 200 to 400mg of o Cimetidine


methotrexate  Rifamficin
o Liver biopsy when patients have been treated with 1 to o Allopurinol
1.5g of methotrexate 136. Cyclosporine level is increased by all EXCEPT:-
o Liver biopsy when patients have been treated with 3g o Erythromycin
of methotrexate  Rifampicin
 Liver biopsies are not necessary as long as patient have o Ketoconazole
liver function tests drawn monthly. o Allopurinol
127. Methotrexate what right 137. Drugs affect cyclosporine metabolism:-
o No need to biopsy in every case o Rifampicin
o Liver enzyme is necessary o Erythromycin
o Pneumonia is more common with rheumatoid arthritis o Ketoconazole
than psoriasis  All of the above
 All of the above 138. Drug that increase level of cyclosporine is
128. Drug causes hepatotoxicity by cumulative effect:-  Ketoconazole and fluconazole
o Ketoconazole o Phenoparbitorat
 Methotrexate o Rifampicin (rifampin)
o Tetracyclines 139. One of the following, the most important side effects
o Ampicillin of cyclosporine therapy are:-
129. Side effects of methotrexate include all except o Leukopenia and bone marrow suppression
o Hepatotoxicity o Granulocytopenia and infection
o Phototoxicity  Hepatotoxicity and nephrotoxicity
o Chills and fever o Neurotoxicity and temporary dementia
o Cytopenia o Colitis and hemorrhagic cystitis
 Keratitis and conjunctivitis 140. Drug more safe in pregnancy is
130. In methotrexate hepatotoxicity : o Gentamicin
o Liver biopsy is not necessary in patient lack clinical and  Ampicillin
laboratory evident of liver disease. o Ketoconazole
o The hepatic c fibrosis in patient with rh arthritis is more 141. Which statement of the following is untrue regarding
benign than in patient with psoriasis. Itraconazole?
o The pnemonitis side effect is far more benign in patient o It is a triazole
with psoriasis than in patient with rheumatoid arthritis. o The absorption is increased by food
o Liver enzymes are essential before treatment despite o Pulse dosing may limit potential toxicity
controversy about their role.  It is fungicidal
 All above o Gastric acid secretion suppressors produce erratic or
131. Methotrexate what the latest decreased absorption
o Teratogens 142. Ketoconazole not used with:
o Myeloplstic o Aspirin
o Bone narrow suppression o Theophylline
o Stomach upset o Erythromycin
 Hepatotoxicity  Astemazol
132. The principal route of excretion of methotrexate is o All of above
o Biliary 143. Fluconazole, what is wrong:-
 Renal  Cannot take in liver disease
o Cutaneous o Half live 30 h
o Respiratory o Taken every one week
o None of the above o The renal excrtectis not important
133. Cyclosporine is 144. As the half life of fluconazole is long , it can be use
o Nephrotoxic every:
o Hepatotoxic o Day
o Hepatonephrotoxic  Week
 All true o 2 weeks
134. Cyclosporine is commonly:- o 1 month
 Nephrotoxic 145. About fluconazole what is wrong:-
o Hepatotoxic o Can be given every week
o Hepatonephrotoxic o Triazole group
o All of the above  Metabolism not affect the kidney function
135. Cyclosporine levels are reduced by: o Its half-life 24:-30 hour
o Ketoconazole o Mainly renal excretion
|Pag e8

146. Itraconazole levels may be elevated if the patient is 157. Most common sensitivity test (skin test)
concomitantly taking which of the following drugs- o Prickle test
 Cimetidine  Patch test
o Phenobarbitone o Rast test
o Phenytoin 158. Photosensitizing drugs include all EXCEPT
o Rifampin o Chloroquine
o Carbamazepine o Nsaids
147. Inhibition of squalene epoxidase is the mechanism of o Thiazides
action of o Sulphonamides
o Nystalin  PABA
o Fluconazole 159. What is best cytotoxic drug can be used in patient
o Itraconazole with severe ocular damage
o Ketoconazole o Corticosteroid
 Terbinafine o Azathrioprin
148. The maximum daily dose of amphotericin B does not  Cyclophosphamid
usually exceed o Infleximab
 1 mg/kg body weight 160. Cytotoxic drug safe on eye
o 10 mg /kg body weight o Azathioprine
o 100 mg/kg body weigh o Methotrexate.
o 1 gram o Cyclosporine
o 10 grams o Mycophenolate mofetil
149. Drug that have the greatest potential phototoxic  Cyclophosphamide
reaction:- 161. Fluorouracil inhibits DNA synthesis by
o Benoxaprofen o Transfer RNA blockage
 Doxycycline o Dihydrofolate reductase blockage
o Sulphonamides  Thymidylate synthetase blockage
150. The most photosentizing drug is: o Adenylcyclase blockage
o Chloroquine o None of the above
 Diphenylhydramine 162. Immunosuppressive therapy
151. All of the following are photoaggravated EXCEPT o Metoterxat
o LE o Cyclosporine
o Herpes simplex o Azathioprine
o Dermatomyositis  All
 Pimphigus 163. Imiquimod when used for treatment of genital warts
o Xeroderma pigmentosa , what is the most popular side effect
152. Systemic photosensitivity caused by which group of o Ulceration
drugs o Erosion
 Sulphones  Pain
o Sulphonylureas o Dyes.
153. The most common cause of drug allergy: 164. Which of the following TNF inhibitors are
 Exanthema. monoclonal antibodies?
o FDE.  Adalimumab and infliximab
o Hyperpigmentations. o Adalimumab and Etanercept
154. Which of the following drugs photosensitivity o Etanercept and infliximab
o Sulfonylurea o Etanercept and Efalizumab
 Sulfones o None of the above
o Hydantion 165. Thalidomide use in treatment of
o Chloroquine o Psudoxanthoma elasticum
155. All of the following are photoexaggerated EXCEPT: o Parapsoriasis
o Herpes simplex  Pyoderma gangrenosum
o Dermatomyositis 166. Black hairy tongue what is the treatment:-
o LE o Antibacterial
o Xeroderma pigmentosa o Antiviral
 Pemphigus vulgaris o Antifungal
156. Photo patch test can read  Brushing the tongue
o 24h 167. What is the following disease has genetic
o 36h susceptibility (Hereditary disease)
 48-72h o EM
o 96h o EN
|Pag e9

o E Margentium o There is predisposition to generalized herpes simplex


 Eczema infections
168. Disease which can be caused by immunological 178. Regarding Seborrehic dermatitis all true EXCEPT:-
reaction is:- o Blepharitis is common
o Urticaria  Incidence is equal in both male & female
o Seborrehic dermatitis o Pityrisporum ovale is the causative organism
 Contact dermatitis o Seborrehic dermatitis of scalp is the most common
o Atopic dermatitis forms (Pityriasis Sicca)
169. The primary histologic feature of eczema is 179. A 3 months infant presented with erythematous rash
 Spongiosis with well-defined border covered with greasy scales in
o Acanthosis groins & the folds were not involved. Diagnosis is:-
o Hyperkeratosis o Psoriasis
o Vasodilation o Atopic dermatitis
o Lymphocytic infiltration  Seborrehic dermatitis
170. All are similar in histopathology EXCEPT:- o Candidiasis
o Contact dermatitis 180. The histologic feature most helpful in distinguishing
o Nummular dermatitis Seborrehic dermatitis from psoriasis is
o Atopic dermatitis o Hyperkeratosis
 Psoriasis o Parakeratosis
o "id" reaction o Acanthosis
171. All of the following are true atopic dermatitis o Inflammatory infiltrate
EXCEPT:-  Spongiosis
o Pruritis 181. Coin sized subacute eczematous plaques with thin
o Chronicity scale and minimal exudation usually few in number,
 2% of patients have Ig E occurring primarily on the extremities in older patient
o +ve family history with no history of atopy:-
172. All are suggesting of atopic dermatitis EXCEPT:- o Cutaneous T cell lymphoma
o Can be localized on cheeks  Nummular eczema
o Very itchy o Atopic eczematous dermatitis
 Depigmentation o Tinea corporis
o >2 months age o Stasis dermatitis
173. The uncommon manifestation of atopic dermatitis is 182. House wife dermatitis can exaggerated by all
o Eczema herpeticum EXCEPT:-
o Infection o Water
 Chicken pox o Soap
174. Child atopic dermatitis what %for having cataract o Gloves
capacity  Ingestion of rice
o 1-5% 183. Hair-dye contact dermatitis is caused by
 10-15% (5-10%)  Para-phenylenediamine
o 20-30% 184. Mason (‫ )عامل بناء‬with contact dermatitis both hands
o 70-80% with remission when he gives in holiday , what most
175. One of the following is a major criteria of atopy:- probably cause :
 Pruritis o Neikle
o White dermaographism  Cement ( potassium dichromate)
o Icthyosis o Cobalt
o Pityriasis alba 185. Cement dermatitis is most commonly caused by:-
176. Infant with atopic dermatitis , what do you suspect  Potassium dichromate
to see in patient : o Nickel
o Flexor lichinification o Cobalt
 Erythematous lesion with crust o Arsenic
o Cataract 186. Contact dermatitis in photographer most commonly
o Decrease igg due to:-
177. In atopic dermatitis, all are true EXCEPT:-  Potassium dichromate
o White dermographism is present o Formaldehydes
o 30-40% of the patient have icthyosis vulgaris o Arsenic
o There is an increase susceptibility to staphylococcal o Gold
infections o Copper
 The ige is elevated in 20% of the patients 187. Typical sensitizing agents in contact dermatitis
include all EXCEPT
| P a g e 11

o Lanoline o Bismuth
o Rubber o Lead
 Ethanol 197. The pigment in talon noir (black heel) is
o Colophony o Melanin
188. Perioral dermatitis due to chewing gum caused by:- o Lipofucin
o Citric acid o Silver
o Lennon oil o Shoe leather dye
o Argenum  Hemoglobin
 Cinnamon ‫القرفة‬ 198. Elastic parts of underwear are rendered allergic
189. The differential diagnosis of perioral dermatitis: through the reaction with:-
o Rosacea o Detergents
o Contact dermatitis o Oily lubricants
o Acne vulgaris  Laundry bleaching
o Seborrehic dermatitis 199. The cause of blister beetle dermatitis is
 All of the above o Beetle sting
190. Perioral dermatitis:- o Beetle bite
o Primarily affects ♂ between the ages of 16 and 45 o Beetle feces
 Primarily affects ♀ between the ages of 16 and 45 o Retained beetle parts
o Is definitely caused by ultraviolet light  Beetle crushes against the skin
o Is definitely caused by fusiform bacteria 200. Papular dermatitis treatment
o None of the above o Permethrin
191. Facial foundations suitable for extremely long o Ivermectin
wearing and for camouflaging should be o NSAID
o Water – based  Antihistamine
o Oil- based 201. Most common topical drug that causes allergic
o Oil- free dermatitis is
 Water- free  Neomycin
o Any of the above o Chloromaephencol
192. Foundation makeup without sunscreen provides SPF o Penicillin
around 202. The usual site of patch test is:-
o <2 o Face
o 3 to 4  Back
o 8 to 10 o Arm
o 10 to 15 o Thigh
 > 15 203. Photosensitive Tatto is due to
193. The most important sensitizers in lipsticks are  Cadmium sulfide
o Perfumes o Ferric oxide
o Antioxidants o Mercury
o Insoluble colors o Chromium oxide
o Mordants 204. The differential diagnosis of flexural dermatitis in
 Fluorescein stains infancy includes all EXCEPT
194. Detergent dermatitis of the hands o Seborrehic dermatitis
 Is mostly irritant dermatitis o Lettere-Siwe disease
o Is mostly allergic dermatitis  Asteotic eczema
o Can be easily diagnosed with patch tests o Wiskott-Aldrich syndrome
o Is usually due to enzyme additives o Miliaria
o Is usually due to perfume additives 205. What is true about Wischott-Aldrich syndrome:-
195. Both allergic contact dermatitis and contact urticaria o Chronic dermatitis resembling atopic dermatitis
may be caused by:- o Thrombocytopenic Purpura
o Parsley o Recurrent infections
o Indian bean  All of the above
o Onion 206. Chronic dermatitis with thrombocytopenia plus
o Tomato recurrent infection occur in
 Garlic  Wiskott Aldrich syndrome
196. Gray pigmentation of the skin of the face and neck in o Bazex syndrome
middle-aged women is most likely due to o Xeroderma pigmentosa
o Silver 207. To differentiate between hyper ige syndrome and
o Gold atopic eczema:
 Mercury o By clinical picture.
| P a g e 11

o Increase ige above5000 iu ,in hyper ige.  Topical retinoid


 Presence of staph lung abscess & skin abscesses in o Dapson
hyper ige. o Doxyclin
208. Primary lesions of acne include:- o Pencellin.
o Papules 221. Acne conglobata all true EXCEPT:-
o Pustules o More in male than in female
 Comedones  Absence of comedones
o All of the above o Hidardentis suppurativa may associate it
209. The pathognomonic lesion of acne vulgaris is the o Most common on posterior neck & back
o Papule 222. Acne conglobata is best treated with:-
 Comedone o Topical Tetracycline's
o Pustule o Systemic Tetracycline's
o Cyst o Ampicillin
210. The comedones are seen in  None of the above
o Acne 223. Teenager male presented with severe inflammatory
o Periorbital ages skin nodulocysistic lesions on face, the most probable
o Radiated skin diagnosis is:-
 All of the above o Acne vulgaris
211. Small papule with invisible opening  Acne conglobata
 White comedone o Acne necrotica
212. Sebum function all of following EXCEPT o Acne rosacea
o Immune regality modations o Drug induced acne
 Vit.E source from skin 224. All are true about acne fulminants EXCEPT:-
o Protect from any microbiocal pathology o Fever
o Immuno preventive barrier o Leucocytosis
213. Unique composition of human sebum is o Arthritis
o Cholesterol is more than wax esters o Lesions contains osteolytic areas
 Squalene is more than cholestrol  Gram- ve infection
214. Sebum production increase if 225. Acne fulminants can be presented by all EXCEPT:-
 Squalene is more than cholesterol o Fever
o Cholesterol is more than Squalene  Hemoptysis
o Wax ester is more than Squalene o Polyartheritis
215. Topical treatment in acne vulgaris that decrees o Leucocytosis
sebum is : 226. For acne fulminans:
 Clindamycin o Arthralgia
o Erythromycin o Fever
o Benzoyl peroxide o Leukocytosis
o Azelaic acid o Malaise
216. Which of the following topical agents has effect of  All of the above
reducing sebum production ? 227. Monomorphic eruption on sun exposure:-
 Topical clindamycin plus nicotinamide o Acne tropica
o Topical erythromycin plus zinc  Acne aestivalis
o Topical benzol peroxide o Acne cosmotica
o Topical azelaic acid o Acne venenata
217. The follicles in which acne lesions develop generally 228. Drug induced acne is characterized by:-
contain o Cause nodule or cyst
o Only sebum  Monomorphic
o No hair o Mainly on the face
 A villus hair o Has connection
o A terminal hair 229. Systemic steroid acne commonly seen on
o Only keratin o Face
218. Acne venenata most common cause  Back
o Perfumes o Abdomen
o Halogenated hydrocarbon o Buttocks
 Cutting non absorbable oil o Thigh
o Compliment 230. Premenstrual acne is mainly mediated by
219. Acne venenata most common cause o Estrogen
 Cosmetic  Progesterone
220. Treatment of acne venenata o DHT
| P a g e 12

o Glucocorticoids  Granular cell layer


o None of the above o Malpgian layer
231. Side effects of Isotretenoin include:- o Basal layer
o Chelitis 243. What is not present in psoriasis. :
o Hair fall o Ospitz sign
o Diarrhea  Degenerated DEJ
o Pseudotumor cerebri o Parakeratosis
 All of the above 244. One of the following is not a typical histopathological
232. Long term side effect of isotretinoin feature & psoriasis:-
 Skeletal hyperostiosis o Acanthosis & elongation of rete ridges
o Cirrhosis o Parakeratotic horny layer
233. The most common well established side effect of oral o Thinning of the suprapapillary regions of stratum
retinoid is:- corneum
o Exfoliative dermatitis  Degeneration of the dermoepidermal junction
o Erythroderma 245. The hispatology of psoriasis is the same as that of
o Alopecia areata eczema except
o Optic abnormalities  Microabcess in psoriasis
 Teratogenicity 246. HLA in psoriasis
234. Treatment of Acne recalcitrant o HLA B12, B17
o Sulfur high dose o HLA B37
o Estrogen  HLA CW6
o Dapsone o HLA B7
o Prednisone 247. Cell cycle in psoriasis duration is
 All of the above o 2 hrs
235. We must not use this drug in excoriated acne: o 20 hrs
 Benzyl peroxide  37.5 hrs
o Tetracyclines o 200 hrs
o Clindamycin 248. Precipitating factors for psoriasis all EXCEPT:-
236. Treatment of scared acne which type of laser o Drugs as B. Blockers & lithium
o Diode laser o Stress
o Nd-yag 1064nm laser o Trauma
 Erbium laser  Egg
237. Ice-pick acne scars are best treated by 249. The most characteristic physical sign of a psoriatic
o Microdermabrasion nail is
o TCA peeling 50%  Pitting
o Eryag laser therapy o Onycholysis
 Punch replacement grafting o Onychomycosis
o Topical tretinoin o Subungual hyperkeratosis
238. Dermbrasion is used to treat acne scar in the:- o Greenish-yellow discoloration
 Face 250. The most common manifestation of psoriasis in nails
o Trunk  Pitting
o Shoulder 251. Oil spot in psoriasis occur on
o Forearms  Nail bed
239. Rosacea can have all EXCEPT:- o Nail matrix
 Comedon o Nail plate
o Follicular papules o Nail fold
o Telangectasia 252. Pustular psoriasis given isotretinon what most side
o Pustules effect
240. Rosacea cause all EXCEPT:-  Xerosis
o Conjunctivitis o Photosencetivty
o Sclera o Cushing
o Blepharitis o Chest toxicity
o Iritis 253. 100 patient psoriasis put under investigation trials
 Retinitis 2years and 2years acteritin treatment doctors taken a
241. Rhinophyma is the complication of:- liver biopsy from these patients 2year before and
 Acne Rosacea after 2 year what you expect
o Acne vulgaris  No correlates acctretin effect patient liver
o Hidardentis suppurativa o Results correlates toinotive of psoriatic disease
242. Psoriasis have abscent of o Deterioration of liver pervious disease
| P a g e 13

o No histological improvement in biopsies of liver 263. Hypertrophic lichen planus associated with
associated disease psoriasis  Sporadic lichen planus
254. Case of psoriasis in children treatment with narrow o Atrophic lichen planus
band UV effectiveness increased in combination o Familial lichen planus
with 264. Violaceous hyperpigmented plaque with wary surface
o Glucocorticoids on legs associated with flat topped papules on wrists &
o Cyclsporin forearms the diagnosis is:-
o Tacrolimos o Viral warts
 Nothing o Kaposi sarcoma
255. The range of concentrations of anthralin used  Hypertrophic lichen planus
topically for psoriasis in the "Ingram regimen" is o Nodular amyloidosis
o 0.01% to 0.25% 265. Most choice of LP treatment is:
 0.1% to 2% o Aspirin
o 0.2% to 0.5% o Methotrexate
o 3% to %  None of above
o 5% to 15% 266. The following statements about lichen nitidus are true
256. Patient developed erytheroderma and has an elevated all except
liver enzyme best treatment o An association with lichen planus is not uncommon
o Isotretinoin o Most cases occur in children or young age
o MTX  Histopathology shows hyperkeratosis, focal
o Corticosteroid hypergranulosis, irregular acanthosis with liqufication
 Cyclosporine degeneration of basal cell layer
257. Methotrexate therapy of psoriasis patient is o A self-limiting dermatosis
contraindicated in:- o Systemic steroids might be helpful in widespread
o D.M lesions
 Liver cirrhosis 267. Lichen straiatus:-
o Pulmonary disease  Self-limited disease
o Pustular psoriasis o More above age of 20
258. Patient with severe psoriasis and have severe o Variant of nevus lateralis
hypertension which type of drug he use it o Always limited to head and neck
o Methtraxate 268. Lichenoid drug eruption caused by:-
 Cyclosporine A o Gold
o Cyclophosphamide o Hydrochlorothiazide
259. Psoriatic patient developed hypertrichosis which o Penicillamine
treatment do that o Captopril
o Cyclophosamide o Quinidine
 Cyclosporine A o Antimalarial
o Methtraxate  All of above
o Retinoid 269. About Pityriasis rosea all true EXCEPT:-
260. The most suitable among the new biologic therapies o Self-limiting
for treatment of psoriatic arthritis with stable psoriasis  Never caused by drugs
is o Generalized
o Infliximab o Symptomatic treatment
 Etanercept 270. About Pityriasis rosea all true EXCEPT:-
o Adalimumab  Worsen by sun light
o Alefacept o Begins with herald patch
o Efalizumab o May be due to viral infection
261. Histopathology of lichen planus:- o Self-limiting
o Orthokeratosis 271. A safe and effective therapy for pityriasis lichenoid
o Saw tooth pattern of hyperplasia chronic is:-
o Band like infiltration of superficial dermis  PUVA
o Civatte bodies o Penicillin
 All of above o Vitamin E
262. Hypertrophic lichen planus most commonly on o Reserpine
o Lower lip o Bleomycin
 Anterior lower leg 272. Best or most treatment of pityriasis lichenoid chronica
o Buccal mucosa  Phototherapy
o Periungual finger and toes o Corticosteroid
o Scalp o Cytotoxic
| P a g e 14

273. Treatment of PLEVA of child 10 year o Solidification of the offending oil in the follicle
 Erythromycin o Promotion of bacterial growth by the offending oil
o Tetracycline  Follicular hyperkeratosis
o Puva o None of the above
274. 6 years old child with 90% of his body shows o All of the above
erythema with scales and there is severe yellowish 283. Gram-negative folliculitis is best treated by
scales on his palm and soles with follicular o Tetracycline
hyperkeratosis on his interphalenges what is the best o Erythromycin
ttt: PRP o Doxycycline
 Retinoid o Minocycline
o Uvb  Isotretinoin
o Steroids 284. A 35 years old man has marked pruritic, flesh colored
275. Anteriolatral of thigh + anteriolatral of arm child papules limited to the face, neck, upper chest and back.
show follicular hyperkeratosis papules resolve no A skin biopsy reveals perifollicular inflammation with
decrease of vit. (PRP) What best ttt: numerous eosinophils vascular proliferation and dermal
o Vit A fibrosis. Bacterial and fungal are negative. Serum ige
o Codliver oil elevated. The most likely diagnosis is:-
o Vit C o Chronic granulomatous disease
 Tropical retinoid , SA  Eosinophilic folliculitis
276. Drug that best covers impetigo is o Eosinophilic cellulitis
o Erythromycin o Urticaria
o Azithromycin o Hyperrimmunoglobulinemia E syndrome
o Clarithromycin 285. Fatal outcome SSSS is least likely in which of the
o Clindamycin following age groups
 Dicloxacillin o 0-1 year
277. Bullous impetigo is caused by:-  1-5 years
o Streptococci o 5-15 years
 Staphylococci (coagulase +ve) o 15-40 years
o Staphylococci (coagulase -ve) o 40-60 years
o None of the above 286. Best treatment of erysipelas in children is:
278. Staph infection can split the DEJ by toxins directed  Penicillin
against : (ssss) o Floxacillin
o Desmoglein III o Erythromycin
 Desmoglein I o Azithromycin
o Desmocollin I 287. Necrotizing Fasciitis what possible treatment
o Desmocollin III  Debridement
279. Which antibiotic has the LEAST activity against o Antibiotic
Staphylococcus aureus ? o Corticosteroid
 Erythromycin 288. What is the MOST important step in the management
o Clindamycin of necrotizing fasciitis ?
o Vancomycin o Bedrest and pain control
o Dicloxilln o Histologic confirmation of the diagnosis
o First generation cephalosporins o Culture and antibiotic sensitivity testing
280. The main local source of S aureus contaminating the  Surgical debridement and decompression
skin is o Assessment of the patient’s immune status
 Nasopharynx 289. Child symptoms had sore throat then L.N. later on
o Scalp palmoplanter peeling and strawberry tongues best
o Axillae treatment (Scarlet fever)
o Perineum  Erythromycin
o Mouth o Acyclovir
281. Most common among factors predisposing to o Retinoids
development of furunculous is o Antipyretic
 Nasal carriage of the organism 290. Patient taken phenytoin and after that developed
o Underlying immune disorder blister and target lesion, erosion 75% of his skin
o Anemia without oral lesion in probable diagnosis
o Diabetes o EM
o Poor skin hygiene o SJS
282. The folliculitis caused by contact with cutting oils is  TEN
due to o Drug exanthen
| P a g e 15

291. Patient presented to ER congestive eye after she o Aspergellous


receive phenytoin she has targetoid lesion on her hand 301. The causative organism in cat disease is
& feet she developed skin lesion dusky cooler which o Bartonella quintata
peel sheets she suffer from painful oral lesion. What is o Brucella melitenesis
your diagnosis o Bartonella bacilliformis
o HSV  Bartonella henselae
o EM o Coxiella bumetti
o SJS 302. The highest concentration of organisms in an infant
 TEN with early-onset neonatal listeriosis is in the
292. Rate of mortality in TEN with Steven Johnson o Liver
syndrome:- o Lungs
o 5% o Brain
o 5%-15% o Skin
 15-30%  Blood
o 50% 303. The treatment of choice for listeriosis is intravenous
293. Most common causes of death in TEN o Ciprofloxacin
o Electrolyte disturbance o Dixocycline
o Hemodynamic shock o Minocycline
 Sepsis  Ampicillin
o Renal failure o Cephalexin
o Hepatic failure 304. The best treatment for steatocystoma suppurativa is
294. In patients with extensive erythrasma one should rule o Oral isotretinoin
out o Topical metronidazole
o Cancer  Oral tetracycline
 Diabetes mellitus o Topical vitamin D3 analogues
o Hypothyroidism o Intralesional IFN-α
o Chronic renal disease 305. On culture reveals klebsiella best treatment is
o Addison disease o Steroid
295. Regarding erythrasma, all true EXCEPT:- o Doxycycline, tetracycline
o Caused by coryenbacterium minutissimum  Retinoid, isotretinoin
o Response to erythromycin o Dapson
 Healing center & vesicles in periphery o Clofozamin
o Coral red in woods light 306. All of the following are RNA viruses EXCEPT:-
296. Axillary lesion that gives coral red fluorescence with o Measles
woods light is:-  V.Z.V
 Erythrasma o Rota virus
o Seborrehic dermatitis o Rhinovirus
o Psoriasis 307. The following are DNA viruses EXCEPT:
297. Tricomycosis axillaries all EXCEPT  Measles.
 Is a superficial fungal infection o Varcella zoster virus
o Is caused by corynebacterium species o Epstein bar virus
o Is often related to poor personal hygiene 308. Which of the following viruses are RNA:-
o Can be associated with hyperhidrosis o Herpes simplex
o Can be treated by nafitine hydrochloride 1%  Hepatitis C
298. Patient has a red maculo-papular rash which begins on o HPV
wrist, ankles and then progress to form petechiae, the o Hepatitis B
most diagnosis o Molluscum contagiosum
 Rocky mountain fever 309. Which of the following virus are DNA virus:-
o Measles o Measles
o Rubella o Rubella
299. Rocky mountain spotted fever (RMSF) is caused by:- o Hand-foot-and-mouth disease
 R. Ricketsii  Infectious mononucleosis
o R. Prowazaki o Hepatitis A
o R. Typhi 310. Regarding herpes simplex all true EXCEPT:-
300. Which of the following organism don’t have systemic  Type I is more recurrent than type 2
involvement like vasculitis and necrotic change o Fever & lymphedema may occur
o Pseudomonas aregonose o Tzank smear is diagnostic
 Group B streptococcus 311. Primary herpetic gingovostomatitis differs from
o Leprosy herpagnia mainly by
| P a g e 16

o The age group involved o Rubella


o Absence of systemic illness in the later  All of the above
o Presence of vesicles in the former 322. Measles is infectious
 More posterior location of lesions in the latter o After 2 days of contact with household relatives
o None of the above o From the appearance of the enanthen
312. Recurrent vesicles on penis o From the appearance of exanthema till skin
 HSV desquamation
o Chancre o Three days before the prodrome till the enanthem
o Chancroid  From the prodrome till 5 days of the exanthema
313. Nerve most commonly affected in herpes zoster:- 323. The most common dermatophyte worldwide is
 Thoracic o T. Mentagrophytes
o Trigeminal  T. Rubrum
o Lumbar o M. Canis
314. A man 69 years old with herpes zoster, which of the o E. Floccosum
following is an indication for ophthalmic consultation:- 324. All type of fungal microorganism diagnosis with
o Involvement of maxillary nerve wood’s light give florescence except :
o Involvement of auditory nerve o T. Sheonlline
 Involvement of the tip of the nose  Violisicum
o Facial paralysis o M. Canis
315. Varicilla complication in adult most common 325. A fungal culture grew light brown, cottony aerial
o Hepatitis mycelia that became powdery in the center. There were
o Encephalitis numerous spindle shaped multiseptate. Microconidia
 Varicilla pneumonia organism is
o Reye’s syndrome o T. Tonsurans
316. Disease which may present as grouped vesicles and o T. Mentagrophytes
bullae include:- o T. Rubrum
o Herpes simplex  M. Canis
o Herpes zoster o M. Audouinii
o Dermatitis herpetiform 326. What is the slowest growing zoophilic fungus in
o Adult linear iga bullous dermatosis tissue culture ?
 All of the above o T. Equinum
317. About verruca vulgaris all are true EXCEPT:-  T. Verrucosum
 It is caused by HTLV in virus o T. Mentagrophytes
o Some of the lesion may resolve spontaneously o M. Canis
o Some serotypes of the causative virus is potentially 327. Which of the following organisms is MOST likely to
oncogenic cause tinea capitis in adult ?
o Genital warts can be treated by podophylline o Microsporum audouinii
318. Adult molluscum contagiosum is common to:- o Trichophyton mentagrophytes
o Mucous membranes o Epidermophyton floccosum
 Genitalia & nearby areas  Trichophyton tonsurans
o Palms o Pityrosporum orbicularis
319. Giant molluscum contagiosum in an 328. One of the following fungi is characterized by
immunocompromised HIV-infected individual can be endothrix type hair invasion:-
treated by  Trichophyton tonsurans
o Curettage 329. Which is false about tinea capitis:-
o Phenol o Common in children
o Lazarotene 0.1% o Black dot tinea capitis is caused by T tensurans
o Topical tretinoin 0.25%  Kerion is preceded by patch of alopecia
 Topical imiquimod cream 330. Most serious complication of untreated kerion is:-
320. Erythema infectiosum is caused by:- o Secondary bacterial infection
o Varicella zoster virus o Non cicatricial alopecia
 Parvovirus B19  Cicatricial alopecia
o E.B.V o Spread of infection
o H.T.L.V 331. Most cases of tinea barbae are caused by
o Coxsackie virus  Zoophilic fungi
321. Viruses cause Morbilliform eruption include:- o Anthropophilic fungi
o Erythema infectiosum o Geophilic fungi
o Measles o Non dermatophyte fungi
o Pityriasis rosea o Saprophytic fungi
| P a g e 17

332. Fungal infection on subungual Pedis , what is the o Exophiala jeanselmei


causative organism : 342. Regarding Sporothrix schenckii, the following
o T. Rubum statements are true except
 T.tensorance o It is dimorphic
333. Endonyx onychomycosis is caused mainly by  Bloodstream dissemination is common
 T. Soudanes o Lymphocutaneous sporotrichosis is the most common
o T. Rubrum presentation
o T. Mentagrophytes o The lipid compound of the cell wall inhibits
o Asperigillus nigra phagocytosis
o Scopulariopsis brevicaulis o It can grow on brain-heart infusion
334. Onychomycosis of fingernails is best treated by 343. Cryptococcus neoformance is commonly disseminated
o Girisofulvin, 12.5 mg/kg/day for 3 months to:
o Fluconazole 150 mg/week for 4 weeks o Lung
o Ketoconazole 200 mg/day for 6 weeks o GIT
 Terbinafine 250 mg/day for 6 weeks  CNS
o Itraconazole 200 mg daily for 7 months o Kidneys
335. All are useful in diagnosis of Pityriasis versicolor o Bone
EXCEPT:- 344. Cryptococcosis ( Cryptococcus neoformans) is most
o Wood's light commonly affect:-
o Clinical examination o C.N.S
 Skin biopsy o Skin
o Scrub & KOH  Lungs
336. Brown or black patches on palms & soles diagnosed o Bone
tinea nigra is caused by:- 345. The antibiotic least likely to be successful in treating
 Phareoannellomyces werneckii an actinomycetoma is
o Microsporum canis o Tetracycline
o Trichophyton rubrum o Streptomycin sulfate
o Epidermophyton flocusum o Dapsone
337. About fungal infection all are false EXCEPT:- o Rifampicin
o Tinea versicolor if treated never recur  None of the above
o A circinate lesion of healed center & active edge is 346. Treatment of actinomycosis
characteristic of T capitis o Streptomysin + miconazol
o Coral red fluorescence is seen in T capitis o Septrin + streptomycin
 Thrush, paronychia & napkin rash are manifestation of  Penicillin
moniliasis 347. Mycetoma madura foot treatment
338. The relationship of C. Albicans to the skin is best  Surgically excised (main treatment)
described as o Streptomycin or amikacin
o Resident flora  Voriconazoleg 1‫فى عدم وجود رقم‬
o Transient flora o Itraconazole and terbinafine more than ketconazole
o An absolute pathogen 348. Which mycetoma is more inflammatory, more
 An opportunistic pathogen destructive, and invasion of bone at an earlier period?
o An accidental contaminant o Eumycetoma
339. Intertrigo may be secondarily infected with:-  Actinomycetoma
o Streptococci o The mycetoma that remains encapsulated for the
o Staphylococci longest period
o Candida o They mycetoma that produces most fibrosis
o A superficial dermatophyte o None of the above
 All of above 349. The treatment of choice for lobomycosis is
340. The safest drug for treating sporotrichosis during o Itraconazole
pregnancy is  Surgical excision
o Ciprofloxacin o Ketoconazole
o Itraconazole o Terbinafine
o Fluconazole o None of the above
o Ketoconazole 350. The period between exposure to scabies &
 Amphotericin B development of symptoms is:-
341. The causative organism of sporotrichosis is o One day
o Histoplasma capsulacum o One week
 Sporothrix schenckii o Two weeks
o Phialophora verrucosa  Four weeks
| P a g e 18

351. About scabies all true EXCEPT:-  Lupus vulgaris


 More in back o Erythema nodosum
o Iry lesion in burrow o Lieshmaniasis
o Sarcoptes scabiei has 8 legs 362. The most common cause of skin TB is:
o Black fecal concretion of sarcoptes scabiei known as o Scrofloderma.
scybala  Lupus vulgaris.
352. Characteristic lesion of scabies o V.cutis.
 Burrow 363. The most common lns involved in Scrufuloderma
353. Scabies typically spare the flowing sites:- are:-
 Mid upper back o Inguinal lns
o Fingers  Cervical lns
o Buttocks o Mesenteric lns
o Genitalia o Epitroclear lns
354. Scabies differs in infants form adults in all EXCEPT:- 364. The principal reason for using a combination of drugs
o 2ry infection is more common in the chemotherapy of TB is
 Burrows are rare o To prevent side effects
o Wide spread involvement in atypical sites as palms, o To ensure effect against other mycobacteria
soles & neck  To avoid drug resistance
o Presence of vesicles & bullae o To prevent overgrowth by opportunistic pathogens
355. When treatment scabetic patient what point u will o For economic reasons
not consider: 365. Thick of nerve and loss of sensation occurs early
o Efficacy and toxicity of drug  TL
o Number of family members affected o LL
 Cost of the drug o BL
o Age of patient o BT
356. The following are true about ivermectin for 366. Leprosy stained with :
pediculosis treatment EXCEPT  Ziehl Neelson stain
o It is not ovicidal 367. Lepra cells or Foam cells of Virchow occur in:-
o Given as 2 doses one week apart o Tuberculoid Leprosy
 Best drug in epidemics o Borderline Tuberculoid
357. A 5 year old child infected with pediculosis best o Borderline Leprosy
treatment is o Borderline Lepromatous Leprosy
 Apply topical for 10 minutes and wash and then fine  Lepromatous Leprosy
combing 368. Foam cells are seen in:-
o Apply topically overnight then fine combing o SLE
o Combing with electronic device o Scleroderma
358. Fleas are vectors for all of the following diseases  Lepromatous leprosy
EXCEPT o DM
o Typhus 369. Neural signs of leprosy:-
o Fungiasis o Anesthesia
o Bubonic plague o Muscle wasting
 Onchoceriasis (african river blindness) o Trophic changes
o None of the above o Nerve enlargement
359. A dermatologist may select the following therapies to  All of above
treat cutaneous Lieshmaniasis EXCEPT 370. Mitsuda test is completely done in:
o Cryotherapy o 3 days
o Parenteral pentostam o 7 days
o Oral Itraconazole o 10 days
 Systemic corticosteroids o 15 days
o Local application of heat  28 days (3-4 wks)
360. Therapeutic modalities of Lieshmaniasis include all 371. Lepromatous leprosy is characterized by all EXCEPT
EXCEPT o Numerous symmetric erythematous or shiny ill-defined
o Rifampicin 600-1200 mg/day for up to 3 months macules, papules or nodules
o Itraconazole 200 mg/day for 4-8 weeks o Affection of internal organs
o Ketoconazole 200 mg 3 times daily for up to 3 months  Strongly positive lepromin test
o Rifampicin + intralesional SSG o Vacuolated macrophages in histology
 Topical 5% fluorouracil o Peripheral nerve involvement
361. Lesion upon pressure by a glass slide give apple jelly 372. Leprosy best diagnostic tool is
appearance is:- o CBC
| P a g e 19

o Slit skin smears o Early congenital syphilis


o Lepromin test o Late congenital syphilis
 Skin biopsy o 3ry syphilis
o Histamine test 383. Patient have painless ulcer in cronal sulcus since 2
373. Retired, endontulous 60 years old male lives alone, weeks what is diagnosis
eating only biscuits soaked in water , he developed  Syphilis
extensive ecchymotic patches , perifollicular o HSV
hemorrhagic purpuric lesions , what is the diagnosis : o Chancroid
 Scurvey o T.B
o Senile purpura 384. Syphilis:-
o Leakemia o Iry stage characterized by painful ulcers on genitalia
o Lp  2ry stage typically affect palm & soles
o Psoriasis o 3ry stage is characterized by extensive scarring
374. Leprosy bactericidal drug 385. Oral lesion in 2ry syphilis is in the form of:-
 Rifampicin  Mucous patch
o Cloforzemine o Erythema
o Dapsone o Vesicles
o Minocycline 386. Transmission of syphilis to the fetus mother occurs:-
375.Erythema nodosum may show all of the following o During 1st trimester
EXCEPT  During 2nd trimester
 Ulcer o During 3rd trimester
o Erythema o During labor
o Hotness 387. Early congenital syphilis manifestation appear
o History of drugs during:-
376. Drug choice in erythema nodosum leprosum:- o 1st week of life
o Dapsone o 2nd week of life
 Thalidomide  6th week of life
o Tetracyclines 388. Early sign in congenital syphilis is –
o Corticosteroids  Syphilitic rhinitis
377. Causes of Saddle nose o Syphilitic rash on palms & soles
o Lepromatous leprosy o Condyloma lata
o Wegner’s granulomatosis o Genital chancre
o Congenital syphilis 389. The MOST common manifestation of congenital
 All of above syphilis is:-
378. Saddle nose deformity is found in the following o Fissures of the lips and anus
EXCEPT:  Copper red macules and papules of the palms, soles and
 Secondary syphilis diaper area
o Multicentric reticulohistiocytosis o Mucous patches
o Relapsing polychondritis o Condyloma lata
o Leprosy o Syphilitic pemphigus
379. Stds all except:- 390. The least common cutaneous manifestation of early
o Penile molluscum contagiosum congenital syphilis is:-
o Genital wart o Rhagada
o Herpes simplex genitalis  Condyloma lata
 Scrotal herpes zoster o Rhinitis
380. Rieter syndrome treatment : o Rash in palms and sole
 NSAID 391. Which condition is not characteristic of early
o Systemic steroid congenital syphilis
o Methotrexate o Weinberger sign
o Dapson  Jaquet erythema
o Colchisin o Syphilitic dactylitis
o Thalidomid o Pseudoparalysis of parrot
381. Treatment of Rieter in children o Condyloma lata
o Tetracycline 392. The first serological test for syphilis to become
 Azithromycin positive in primary syphilis is usually:-
o Ketoconazole o TPHA
382. The most infectious stage of syphilis is:-  FTA
o 1ry syphilis o VDRL
 2ry syphilis o Wassermann reaction
| P a g e 21

o T. Pallidum complement fixation test o Bejel


393. VDRL is usually +ve in o Pinta
o 1ry S o All of the above
 2ry S  None of the above
o 3ry S 403. Yaws is caused by Treponema pallidum subsp
o Latent o Endemicum
394. The earliest +ve test syphilis is:-  Pertenue
 FTA-ABS o Carateum
o RPR o Vincenti
o TPHA o None of the above
o All of the above 404. Yaws what causative organism
395. Patient has asymptomatic papule & plaque on trunk o T. Palleduim
back sole a generalized lymphoadenompathy after taken  Ssp pertenue
treatment. Which antibiotic he takes. 405. Pinta is unique among the treponematoses as it
 Pencillin  Has only cutaneous manifestations
o Erthromycin o Is found on the Eastern hemisphere
o Clindamycin o Affects mainly the elderly
o Dapson o Is not curable
396. Clues to the diagnosis of late syphilis include o All of the above
o Chancre scar 406. Pinta causative organism
o Nuchal leukoderma  T. Carateum
o Macular atrophy 407. Endemic syphilis (bejel) (Endemicum)
 All of the above  Is prevalent in dry arid climates
o None of the above o Was eradicated from Africa in 1963
397. The drug of choice for non-venereal treponematose is o Affects males more than females
o Penicillin G procaine o Tends to affect children between 2 months to 2 years of
 Penicillin G benzathine age
o Erythromycin o Can be transmitted through breast milk
o Chloramphenicol 408. Mixed chancre on the penis may be due to syphilis
o Tetracycline and:-
398. Penicillin:- o Staphylococcal infection
o Is the only effective treatment for syphilis o Streptococcal infection
o Is effective treatment for non-gonococcal urethritis  Haemophilus ducreyi
 Is ineffective against B-Lactamase producing strains of o Pseudomonas infection
gonorrhea 409. The incubation period for chancroid is:-
o Must be given daily to treat uncomplicated syphilis o 1-2 days
successfully  1-5 days (2-4)
399. Penicillin:- o 5-10 days
o The only treatment of syphilis o 7-14 days
o Used for resistant penicillinase producers Bactria o 14-28 days
 Used for treatment of syphilis as penicillin G 410. LGV may show
o All true o Multiple painfull ulcerating
o All wrong o Multiple erosion
400. The most frequent gummatous involvement in  Disapear 7 day
abdominal syphilis is o 4 week
o Gastric 411. Gonorrhea most common site for discharges
o Intestinal  Cervix
 Hepatic o Urethra
o Splenic o Post vaginal fornix
o Pancreatic 412. The most common site for taking swab to diagnose
401. Non-venereal treponematoses in comparison with female gonorrhea is:-
venereal syphilis o Urethra
o Are caused by distinct organisms o Vagina
o Do not have a relapsing course  Cervix
o Are best treated with aminoglycosides o Labia minora
 Tend to affect predominantly children 413. The most suitable schedule for treatment of
o Are generally less destructive gonococcemia is:-
402. The TPI test will distinguish syphilis from o Procaine penicillin G 4gm
o Yaws
| P a g e 21

 Ceftriaxone 1gm/day IV for 48h after improvement + o Lymphoblastic and histiocytic


ciprofloxacin 500mg orally twice daily for 7 days o Fibroblastic and granulocytic
o Doxycycline 100mg twice daily for 15 days o Neuroblastic and endothelial
o Erythromycin 500mg 4 times daily foe 10 days o Endothelial and neuroblastic
414. The most suitable schedule for genocaceiemia 423. Classic Kaposi sarcoma occurs most commonly in:-
o 4 g procaine o Africans
 Cefitroxeno 1gm/7days  Jewish
o Tetracycline 2g/day -15 days o Southern & Eastern European
o Doxycyline 100mg twice /day – 15days o Asian
o Erythromycin 1g/day – 10days 424. Classical kaposis sarcoma occurs most common in
415. Gonococcemia can be most definitely distinguished  Mediterranean and ashkenazi jewish descent
clinically from chronic meningococcemia on the basis o Negroids in central Africa
of 425. The site most frequently involved KS is
 Systemic toxicity and fever o Face
o Hemorrhagic cutaneous lesions around joints  Legs and feet
o Arthralgia o Lymph nodes
o Tenosynovitis o Liver
o None of the above o Bone
416. A culture of N. Gonorrhea can be most clearly 426. Kaposi can manifest in young African as
distinguished from one of N. Meningitis on the basis of o Benign nodular
o Growth on Thayer-Martin medium o Aggressive
o Growth upon incubation in an elevated CO2 o Florid
atmosphere  Lymphadenopathic
o Oxidase test 427. 19 yrs. Old African male with +ve HIV & suffering
 Fermentation of sugar from Kaposi sarcoma, what is the clinical presentation
o All of the above of the sarcoma:-
417. Reverse transcriptase is characteristic to one of the  Nodular lesion
following:- o Macular patches
 HIV infection o Hyperpigmented brown maculo-papular lesion
o HSV o Ulcerative lesion
o EHV o All of the above
418. In HIV-infected patients, one of the following is NOT
a clinical subtype of anal intraepithelial neoplasm 428. KS with AIDS
o Bowenoid o Lesion usually pruritic
o Verrucous o Tumor never metastasis
 Porokeratotic  EBV nearly always found within the tumor cell
o Leukoplakic o Number of lesion decrease during period of
o Erythroplakic opportunistic infection
419. In general, immunodeficiency should be suspected 429. Kaposi sarcoma in HIV patient most commonly
when a patient has:- associated with:-
o Recurrent infections of increased duration and severity  EBV
o Recurrent infections with unusual organisms o HCV
o Incomplete clearing of infections of poor response to o Molluscum contagiosum virus
antibiotics 430. Which of the following are more affected in HIV
 All of the above associated Kaposi sarcoma:-
o None of the above  Lungs & GIT
420. HIV patient develop lipotrophy of cheeks what the o Lungs & Urinary bladder
filling material to use o GIT & Urinary bladder
o Hyaluronic acid o Urinary bladder
o Human collagen 431. Kaposi's sarcoma AIDS-related:
 Poly-L-lactic acid (Sculptra). o Always painful lesions
421. Etiological agent of Kaposi sarcoma is:-  Mimic bacillary angiomatosis and dermatofibroma
o Herpes simplex virus type 1 o E.barr virus always within the tumor
o Herpes simplex virus type 2 o Lesions development decrease when opportunistic
o Herpes simplex virus type 6 lesions increase
 Herpes simplex virus type 8 432. Staph toxin in (pemphigus vulgaris) because it
422. The main proliferative cellular elements in Kaposi  Dsg I
sarcoma are o Dsg III
 Endothelial and fibroblastic o Dsc I
| P a g e 22

o Dsc III o Cyclosparin A


433. Regarding pemphigus vulgaris all are true EXCEPT:- 443. Patient with pemphigus not responds to steroid which
o Immune complexes are seen in the intercellular spaces drug can be used adjuvant
 Subepidermal bullae formation  Cyclophosamid, azathioprine
o Commonly involves mucous membranes  Cyclosporine
o IgG is found in both involved & clinically free skin o Methotraxate
434. Paraneoplastic pemphigus is characterized by the o Gold
presence of igg antibodies against of the following o Dapsone
antigens ? 444. Treatment of choice for treating IgA pemphigus is
o Desmocollin 1 o Colchicines
o Desmoplakin 1 o Potent topical steroids
 Desmoglein 3 o Minocycline
o Desmocollin 3  Dapsone
435. Oral lesion in pemphigus vulgaris:- o Nicotinamide with tetracycline
o 10% 445. The immunoglobulin precipitated in paraneoplastic
o 30% pemphigus
o 20% o Iga
 50% o Igg
436. All of the following diseases, there are deposition of  Igg and C3
igg at BMZ EXCEPT:- o Igm
 Pemphigus vulgaris o All of the above
o Dermatitis herpetiform 446. Of the following, the malignancy that has been
o Bullous pemphigoid associated most frequently with paraneoplastic
o Herpes gestation pemphigus:-
437. The worst prognosis is expected in:- o Gastric carcinoma
o Bullous pemphigoid o Hepatoma
o Pemphigus foliaceous o Malignant carcinoid
 Pemphigus vegetans  Chronic lymphocytic leukemia
o Dermatitis herpetiform o Adenocarcinoma of the breast
o Herpes (pemphigoid) gestations 447. Immunoglobulin in bullous pemphighoid are located
438. Igg deposition occurs at BMZ in all of the following in
EXCEPT: o Desmosome
 Pemphigus vulgaris  Hemidesmosome
o Dermatitis herpetiform o Anchoring fibrils
o Bullous pemphigoid o Anchoring fibers
o Herpes gestation 448. The sites of immune globulins in bullous diseases
o EBA  Inter cellular and liner basement membrane deposition
439. Pemphigus foliaceous:- of igg mainly
 Have no racial base 449. Immunoglobulin's in bullous pemphigoid are:-
o More in Jewish  Igg & C3
o Commonly have mucosal lesion o Igm
o All of above o Iga
440. All the following lesions are present in Pemphigus 450. A case of violacous hyperpigmented patch over lower
foliaceous EXCEPT – leg after and he developed tense bullae over norma
 Mucosal lesions. skin and away from it… DIF is positive what is the
441. P. Foliaceous like lesions on chest with erythema and diagnosis
scaling on face we can give all EXCEPT o Bullous lichen planus
o Cyclophosphamid  Lichen planus pemphigoides
o Azathioprine 451. Treatment of bullous pemphigoid
o Dapsone o Azathioprine
 Thalidomide o Cyclosporine
442. A-35-year-old patient with pemphigus vulgaris is o Cyclophosphamid
being treated with prednisalone 40 mg daily for last 6  Corticosteroid
months with inadequate control of disease activity. 452. The treatment of cicatricial pemphigoid include:
Which of the following adjuvant would be most o Predinsolone
effective for this patient ? o Dapsone
o Methotrexate o Predisolone+cyclophosfamide.
 Cyclophosphamide o Predinsolone+ azathioprine.
o Colchicine  All above.
| P a g e 23

453. Most steroid sparing drug used in bullous o Prednisolone


pemphigoid o Prednisolone + azathioprine
o Mycophenolat mophetyl o Prednisolone + Cyclophosphamide
 Azathioprine o Dapsone
o Ritiximab  All of above
o Mtx 464. All have igg deposition along BMZ by DIF
454. What is the best cytotoxic drug can be used in EXCEPT:-
bullous pemphigoid patient. A severe ocular o BP (Bullous Pemphigoid)
damage o SLE (Systemic Lupus Erythematosis)
o Cs o EBA (Epidermolysis bullose Acquired)
o Azathioprine  DH (Dermatitis Herpetiform)
 Cyclophosphamide 465. Male with severely pruritic papules and tense vesicles
o Infliximab on the back and shoulder and tense urticated plaque on
455. Herpes gestation what is TRUE abdomen what is probable IG if no oral lesion (DH)
o Cannot occur without pregnancy  Granular Ig A
 HG antigen is always found in DEJ o Igg
456. Pemphigus gestationis is deposition of whish ig in o C3
dermoepidermal junction by immunofluorescence: 466. The most reliable test for diagnosis of dermatitis
o Ig A herpetiform is:-
o Ig G o Hematoxylin/eosin stain of perilesional skin
o Ig M o Indirect immunofluorescence demonstrating iga
 C3 antibodies to reticulin
o Ig C4 o Immunoblotting study demonstrating presence of gluten
457. HG:- antigen in skin
o Occurs during pregnancy only  Direct immune fluorescent staining of perilesional skin
o Carry significant effect on mother morbidity and infant showing dermal papillary iga deposits
morbidity and mortality o Detection of HLA-Qw2 antigen
o Differs completely than bullous pemphigoid 467. Best reliable test for DH diagnosis is –
 Immunoglobulin deposits at Lamia Lucida  Direct immunofluorescence
458. H. Gestations what is TRUE:- o Of non-lesional skin
 HG factor present in the lamina lucida. 468. All are true regarding DH EXCEPT:-
o HG morbid to the mother  Good response to steroids
o HG morbid to the fetus o Symmetrical lesions
o HG morbid to both mother and fetus. o Immune florescence show iga antibodies in uninvolved
459. Herpes gestation : all EXCEPT skin
o Occurs only during pregnancy o Severely pruritic
o Occurs first time in multipara only 469. In patient with dermatitis herpetiformis may be
o Completely different in histopathology from bullous treated with sulfapyradine 1-1.5 g/day if:
pemphigoid o Dapsone is not tolerated.
 All of the above o In elderly patient.
o None of the above o In patient with cardiac disease.
460. One of the following differentiates H.G from PUPPP:-  All above.
o PUPPP occurs in the 1st trimester 470. 4 years old boy presented by 2 months duration of
o PUPPP occurs in the last trimester Perioral & napkin annular vesicular eruption. The most
o PUPPP has intense pruritis probable diagnosis is:-
 PUPPP has urticarial plaques & papules  Iga bullous dermatosis
461. A30-year-old patient who is 6 months pregnant o Impetigo contagiosum
presents with multiple urticated papules and plaques o Contact dermatitis
and tense bullae on the obdomen and legs. Itching is o Chronic herpes simplex
severe . What would skin biobsy from the bulla show ? 471. IgA linear dermatosis:-
o Intrapidermal split with Tzanck cells o Associated with gluten enteropathy
o Subcorneal split with numerous red blood cells  Dapsone is needed in treatment
o Papillary dermal neutrophilic microabcesses o Carry bad prognosis
 Subepidermal split with eosinophils o Bullous formation uncommon
462. The treatment of herpes gestation 472. In linear iga bullous disorder , what is the initial
o Topical steroids dose of Dapsone in mg/kg?
 Systemic steroids o 0.25
o Anti histamine  0.5
463. Treatment of Cicatricial pemphigoid o 1.0
| P a g e 24

o 2.0 483. Pt. Photo sensitivity malar rash and have develop
473. Chronic bullous of childhood dermatosis closely lesion like pemphigus have sterile pustule in palm
associated with: *sole which not you give
o HLA – B27  Acetritin
o HLA _B13 o Azathioprin
o HLA – B17 o Cyclophosphamide
 HLA – DR3 484. Female patient with recurrent malodorous lesions
o HLA – DR4 around her neck, in axilla, groin histopathology
474. Chronic bullous dermatosis of childhood (CBDC) is reveals familial benign pemphigus what is best ttt:
closely associated with:- o Dapson
o HLA-B27  Antiherpetic/anti- infective drugs
o HLA-B13 o Steroid
 HLA-B8 485. The inheritance of benign familial pemphigus is
o HLA-DR4 o Autosomal recessive
475. Treatment of choice of CHDC is:- o Autosomal dominant
 Dapsone  Autosomal dominant with variable penetrance
o Corticosteroids o X-linked dominant
o Sulphapyridine o X-linked recessive
o Plasmapharesis 486. EBS (simplex) defect
476. The organism isolated from subcorneal pustular  Keratins 5 and 14
dermatosis is:- o Collagin
o Staphylococci o Hemidosoms oto hmunity
o Streptococci 487. EBA is commonly characterized by:-
o Mycobacterium tuberculosis o Vesicles & bullae following trauma
 None of the above o Tends to occur in frictional skin areas
477. Subcorneal pustular dermatosis is best treated by o Mucosal lesions are common
o Tetracycline  All of the above
o Erythromycin 488. The clinical presentations of EBA include:-
o Isotretinoin orally o The "classical" presentation
 Dapsone o A bullous pemphigoid like presentation
o Topical corticosteroids o A cicatricial pemphigoid like presentation
478. The underlying etiology of Hailey-Hailey disease is a  All of the above
mutation in the o None of the above
o ATP 2A2 gene 489. A 6 – year- old boy with dystrophic epidermolysis
 ATP 2C1 gene bullosa with recurrent blistering is treated with low
o Desmoglein 1 gene dose of isotretinoin for controlling his disease and
o Desmoglein 3 gene reducing the risk of squamous cell carcinoma. What is
o Desmoplakin 1 gene the most likely cutaneous complication that would
479. Differential diagnosis of H & H disease is:- occur on increasing the dose of the drug
o Impetigo  Mechanical fragility
o Pemphigus vulgaris o Photosensitivity
o Darier's disease o Xerosis
o Pemphigus vegetans o Ulceration of the lesions
 All of the above 490. Different between DLE & SLE is
480. Treatment of hailey - hailey disease :  Atrophy
 Steroid o Photosensitivity
 Dapson o Erythema
o Azathioprine o Scaling
481. Haily-Haily disease treated by ‫كله صح بس بالترتيب‬ o Telangectasia
 Steroid 491. The most important histological features of DLE, for
 Antibiotic diagnosis is
 Dapson o Hyperkeratosis
482. Treatment of grover disease (transient acantholytic o Malpighian layer atrophy
dermatosis)  Liquefaction degeneration of the basal cell layer
 Isotritinoin for systemic o Patchy lymphocytic infiltrate
 Steroid for topical o Endothelial proliferation
o Dapson 492. Diagnosis of DLE is confirmed by:-
o ESR
 Skin biopsy
| P a g e 25

o Photopatch test o Expose skin sites


493. Complications of scalp DLE include all EXCEPT  Sites that are intensively treated
o Ulceration 504. Positive lupus band- test in the normal sun exposed,
o Koebnerization occurs with :
o Metastatic squamous cell carcinoma o DLE
o Cosmetic disfigurement  SLE
 Premature canities o Pemphigus
494. Which of the following clinical findings distinguishes o Scleroderma.
scalp DLE from follicular LP? 505. SLE all are false EXCEPT:-
o A positive pull test result for anagen hairs o C3, C4 ↑↑
o Response to topical corticosteroids o No ANA +ve
o Development of lesions in non-scalp areas  Photosensitivity
 Erythema and scale affecting the center of the lesion in o Strawberry tongue
active disease 506. Severity of SLE is gauged
o All of the above o Leukocyte
495. Drug induces SCLE o Cd4
o Hydrachlorothiazid o Ss dna ‫ للخطورة‬risk
 Hydralazin  Ds dna ‫ للمراقبة‬monitoring
o Hydantoin 507. Good prognosis is SLE depends on:-
o Procaine o Liver affection
496. All are manifestations of SLE except:-  Renal affection
o Malar flush o Lung affection
o Photosensitivity CNS affection
 Telangectasia 508. One of the most important factors in determining the
o Chronic discoid lesions prognosis of SLE ?
497. Drug which most induce SLE o Initial ANA titer
o Penicillin o Severity of arthritis
 Procainamide o Sex of patient
o Tetracycline  Degree of renal involvement
o Chlore hexidine o Leukocyte count
498. Drug most proven causes SLE 509. The renal manifestation most often associated with a
o Penicillamine poor prognosis in SLE is
 Isoniazide o Glycosuria
499. In SLE, the most important pathologic feature in o Proteinuria
connective tissue is o Pyuria
o Inflammatory infiltrate  Nephrotic syndrome
o Edema o None of the above
o Fibrosis 510. SLE in childhood
 Fibrinoid change o Is more common in boys than girls
o Proliferation of fibroblasts o Seldom manifests cutaneous lesions
500. The cutaneous marker of high risk in SLE is – o Often has negative fluorescent ANA tests
o Oral ulcers  Has a worse prognosis than in adults
o Leg ulcers o Does not occur
o Development of multiple dermatofibroma lesions 511. A patient with photosensitivity, facial rash and
 Levido reticularis investigation showed +ve anti dsdna, ANA. Best
501. %of oral lesion in SLE treatment to start with is
o 7% o Predenisolone
o 8%  Hydroxychloroquine
o 10% 512. The pattern of nuclear antibody that shows speckled
 20% pattern seen in:
502. Most common haematologic finding in SLE is o SLE.
o Lymphopenia o Scleroderma.
o Thrompcytopenia o Dermatomyositis.
o Leucopenia  Mixed connective tissue disease.
 Mild normocytic normochromic anaemia
o Haemolytic anaemia 513. One of the following statements is true:-
503. To assure high sensitivity in taking skin biopsy for  Anti-ndna+ve in SLE patient means active nephritis
immunofluorescence in SLE you have to avoid the o Anti-Jolis+ve in cancer associated myositis
following site o Anti-nucleohistone are +ve in drug included SCLE
| P a g e 26

o Anti ss- DNA+ve means systemic sclerosis o Edema of the upper dermis
514. All true about morphea EXCEPT:- 524. Pseudoscleroderma can be due to all EXCEPT
o Hypopigmentation o Eosinophlic facilities
o Thickening of the skin o Bronchial carcinoma
 Asymptomatic o Hypothyroidism
515. Female patient have multiple skin lesions on o Bleomycin
abdomen and face which is hyper pigmented,  Histidinemia
shiny,devoid of hair,indurated. Histopathology 525. Scleredema subside after
reveals collagen deposit in the dermis what do u  1-2 year
suspect (morphea): o 4-5 years
 25% of pt. Have anti ss DNA o 8-10 years
o 50% have anti-cardolipin 526. Child had indurated swelling on neck upper trunk
516. The most common monoclonal gammopathy after pharyngittis when they consider resolved
associated with scleroderma is:- (scleredema adultorum of buschke)
o Iga  1-2 year
o Igm o 4-5 years
 Igg kappa o 8-10 years
o Igg Lambda o 6-12 monthes
o Ige 527. Eosinophic Fasciitis (Shulman syndrome) treatment
517. The histologic alterations in scleroderma typically o Isotritnoin
begin  Corticosteroid
o In papillary dermis o Retinoid
o In reticular dermis above eccrine glands o Antibiotic
o In perivascular collagen o Cytotoxic drugs
o In the perappendageal collage 528. Treatment for eosinophilic fasciitis
 In the dermal-subcutaneous junction area o Pencillamine
518. Visceral affection of progressive systemic sclerosis  Prednisolone
(PSS) involve:- o Melphalan
o CVS 529. Juvenile hyaline fibromatosis is characterized by
 GIT o An autosomal dominant inheritance
o Lung  Large cutaneous nodules, gingival hypertrophy and
o Dihydrotestosterone joint contracture
519. Most common organ involved in Scleroderma is o The collagen present is type IV
 Esophagus o The main glycosominoglycan present in tissue is
o Colon hyaluronan
o Kidney o All of the above
o Heart 530. Leucocytoclastic vasculitis is due to:-
520. Scleroderma all EXCEPT: o Delayed hypersensitivity reaction
o Causes calcification of the skin  Circulating immune complexes
 Shows periorbital lesion o Immediate hypersensitivity reaction
o Scl 70 531. What not associated with vasculitis
o Raynaunds phenomenon is common than is SLE o Erythema elevatum duntenum
521. About systemic sclerosis all are true EXCEPT:- o Erythema induratum
 Hepatosplenomegaly o Gainuloma fascula
o Swollen hands  Reticular histiocytosis
o Hypopigmentation
o Atrophy of terminal phalanges 532. All are vasculitis disorder EXCEPT
522. Heliotrope eyelids are seen in:- o EED
o DLE o Granuloma faciale
o SLD  Multicent reticulo histocyte
 Dermatomyositis 533. Leukocytocloclastic vasculitis can be caused by the
o Scleroderma following drugs EXCEPT
o Rheumatoid Arthritis o Insulin
523. Histopathology of Dermatomyositis all except o Tamoxifen
 Acanthosis o Sulphonamides
o Hydropic degeneration of basal cell layer o Oral contraceptives
o Flattened epidermis  Cyclosporine
o Accumulation of acid mucopolysaccharides in the 534. All of the following characteristics urticarial vasculitis
dermis lesions except
| P a g e 27

o They favor the trunk over acral surfaces o Systemic steroid


o Posinflammatory hyperpigmentation
 Itching 543. Which treatment seems most effective in treating the
o Duration longer than 24 hours rash of Hench-Schlloein purpura?
o Purpura o Corticosteroids
535. What in urticarial vasculitis o Nsaids
o B cells o Factor XIII
o T cells  Dapsone
 Complement deficiency o Azathioprine
o Ige 544. First line treatment of granuloma faciale is
536. Purpura is seldom thrombocytopenic in origin if the o Oral isotretinoin
platelet count is below o Topical adapalene
o 3000 o Oral doxycycline
o 7000  Dapsone
o 10,000 o Oral corticosteroids for a 5-7 days
o 20,000 545. Which of the following statements is NOT TRUE of
 50,000 Polyarteritis nodosa
537. Henoch-schönlein purpura all EXEPT o Rarely involves the skin except as a benign form
o Iga deposits are specific but not sensitive (not effect limited to skin and subcutaneous tissue
diagnosis or prognosis)  Involves mainly arterioles
o Early steroid treatment decreases clinical manifestations o Exhibits necrosis, inflammation, granulation and
o Relapse occurs in one third of patients fibrosis of all vascular wall layers
o Third cases of patients will remit o Is a focal disease of vessels
 CNS is affected o None of the above
538. What is TRUE concerning renal involvement in 546. Which organ is classically spared by SPAN?
Henoch-Schonlein purpura ?  Lungs
o It is second after skin in frequency of appearance o Kidney
o It tends to occur early in the course o Genitourinary system
o It never progresses to renal failure if the only presenting o Gastrointestinal tract
sign is microscopic hematuria o Central nervous system
 It is unlikely to develop later if renal manifestation are 547. 43 year old patient presented with multiple, bright red,
obsent in the first episode of HSP friable papules of the gingival as well as palpable
o It is always associated with linear deposits of IgA in purpura of the lower extremities. The diagnosis most
kidney biopsy specimens probably is
539. Which of the following statements is true regarding o Cryoglobulinemia secondary to HCV infection
Henoch-Schonlein purpura and renal disease? o Polyarteritis nodosa
o End-stage renal disease is more common occurs in 30%  Wegener granulomatosis
of patients o Churg-Strauss syndrome
 Renal disease is more common in patients with purpura o Microscopic polyarteritis
above the waist 548. Typical features of Waardenburg include all EXCEPT
o Azathioprine is not an effective treatment of nephritis o Dystopia canthorum
o ESRD is more likely to occur in children than in adults o White forelock
o ESRD is more common in the setting of an iga-ANCA o Confluent eyebrows
540. Child with abdominal pain , purpuric lesions and  Low-set ears
joint pain , what the diagnosis o Congenital deafness
o Rheumatic fever 549. Each of the following distinguishes Churg-Strauss
 Henoch-Schenolen Purpura syndrome from WG except
o Kawasaki syndrome o Myocardial involvement
o Eosinophilia
541. Hench schohlein purpura what is TRUE o Long history of asthma
o Recurrent 1/3-2/3 patient o High incidence of anti-mpo antibodies
o Renal affection  Granulomatous inflammation
 IgA perivascular deposition is specific 550. The pathogenic process in Degos' disease is
542. Child with palpable purpura on leg with out  Ischemic infarction
abdominal complication , urine analysis normal , o Necrobiosis
what treatment (henoch–schönlein purpura history) o Hemorrhagic necrosis
 NSAID (first line is symptomatic therapy) o Malignant infarction
o Topical steroid o None of the above
o Dapson 551. Neutrophilic dermatosis include all EXCEPT
| P a g e 28

o Familial Mediterranean fever recurrent bilateral conjunctivitis what is the best


o Behcet's disease treatment: behcet disease
o Sweet syndrome  Steroid
o Pyoderma gangrenosum o Dapsone
 Carcinoid syndrome 562. Pyoderma gangrenosum is commonly associated
552. Pathergy reaction occurs in the following:- with:-
o Behcet's syndrome o Cardiovascular diseases
o Pyoderma gangrenosum  Inflammatory bowel diseases
o Sweet syndrome o Sarcoidosis
 All of the above o Internal malignancy
553. Sweet syndrome not associated with 563. All the following are types of pyoderma
 PCT gangrenosum except one:
o LE o Bullous form
o Myelocytic leukemia o Pustular form
o Rheumatoid arthritis o Ulcerative form
o Inflammatory bowel disease  Psoriasis form
554. There is no relation between sweat syndrome o Vegetative
o Pyoderma gangenrasum 564. Inflammatory bowel disease can be associated with
o IBD the following EXCEPT
o LE o Vitiligo
o Myloproliferative disorder o Psoriasis
 None of the above  Pemphigus foliaceous
555. Sweet syndrome associated with o Arthritis
o Myeloproliferative leukemia )acute myelogenous o EBA
leukemia( 565. What is the common association these 3 disease
o Inflammatory bowel disease EED, PG, SCPD
 Myelocytic leukemia  Inflammatory bowel disease
o Lymphoma o Sweet’s syndrome
 All of the above o HCV
556. Sweet syndrome is commonly associated with:- o DM
 Hematopoietic malignancy 566. Large plaque parapsoriasis is associated with:-
o Malaria  Mycosis fungoids
o Dwarfism o Hodgkin lymphoma
557. Adult male he is febrile having vesiculopapular o Kaposi sarcoma
lesion on left malar area with cervical LN what is 567. Besides the skin, the most frequent organ involved in
the best treatment: sweet syndrome Mycosis Fungoids is the:-
 Steroid o Spleen
o Anti histaminics drugs  Lymph node
558. Percentage of oral lesion in Behcet's disease is:- o Heart
o 10% o Bone
o 30% o Liver
o 50% 568. Resolution of mycosis fungoids lesions under electron
 90% beam therapy can be expected with:-
559. The most classic ocular involvement in Behcet's o Eczematous lesions
disease is o Plaques,
 Posterior uveitis o Erythroderma,
o Conjunctivitis o Tumors
o Retinal detachment  All of these
o Optic atrophy o None of these
o Vitreous hemorrhage 569. The second site include in MF after the skin is
560. In Behcet's disease all the following can occur o Bone
EXCEPT o Interastinal
o Uvietis  Lymph nodes
o Optic neuritis 570. A pathological feature diagnostic of mycosis fungoids
o Keratoconjuctivitis  Pautrier microabsceses
 Urithritis o Atypical histocyte in the dermis
561. Patient comes to ER having fever & history of o Atypical histocyte in the epidermis
recurrent ulcers on tongue , penis and sometimes o Multinucleated histocyte
o None of the above
| P a g e 29

571. The atypical cells in lymphomatoid papulosis stain o Liver


with which of the following markers? o Brain
o S-100  Bowel
o HMB-45 o Lung
 CD30 o Bone
o CD20 581. Least affected site by MM
o CEA o Liver
572. Primary CD30 (Ki-1) anaplastic LCL is characterized o Brain
by all the following except o Bowel
o It affects mostly adults o Lung
o Draining lymph nodes can be involved in 25% of  Bone marrow
patients 582. Malignant melanoma with ↑ S 100 protein high level
 It is highly associated with (2,5) translocation means:
o Spontaneous remission can occur in 25% of patients o Impeding metastasis
o It carries a better prognosis than the systemic form  Metastasis is excising
573. Comments organ affected in Langerhans's cell 583. Increased s100 in melanoma indicates:
histiocytosis is  Metastatic tumor
o Skin o No complete surgical removal of 1ry tumor
o Lung 584. The most common type of malignant melanoma is:
 Skelton (Bone) o Nodular melanoma
o L.N. o LMM
574. Langerhans Cells histiocytosis most complication  SMM
 Skeletal o Acrolentigomaligna melanoma
o Skin 585. Malignant melanoma in female appears on:
o Blood o Upper limb
o Lung o Face
575. Diabetes insipidus is a characteristic feature of o Head & neck
o Eosinophilic granuloma o Chest
 Hand-Schuller-Christian disease  Lower limb
o Urticaria pigmentosa 586. The most common type of malignant melanoma that
o Wegener granuloma strongly related to sun exposure is:
o Lymphatoid granulomatosis o Nodular melanoma
576. Cyanotic areas that appear on sites of dense  LMM
subcutaneous fat as the thigh & lower leg of adult o SMM
female that during winter is due to – o Acrolentigomaligna melanoma
o Acrocyanosis 587. Most nevus cell nevi of the oral mucosa are
 Erythrocyanosis o Junctional
o Livedo reticularis o Compound
o Chilblains  Intramucosal
o Perniosis o Blue
577. Physiological livedo reticularis means:- o Malignant
o Acrocyanosis 588. Typical features of melanocytic nevi include
o Erythrocyanosis  Usually appear in early childhood
o Perniosis o Develop after the age of 40 years
 Cutis marmorata o Juvenile nevi are smooth, papillomatous, hairy nodules
578. The most common cause of livedo reticularis is o Intradermal nevi are circular, brown macules <10mm in
o Connective tissue disease diameter
o Polyarteritis nodosa o 20% lifetime risk of malignant transformation
o Lymphosarcoma 589. Lentigo simplex should be different clinically from:
 Physiologic reaction to cold o Lentigomaligna
o Arteriosclerosis  Freckles
579. The most common sites of malignant melanoma in o Junctional melanocytic nevus
Black & Asians:- 590. Lentigo simplex should be clinically differentiated
o Mucous membranes from:-
o Palms o Lentigo maligna
o Soles o Junctional melanocytic nevi
o Nail  Freckle
 All of the above 591. Small and giant congenital nevi share the following
580. Latest affected site by malignant melanoma histologic features
| P a g e 31

 Nevomelanocytes are present within the lower two third o Filiform wart
of the dermis and within the subcutaneous tissue o Seborrehic keratosis
o No nevomelanocytes are found within hair follicles, o Keratoacanthoma
sebaceous glands, eccrine apparatus, vessel walls, and  All
nerves 600. All is true about cutaneous horn EXCEPT
o The presence of perivascular and perifollicular o Usually it occurs in the elderly
inflammation resembling a figurate erythema o Both sex are equally affected
o All of the above o SCC can be presented at its base
o None of the above  It occurs exclusively on sun-exposed areas
592. The following statements regarding Atypical moles o Skin biopsy with adequate specimen depth is obligatory
are correct except in almost all patients
o They are in blacks and middle eastern populations 601. Which malignant sign is itchy
o They can occur in sun-protected areas  Lasser-Trelait sign
o They often are larger in size than common moles o Erythema gyratum repens
 Lesions have a well-defined edge with uniform o Trousseau sign
pigmentation 602. Pruritis is the most common symptom in
o Lesions can progress to melanoma o Acanthosis nigricans
593. All are changes suggestive malignancy in pigmented  Lasser-Trelait sign
navi except;- o Trousseau sign
o Presence of new pigment around a nevus o Hypertrichosis
o Development of uneven surface pigmentation 603. Sudden appearance of numerous itchy Seborrehic
o Sudden enlargement of previous quiets mode keratosis in adult is called:-
 Increase in hair in hairy nevus  Sign of Lesser-Trele
594. Becker nevus is o Auspitz sign
 Hyperpigmented hairy epidermal nevus o Darier's sign
o Premalignant lesion o Nicolesky's sign
o Variant of nevus of Ito 604. Malignant acanthosis nigricans is associated with
o Collage nevus malignancy in:-
o Persistent extensive junctional nevus  GIT
595. Nevus arenus in pregnancy o CNS
 Is of only cosmetic significance o Kidney
o Indicates a trophoblastic tumor o Lungs
o Is quite rare 605. What is painful:-
o Seldom resolves after pregnancy o Eccrine poroma
o Indicates a hepatic disease o Syringoma
596. The most common location for the appearance of the o Steatocystadenoma papilliferom
common blue nevus is:-  Eccrine spiradenoma
o Head and neck 606. The most common cutaneous neoplasm with
o Torso ossification is:-
 Hands and feet o Nevi without inflammation
o Genitalia o Nevi with inflammation
o Buttocks o Basal cell carcinoma
597. All is true about desmoids tumors EXCEPT o Chondroid syringoma
o They are benign fibrous neoplasms  Pilomatrichoma
o They arise most commonly from the rectus abdominus 607. Possible cutaneous manifestations of systemic
in postpartum women malignancy include all EXCEPT
o They are most common in persons aged 10-40 o Generalized pruritus
o They are locally aggressive o Acanthosis nigricans
 They are resistant to radiation therapy o Late-onset dermatomyositis
598. A man presented with a nodule on the side of the neck o Generalized hyperpigmentation
with a fistula histopathology showed hyper keratosis  Seborrehic dermatitis
and papillomatosis and a cratwe with upper portion 608. A 50- year old woman and her mother have multiple
lined with squamous epithelium and lower portion lined cutaneous and uterine leiomyoma. She has an increased
with apocrine epithelium diagnosis is risk for
o Parotid gland tumor o Breast carcinoma
o Paget's disease o Thyroid carcinoma
o Trichoepithelioma o Ovarian carcinoma
 Syringocystadenoma papilliferum  Renal cell carcinoma
599. Cutaneous horn may arise from o Colon carcinoma
| P a g e 31

609. Glomous tumor arises from:-  Lower lip


o Nerve cell 620. Which of the following treatments used for
o Nerve fiber heamangioma may cause dysplastic paraplegia
 Smooth muscle cell(Glomus body)  Interferon alpha
o Fibrocyte o Prednisolone
610. Granular cell tumor affects mainly:- o Embolization
 Tongue o Radiation
o Palms 621. Involution in haemangioma of infancy is most
o Lobule of the ear typically heralded by
611. Treatment of keratoacanthoma o Ulceration
 Excisional biopsy  Color change from bright to dull red
o Radiotherapy o Increased prominence of telangiectases
o Punch biobsy o Rapid decrease in size
o Jucisiomol biobsy o Pain
612. Face and scalp crusted papules on removal leaves 622. Which category best describes a plaque like
erythematous surface best least invasive medicine to hemangioma of infancy distributed along an
remove it embryologic mesenchymal prominence?
o Retinoid  Segmental
 Cryotherapy o Developmental
o Surgical o Problematic
o Currete o Syndromic
o 5FU o Multifocal
613. A constant feature of both actinic keratosis and 623. An infant with several cutaneous haemangioma of
leukoplakia is infancy (HOI), tachypnea, and increasing abdominal
o Epidermal hyperplasia girth is likely to demonstrate
o Acantholysis o Absent femoral pulses
o Parakeratosis o Digital clubbing
 Atypical epidermal cells o Heure-positive stools
o Spongiosis  Hepatomegaly
614. Histopathology of stucco keratosis is most closely o Splenomegaly
resembles 624. Regarding HOI, ulceration is most likely to occur in
o Verruca vulgaris what phase of growth?
o Actinic keratosis o Involuted
o Lichen planus o Involuting
 Epidermodysplasia verruciforms  Proliferating
o Dariers disease o Plateau
615. Characteristic features of bowen disease include all o Nascent
EXCEPT 625. A 7-year-old child presents whith port wine stain of
o An uncommon condition occurring mainly in the the left cheek and verrucous epidermal nevus on the
middle and old age leg . What is the diagnosis ?
o Can occur on top of porokeratosis and  Phakamatosis pigmentovascularis I
epidermodysplasia verruciforms caused by HPV-5 o Phakamatosis pigmentovascularis II
o Ulceration of lesion often is a sign of invasive growth o Phakamatosis pigmentovascularis III
 Transformation into SCC occurs in 20% -30% of cases o Phakamatosis pigmentovascularis IV
o Early treatment with PDT using topical 5-ALA yields 626. Kazbach merrit
high cure rate with excellent cosmetic results  Haemangioma
616. What is not predisposing SCC 627. Best treatment of Angiokeratoma of srotum
 Old scar o Surgical excision
o Sunexposed  Cryosurgery
o Actinic keratosis o Topical treatment
617. SCC the least location 628. The most common type of leukemia in adults is:-
o Scar o Granulocyctic leukemia
 Trunk  Chronic lymphocytic leukemia
618. Pre SCC condition is : o Acute myelomonocytic leukemia
 Erythroplasia of queyrat o Myelocytic leukemia
619. SCC is most likely to metastasis from: 629. The most frequent non specific feature of leukemia
o Tragus of the ear is:
o Upper lip o Bulla.
o Side of the nose o Erythroderma.
| P a g e 32

 Pruritus. o None of the above


o Vasculitis. 640. Hair loss may be caused by all EXCEPT:-
o Paller. o Cyclophosphamide
630. Patient has coin size lesion severe itch subacute  Tetracycline
erythroderma on neck extrematisis no history of o Heparin
atopy what diagnosis o Contraceptive pills
o Numeuler eczema 641. Telogon effluvium occurs after all EXCEPT:-
 Cuteneos T cell lemphoma o Physical & emotional stress
631. Gingival hyperplasia is characteristic of o Fever
o Acute lymphocytic leukemia  Methotrexate therapy
o Chronic lymphocytic leukemia o Severe nutritional deficiency
o Acute granulocytic leukemia 642. Eyebrows may appear not to regrow after shaving
o Chronic granulocytic leukemia because
 Acute Myeloblastic Leukemia o Their growth rate is extremely low
632. Gingival hyperplasia is most commonly occurs with:-  Their telogen phase is relatively long
o Hodgkin's lymphoma o Shaving damages the matrix
o MF o Shaving induces the telogen phase
 Acute myelomonocytic leukemia o None of the above
o GVHD 643. Alopecia due to antimicrobial drugs is called:-
633. The most common cutaneous lesion of Hodgkin  Telogon effluvium
disease is o Anagen effluvium
 Papules o Catagen effluvium
o Tumors o None of the above
o Exfoliative dermatitis 644. Anagen alopecia caused by:
o Ichthyosis  Cytotoxic drugs.
o Ulcers o Fever.
634. A case of pruritus, icthysosis & hyperpigmentation is 645. A 6 years old boy under chemotherapy presented by
more likely correlated with:- scalp hair fall diagnosis is:-
 Hodgkin lymphoma o Alopecia totalis
o Mycosis fungoids o Telogon effluvium
o Psoriasis vulgaris  Anagen effluvium
635. Skin disorders that have been associated with MM o Alopecia areata
include all of the following except 646. Regarding Cicatricial alopecia all are true EXCEPT:-
o Scleroderma o DLE
o Scelromyxedema o HZ
 Polyarteritis nodosa  Oral contraceptives
o Subcorneal pustular dermatosis o LP
o Sweet syndrome 647. Alopecia areata:
636. Hair development in utero start at o Cause scaring alopecia
o 7th w o Presented with patchy hair loss with question mark
 9thw  There is a potential for regrowth of hair
o 11thw 648. Patient with cicatriatial alopecia & surround with
o 13thw pustules, papules, on biopsy prolonged remission
637. The daily growth rate of scalp hair is approximately:- neutrophiles &eosinophiles infection from one year
 0.3 mm (Folliculitis Decalvan)
o 0.1 mm o Girisofulvin
o 1 cm  Rifampicin
o 2 cm o Steroid
638. Ophiasis is 649. About alopecia areata what is TRUE:-
 A disease of the hair o Nail involvement is 10%
o A parasitic disease o Increase incidence with atopic dermatitis
o Growing together of the eyebrows o +ve family history in 25% of cases
o A disease of the eye  All of the above
o None of the above 650. Bad prognosis in alopecia areata:-
639. The "hairs" of black hairy tongue are o Ophiasis
 Hyperplastic filiform papillae o Prepubertal with duration longer than 5 years
o Hyperplastic fungiform papillae o Onychodystrophy
o Fungal hyphae o Atopic patient
o Colonies of bacteria  All of the above
| P a g e 33

651. Alopecia areata any of the following is TRUE:- o Can be mistakenly diagnosed in children with
o Cause localized scarring patch of the scalp inflammatory tinea capitis
 There is potential ability of the hair to regrow in all o Is best treated by oral isotretinoin
causes 662. The lunula is
o Characterized by question mark hairs o A hypovascular area
652. All of the following are implicated in the pathogenic o A parakeratotic area
of Rosacea except : o An area of increased nail plate thickness
 Increased sebum  The visible part of the matrix
o Dermal tissue changes o Caused by air spaces in the nail plate
o Helicopacter pylori 663. Which of the following structures contribute
o Vasculature hyperactivity substantially to the nail as it appears at its distal free
653. Alopecia Aerate in children the safe treatment margin
 Intralesional steroid o Matrix
o UVB o Nail bed
o Systemic steroid o Proximal nail fold
654. Children with traction alopecia best treatment  Hyponychium
o Intralesional steroid o All of the above
o Minoxidil 664. Black toe nail is produced by:
 Change hair style o Trichophyton rubrum
655. Conversion of testosterone to dihydrolestosterone o Tetracycline
occurs in – o Trauma
 Target organs  All above
656. Most common alopecia in men:- 665. Drug causes nail pigmentation:
 Androgenic alopecia o Bleamycin
o Telogon effluvium o Doxorubicin
o Anagen effluvium o Zidovudine
o Alopecia areata  All of the above
657. If andro-genetic alopecia is manifested in young 666. These drugs cause nail pigmentation:-
women, an endocrinological evaluation is especially o Chemotherapy (bleomycin)
indicated if there is also present significant:- o Antimalarial
o Acne o Antibiotic (minocycline)
o Hirsutism o Arsenic poisoning
o Virilization o Anti-viral (Zidovudine)
 All of the above o Fungal (T. Rubrum)
o None of the above  All of the above
658. Etiologic causes of hirsutism include all EXCEPT 667. Black longitunal lines in nail is due to:
o Polycystic ovarian syndrome o Trauma
o Anabolic steroids o Bacterial
o Congenital adrenal hyperplasia o Viral
o Acanthosis nigricans  Antimalarial drugs
 DLE 668. Patient suffer from black nail plate and skin around
659. Studies required in a suspected case of hirsutism nail is black, best treatment
include:-  Surgical removal
o Serum unbounded testosterone level o Curette
o Serum dehydroepiandrosterone sulfate (DHEAS) o Biopsy
o LH/FSH ratio 669. Most cases of fingernail hypertrophy originate with
o Prolactin o Fungal infection
 All of the above o Psoriasis
660. With virilizing ovarian tumors one observes  Trauma
o Hirsutism o Alopecia areata
o Masculinization o Lichen planus
o Elevated urinary 17-ketosteroids 670. Depressed area with postinflamatory
 All of the above hyperpigmentation after repeated injection what's better
o None of the above  Reassurance
661. Perifolliculitis capitis abscedens et suffodiens all o Topical steroid
except o Dapsone
 Most commonly affects white males o Topical retinoid
o Is associated with certain forms of arthritis 671. In disorders of the nail all except
o Can be painful o Koilonychias suggests B12 folate deficiency
| P a g e 34

o Onycholysis is associated with psoriasis  Vasodilation


o Leuconychia is a feature of severe liver disease o Obscured by the precedence of sympathetic vasomotor
 Splinter hemorrhage is a feature of severe liver disease control
o Beau lines disappear faster from fingernails than o Negligible at ordinary encountered temperatures
toenails o Dependent upon intact sympathetic innervations
672. White nails associated with leuconychia 683. The primary pathogenic event in milliaria is
o Anemia o Bacterial infection
o T.b o Ductal leakage
o Niphritis o Ductal hypercornification
o Liprosy  Portal occlusion
 All true o Increased sebum production
673. Patient with liver cirrhosis all nail have white 684. A man with skin coloured papules and papulovesicles
proximal part while distal part are normal in color on central face he noticed seasonal variation in which
o Onychodystrophy lesions increase in size during summer months on
o Twenty nail syndrome biopsy a clear fluid was seen diagnoosis is
o Leukonychia o Miliaria rubra
 Terry nail o Trichoepithelioma
674. Onychorrhexis is mostly present in:-  Hydrocystoma
 Lichen planus 685. Atrophy of the sweat glands is well known in:-
o Dariers disease o Systemic sclerosis
o Alopecia areata o Systemic lupus Erythematosis
o Psoriasis o Siogren's syndrome
675. Ptrygium nails are commonly associated with:-  All of the above
o Darier's disease o None of the above
o Psoriasis 686. The areas of hyperhidrosis in the following condition
 Lichen planus may be explained by sensory or sympathetic
o Onychomycosis dermatomal
676. The following are characterizing the nails in Dariers o Hyperhidrosis associated with autonomic dysre……
disease:- flexia
o The edge of the nail is showing V:-shaped scalloping o Hyperhidrosis sensory to orthostatic hypotenia
o Subungual hyperkeratosis o Hyperhidrosis due to post-traumatic syringemyelia
o Longitudinal red & white lines  All of above
o Keratotic papules o None of above
 All of the above 687. Concerning wheal all EXCEPT
677. Onycholysis occurs with – o Smooth raised flat skin lesion
 Tetracycline  Asymptomatic
678. Tetracycline affect nails in the form of:- o Disappears within 24 hours
 Onycholysis o Most commonly with urticaria
o Pitting 688. The most frequent cause of urticaria and angioedema
o Subangular hyperkeratosis is:-
o None of the above o Food additives
679. Periungual telangiectasia are diagnostic of o Foods
o SLE o Drugs
o Dermatomyositis  Infections
o Scleroderma o Emotional stress
o Glomous tumor 689. About urticaria:-
 None of the above o Can be non-immunological reaction
680. Painful nails can be due to all except o Drugs can caused histamine release without immune
o Ingrowing toenail reactions
o Glomous tumor o Infections agents can cause it
o Herpetic whitlow  All of the above are true
 Periungual fibroma 690. The most common involved mediator in urticaria is:-
o Subungual hematoma o Serotonin
681. Most common nail skin malignancy  Histamine
 Squamous cell carcinoma o Gasterin
o Basal cell carcinoma o Acetylcholine
o Malignant melanoma 691. Choose the wrong answer:-
682. The vasomotor effect of local heating of the skin is o SLE is more common in renal involvement than
o Vasoconstriction dermatomyositis
| P a g e 35

o Nail involvement is common in LP abdominal pain and joint pain. He had two brothers
o Pemphigus vulgaris is commonly involve mucous died from this case this patient must take HAE
membranes o Steroids again
 Urticaria is type III in immunological reaction o Bradykinine
692. About urticaria what is TRUE:- o Thalidomide
o The most common type in physical urticaria  C1 inhibitor concentrate
 Dermogarphism is the most common type of physical 703. Regarding hereditary angioedema all of the following
urticaria statements are true except:-
 Aspirin causes exacerbation of urticaria o C1 esterase inhibitor levels may be normal or elevated
o Acquired cold urticaria is the most type of physical o Non-pitting edema is known clinical manifestation
urticaria o Transmitted as autosomal dominant trait
o All of the above  The skin lesions are itchy and discolored
693. Patient have urticaria and not response to treatment o Can be associated with abdominal pain, vomiting and
what you should give him diarrhea
o Methotraxate 704. All are treatment of angioedema EXCEPT
 Cyclosporin . o Daazol
694. Patient have urticated wheal in back, which druge o Stanazol
avoid it o C1 esterase inhibitors
 Asprin  Corticosteroid + scadrenalin
o Corticosteroid 705. What is not used as prophylactic for hereditary
695. Which of the following is also known as stress- angioedema
induced urticaria: o Stanazol
o Pressure urticaria o Danazol (best treatment )
o Aquagenic urticaria  Fresh plasma(used in emergency)
o Cold urticaria o Traneaxamic acid
 Cholinergic urticaria 706. Lower face edema appear after the patient change
o Papular urticaria with anti HPT drug what may be that new drug
696. Urticaria due to stress is :  Captopril
 Cholinergic urticaria o Atenolol
o Adrenergic urticaria o Deltazem
o Heat urticaria o Propranolol
697. Stress uricaria 707. Urticaria pigmentosa is characterized by accumulation
 Cholinergic uricaria of mast cells
o Cold uricaria o At the DEJ
o Aquagenic uricaria  Around venules
o Pressure uricaria o Around arterioles
698. Dermaographism not occur with o Around epidermal appendages
o Hypothyroidism o Diffusely in the dermis
o Hyperthyroidism 708. Which of the following statements is wrong about
o Penicillin allergy vitiligo:-
 None of the above  It can be caused by contact with bleaching agents
699. White dermogaphism is seen in o Increase incidence of organ specific autoantibobies in
 Atopic dermatitis serum
o Psoriasis o Association of pigmentary disorders of ocular area
700. In hereditary angioedema there is defect in:- o Pigmentation is first seen perifollicular after starting
o Tyrosinase enzyme treatment
o Steroid sulfatase enzyme 709. Which is false about vitiligo:-
 CI esterase inhibitor enzyme o Rarely progress to involve whole body
o Cytochrome p 450 enzyme  Depigmentation of hair present in vitiligo patch
701. Patient presented to er with facial edema ,difficulty in indicates good response to treatment
breathing change of voice and abdominal pain his o Infant is not born with it but develops it later while
brother died from an episode of breathing difficulty. For growing up
above case 710. Vitiligo may be associated with all except:-
o Antihistamines will treat the patient o Addisons disease
 C1 esterase concentrate is the drug of choice o Thyrotoxicosis
702. Patient has criticaria and respiratory distress, takes o D.M
corticosteroid, withdraws in other special clinic, and  Tinea versicolor
now has respiratory distress, urticaria, myalgia, 711. Most common presentation of vitiligo
 Acrofacial
| P a g e 36

o Segmental o Hup2
o Focal  Kit gene
o Universal o PTEN
712. The following disorders are associated clinically with o Nemo/ikky
vitiligo all except o RET
o Addison disease 723. Tyrosinase enzyme deficiency results in:-
o Malignant melanoma o Vitilligo
o Pernicious anemia o Tinea versicolor
 Acromegaly  Albinism
o Diabetes mellitus o Hyperpigmentation
713. 8y child with upper eye lid hypopigmented macule 724. Child has white forelock, hair porest, developed wide
best treatment area of pigmentation on chest abdomen, and intestinal
o Corticosteroid hyperpigmented macule best treatment
 Tacrolimus o PUVA
o Retinoid  UVB
o Topical PUVA o Excimer laser
714. Resistant Vitiligo treatment o Topical steroid
o Spickle skin graft 725. Patient with hyper pigmented patches due to
 Cultured melanocyte competitive reaction with tyrosinas enzyme caused by
o ACTH o Vitiligo
o Surgical o Piebaldism
715. In vitiligo, which of the following sites is usually the  Pityriasis versicolor
first to respond to PUVA treatment 726. Hypomelanosis or depigmentation that result from
 Face complete inhibition of tyrosinase is:
o Umbilicus  Pityriasis versicolor
o Nipples o Vitiligo
o Perineum o Nevus of Ito
o Hands and feet o Piebaldism
716. The chief hazard of topical therapy in vitiligo is:- 727. Non-hodgkin lymphoma can be associated with:
 Phototoxic reaction o EBV-infection
o Chills & fever o HCV-infection
o Gastric disturbances o HIV-infection
o All of the above o H.pylori infection
717. Halo nevus associated with:  All of the above
 Vitiligo o Non of the above
o Melanoma 728. Which of the following therapies can lead to further
o Melanocytic navi hyperpigmentation in melasma some groups of patients
718. Percentage of association of halo nevus with vitilligo o Broad spectrum sunscreens
is:  Kligman formula
o 50% o Topical azelaic acid
o 2% o Chemical peels
o 10% o Steroid
 20% 729. Phenolic germicides cause cutaneous depigmentation
719. Halo nevus present with by
 Nevocellular nevus o Inhibition of tyrosine formation
o Blue nevus  Inhibition of tyrosine oxidation
o Neurofibroma o Inhibition of melanin oxidation
720. Halo nevus is most commonly seen around o Inhibition of cytosine action
o Blue nevus o Destruction of all melanocytes
 Spiz nevus 730. Neurofibromatosis , type 1 what is WRONG
721. Halo nevus o Start at puberty
o Usually appears soon after birth o Defect in neurofibromin
o Is inherited as an AD trait  Highly vascular
o Usually occurs as a single lesion 731. Malignant degeneration of Neurofibroma in von
 In histology, it is a compound nevus with lymphocytic Recklinghausen neurofibromatosis is
infiltrate admixed with intradermal nevus cells o Unreported
o Need surgical excision in young patients  2%-5%
722. Piebaldism results from mutations in which of the o About 25%
following genes? o About 50%
| P a g e 37

o More than 80%  Adenoma sebaceum (Sebaceous adenoma)


732. A child presented by café au lait patches, all should be o Adenomatous carcinoma
done except:- o Sebaceous carcinoma
o Complete physical examination o Sebaceous epithelioma
o Skull x:-ray 742. Histopathology of porokeratosis shows
o Fundus examination  Coronoid lamille
 Examination of the fundus of the mother & the father 743. Keratin in coronoid lamellae is
733. The most common manifestation of tuberous sclerosis  3-12 k
o Cardiac arrhythmias 50% 744. Netherton syndrome defect
o Mental retardation 60-70%  Serine protease
 Seizures 80-90% o Conexins
o Enamel pitting  Spink 5 gone
734. Basal cell nevus syndrome includes all of the 745. Investigation of Addison syndrome
following EXCEPT  Cortisol level
o Multiple BCC in at least 50% of patients o ACTH level
o Multiple jaw cysts 746. Ichthyosis linears circumflexa, trichorrhexis
o Palmoplantar pits invaginata and atopy are characteristic of
o CNS anomalies o Refsum syndrome
 Follicular atrophoderma on extremities o Dorfma-chanarin syndrome
735. Marfan’s syndrome is characterized by striking lack  Netherton syndrome
of which of the following components of the connective o Rowel syndrome
tissue ? 747. Kawasaki syndrome is manifested by:-
o Collagen I o Cervical lymphadenopathy
o Elastin I o Skin desquamation
 Fibrillin I o Strawberry tongue
o Collagenase I  All of the above
736. All of the following is present Sjogren Larsson 748. Child suffered from fever then peripheral peeling
syndrome EXCEPT:- strawberry tongue best ttt (kawasaki)
o Congenital icthyosis o Erthromycin
o Mental retardation o Emollient
o Spastic diplegia  IV Ig
 Anterior Congenital cataract o Assurance
737. Patient have multiple pigmented macules in lip, 749. About Bloom's syndrome what is WRONG:-
palate, tongue with no other anomalies the diagnosis o Telangectatic Erythema on butterfly area
 Peutz-jeghers syndrome o Dwarfism
o Melasma o Photosensitivity
o Hemochromatosis  Autosomal dominant
o Addison disease o None of the above
738. Difference between number of hair follicles in 750. Which of the following disorders of pigmentation is
newborn& adultlife thought to result from mosaicism?
o Number increase in adultlife o Piebaldism
o Number decrease in adultlife o Vitiligo
 No difference in number in newborn , adultlife  Hypomelanosis of Ito
739. Features of Preus syndrome include all of the o Waardenburg syndrome
following EXCEPT o Woof syndrome
o Generalized hypopigmentation 751. Cutaneous manifestations of Darier's disease may be
o Hypochromic anemia exacerbated by:-
o Growth retardation o Dapsone
 Lichenoid skin eruption o Beta blockers
o High-arched palate  Lithium
740. The characteristic histologic feature of epidermolytic o Thiazides
hyperkeratosis is : o Antimalarial
 Hyperkeratosis 752. Axillary slight erythematous papules since 7 years
o Acantholysis H/P hyperkeratotic and dilated hair follicle best
o A subepidermal bulla treatment(Fox–Fordyce disease)
o A spongiform pustule  Retinoid
o A reticular pattern of vacuolization of the epidermis o Antibiotic
741. Muirr –tori syndrome NOT develop with: o Tacrolimus
o Keratoacanthoma o Emollient
| P a g e 38

753. Dirty warty papules on the neck of male offensive  Sudden blindness
edema +subrabasal cleft vesicles best treatment 762. Which of the following diseases may cause elastic
 Isotretinoin fibers pathology in dermis
o Emollient o Darier disease
o Cytotoxic  Pseudoxanthoma elasticum
o Antifungal 763. Pregnant female no straie in
754. Skin colored or slightly pigmented follicular papule in o Psudo xanthoma elasticum
both axillae. H/P keratin plug in follicular infudibulum o Cutis laxa
best treatment o Marfan Syndrome
o Dapson  Ehlers danlos
o Antihistamin 764. Inheritant of Ectodermal dysplasia
 Topical retinoid o AD
o Topical Vit D3 o AR
755. A 30-years-old patient presents with greasy, keratotic, o XL-D
yellowish-brown papules and plaques over the  XL-R
retroauricular area, scalp, forehead, front of chest, and 765. Inheritant of Tuberous Sclerosis
midline on back. Large vegetating foul-smelling crusted o AR
plaques are present on axilla . Small punctuate keratotic  AD
pits are present on the palms and soles . Biopsy from o Sporadic
plaque reveals suprabasal cleft , acantholytic cells, and o XL
focal dyskeratosis . What is the treatment of choice ? 766. The differential diagnosis of acrokeratosis
(Darier) verruciforms include all EXCEPT
o Systemic corticosteroids o Dariers disease
o Cyclophosphamide o Plane warts
 Acitretin o Seborrehic keratosis
o Thalidomide o Epidermodysplasia verruciforms
756. Acanthosis nigricans is commonly associated with:-  Subcorneal pustular dermatosis
o Internal malignancy 767. The most common presentation of lipoid protenosis
o Diabetes is:-
o Obesity o High blood lipids
o Insulin resistant diabetes o Convulsions
 All of above  Hoarseness of voice
757. Commonly acanthosis nigrigans is associated with: o Albuminurea
o Diabetes insipidus 768. Treatment of nevus sebaceous is:-
 Insulin resistance o Electrocautery
o Adenocarcinoma  Total excision before puberty
758. Female patient have Pseudoxanthoma Elasticum and o Total excision after puberty
she is pregnant what do u suspect she will not suffer 769. Patient with unilateral hyperhidrosis in the chest
from: ,face& neck;should alert dermatologist about the
o Abortion possible chest problrms; harlequin syndrome
o Cardio vascular system problems o True
o Early striae  False
 Sudden blindness 770. Steroid sulfatase enzyme deficiency is present in:-
759. Pregnant female suffers from psudoxanthoma o Icthyosis vulgaris
elasticum what is true o Epidymolytic hyperkeratosis
 Highly liability to stria occurrence o Lamellar icthyosis
o Abortion  X-linked recessive icthyosis
760. Pseudoxanthoma elasticum-like illness is reported to 771. New born girl vesiculobullous linear tense bullae on
be associated with all of the following EXCEPT : arm incontinentia pigmentia
o Penicillamine intake o XLR (X-linked recessive )
 Vitamin A administration  XLD (X-linked dominant )
o Use of nitrate fertilizers o AD (autosomal dominant)
o Multiparity o AR(autosomal recessive)
761. Female patient have pseudoxanthoma elasticum. 772. Hypomelanosis of Ito
And she is pregnant what do u suspect she will not o Is a variant of incontinentia pigmenti
suffer from: o Is inherited as XLD trait
o Abortion o Occurs much more frequently in males than females
o Cardio vascular system problems
o Early striae
| P a g e 39

 Can be associated in 3/4 of patients with skeletal o Melanoma


abnormalities, mental retardation and ocular 782. Asymptomatic skin colored multiple papules on the
abnormalities back and proximal arm of an 8 years child and history
o Is best treated by oral retinoids of cough and mild fever. What diagnosis
773. The following statements about incontinentia pigment o Lichen nitidus
(IP)is /are correct all EXCEPT o Keratosis pilaris
 It affects males or than females  Papular Sarcoidosis
o Cutaneous lesions occur in 100% of patients and appear o Vit A
at birth or shortly after it 783. Patient Comes to the ER &his chest x ray reveals
o Leucocytosis and peripheral eosinophilia are not bilateral hilar ln and he gives u previous history of
uncommon red indolent lesion on the face, ear, nose and lip
o Ocular anomalies are among the most severe systemic what ur diagnosis:
anomalies and can lead to blindness o SLE
o Anodontia or late dentition are not uncommon  Sarcoidosis
774. Histopathological differential diagnosis of granuloma 784. Triad of hilar LN,arthritis,skin lesion:
annulare is:-  Lefogrin syndrome
o Sarcoidosis o Netherton
 Necrobiosis lipodica o Herodite
o Behcet's disease 785. Erythema nodosum ,bilateral hailar lymph node
o Pyoderma gangrenosum adenopathy
775. 21 year old nurse has plaques on both of her hands. o Sjogern
The histopathology of the lesions showed palisaded  Lefogrin syndrome
granuloma with histiocytes and clear mucin in the o Heartoed syn
centre that is stained with alcian blue.diagnosis is 786. The most common cutaneous manifestations in
 Granuloma annulare amyloidosis:-
776. Necrobiotic palisading granuloma with mucin o Bullous amyloidosis
o Necrobiosis lipodica o Nail dystrophy
 Granuloma annulare o Alopecia
o Rheumatoid nodules  Purpuric lesions & ecchymosis
o Sarcoidosis 787. Amyloidosis be secondary to chronic diseases such as
777. How many percentage of diabetics with necrobiosis all of the following EXCEPT :
lipoidica diabeticorum o Rheumatoid arthritis
 0-3% o Sjogren’s syndrome
o 1% o Inflammatory bowel disease
o 5-10% o Hodgkin’s disease
o 50-70  Generalized psoriasis
778. Male to female ratio in Necrobiosis lipodica:- 788. The most common skin sign of systemic amyloidosis
o 10: 1 is/are:
o 1: 10 o Bullous lesions
o 1: 1 o Alopecia
 1: 3  Petechiae, purpura, and ecchymoses+
o 3: 1 o Nail dystrophy
779. Diabetic dermopathy characteristically occurs on the:- o Cutis laxa
o Scalp 789. Amyloidosis be secondary to chronic diseases such as
o Neck all of the following EXCEPT :
o Axilla o Rheumatoid arthritis
 Shin o Sjogren’s syndrome
o Sole o Inflammatory bowel disease
780. Sarcoidosis in black patients' mostely o Hodgkin’s disease
o SC nodules  Generalized psoriasis
o Erythema nodosum 790. Photosensitivity of the skin occurs in all types of
o Scar porphyria EXCEPT
 Maculopapular o PCT
781. Serum angiotensin 1 converting enzyme ACE is o HCP
typically elevated is:- o VP
o Atopic dermatitis  AIP
o Chronic urticaria o CEP
o Pemphigus vulgaris 791. The blisters of the porphyria is formed
 Sarcoidosis o Subcorneal
| P a g e 41

o Intraepidermal o Erythema nodosum


 Subepidermal  All of the above
o In different levels depending on the type of porphyria 799. The most biologically active form of vitamin E is
o Because of cutaneous hemorrhage  Alpha tocopherol
792. Factors known to be provocative of variegate o Alpha tocopherol acetate
porphyria include EXCEPT o Alpha tocopherol sulfate
o Anticonvulsants o Beta tocopherol
o Alcohol o Beta tocopherol acetate
o Pregnancy 800. Male has follow scales in front of thigh with no
 Vitamin C deficiency evidence of vitamin ↓↓ best treatment
o Febrile illness  Retinoid
793. Nodular lesions on elbows & knees with small yellow o Vit A
papules on thigh & buttocks the diagnosis is:- o Vit B
o Eruptive xanthoma o Vit C
o Tuberous xanthoma 801. Acrodermatitis entropathica
 Tuberoeruptive xanthoma  Zinc
o Nodular xanthoma o Ca
794.A 10-year-old patient presents with multiple tuberous o Vit C
and tendinous xanthomas over the albows , Achilles o Fe
tendon, knees, and corneal arcus. An older sibling has 802. Drug cause pellagra like symptoms all EXCEPT
similar lesions . What is the MOST probable defect ? o 5 mercaptopurine
o Mutation in lipoprotein lipase enzyme o INH
o Deficiency of apolipoprotein C2  Captopril
o Defect in apolipoprotein A 803. All of the following are drugs which may induce a
 Defect in LDL receptor pellagra like eruption EXCEPT
794. Patient with nodular lesions on the elbows and knees o Sulfonamides
with small eruptive yellow papules on the thighs and o 6-mercaptopurine
buttocks. Which is the disease?  ACE inhibitors (Antihypertensive)
o Eruptive xanthema o Isoniazid
o Nodular xanthema o Anticonvulsants
o Tuberous xanthoma 804. Pellagra caused by deficiency in
o Tendinous xanthema  Nicotinic acid Niacin (vitamin b3)
 Tubereruption xanthoma o Folic acid
795. Child presenting with tendinous or tuberous o Riboflavine B12
xanthomas and has a brother with the same o Thaiamine B1
manifestation, the defect is probably in 805. Pellagra is due to
o Lipoprotein lipase o Zinc deficiency
 Ldl receptors o Vit B deficiency
o Apoprotein b 100  Niacin deficiency
o Apoprotein c 806. The mucin of myxedema is predominantly
796. 61 year old man with yellowish ulcerating lesion on  Hyaluronic acid
eyes. Histopathology of lesion shows epitheloid cells o Chrondroitin sulfate A
with multinucleated giant cells and touton cells o Chrondroitin sulfate B
(necrobiotic xanthogranuloma) o Heparin
o Mostly associated with Ig M monoclonal gammopathy o None of the above
o High levels of cholesterol and vldl 807. In relation to fixed drugs eruption which is TRUE:-
o High level of TG o Is cell mediated immunity
o Diabetis insipidus common complication  Is type in hyper sensitivity
 C1 esterase inhibitor decreased o Toxic effect of drug
 Mostly associated with igg monoclonal gammopathy 808. One of the following drugs may cause eruptive
797. Female with yellowish plaques and papules on xanthoma:-
bilateral medial canthus, biopsy revealed touton cell  Retinoids
(xanthelasma) what is TRUE o Ampicillin
o VLDL & TGS o Dapsone
o LDL 809. Most common clinical manifestation of drug eruption
 Hyperlipoprotenemia is:-
798. Differential diagnosis of gouty tophi include:-  Morbilliform eruption
o Tuberous xanthoma o Lichenoid drug eruption
o Sarcoidosis o Photo allergy
| P a g e 41

o Fixed drug eruption  Prolonged PTT& PT.


o T.E.N 821. Preferred laser used in treatment of port wine nevus in
810. The most common form of drug eruption is:- children is:-
o Bullous o Nd-Yag laser
o Fixed 565 pulsed dye laser
 Exanthematous  585 pulsed dye laser
811. Onset of exanthematous drug eruption after taking o Argon laser
therapy is 822. The wavelength of the Excimer laser that can be used
 2 weeks in the treatment of vitilligo is
o More than 3 weeks o 755 nm
o 1 month o 311 nm
o 3 months o 585 nm
812. The only drug that cause anetoderma is:  308 nm
o Delatin o 1064 nm
 Pencillamine 823. The "gold standard" laser for laser-assisted hair
o Procainamide removal in people with skin types I-III is
813. To diagnose drug reaction:  Alexandrite laser
 Skin test. o Ruby laser
o Brick test. o Diode laser
o Rast test. o Nd: Yag laser
o Increase blood oesenophil. o Co2 laser
o None of the above. 824. The Q-switched ruby laser delivers:-
814. Ochronosis caused by all the following drugs except o UV light
o Topical formulation containing hydroquinone o Green light
o Phenol o Yellow light
 Beta blockers (antihypertensive drugs) o Infrared light
o Amiodarone  Red light
815. Which of the following medications would not 825. Color of q switched ruby:
induce ochronosis ? o Infrared
o Cream containing hydroquinone  Red
o Antimalarials o Green
 Antihypertensives 826. Patient with keloid residual after treatment with
o Products containing resorcinol and phenol intralesional steroid best laser
816. Contact dermatitis on one eyelid commonly due to o CO2
o Mascara  585 dye laser
o Eyeliner o IPL
o Eyeshadow(if both eyelids) o Fraxel
o Solution of contact lenses 827. Magnolian spot best ttt
 Nail polish  Q switched ruby
817. Dose of nifedipine in treatment of Raynaud o PDL
phenomena o Co2 laser
o 10 mg twice daily o IPL
o 20-30 daily 828. Nevus of OTA best treatment
 30-40 daily  Q-switched Nd:YAG(532)
o 10-80 daily o Q- switched ruby
818. Dose of nifedipine in scleroderma o IPL
o 10-20 mg o Long pulse
 10-80 mg 829. What is true about PUPPP:-
o 20-40 mg o Occurs in the 1st trimester
819. Warfarin toxicity (painful lesion on LL darkness o Neutrophils are found around blood vessels
bluish) treatment  Occurs in the 3rd trimester
o Vit k1 IV in emergency 830. PUPP (pruritic urticarial papules and plaques of
o Heparin pregnancy)the best treatment
o Monoclonal anti body - purified protein C concentrate  Topical corticosteroids &anti histamine
o Debridement grafting o Mtrx
 All true o Colshecin
820. In patient treated with warfarin and develop o Oralcs
echomosis in lower limbs, the most probable cause 831. Patient with rubbery swelling in the lower lip and
is : side of the tongue which sometimes decrease or
| P a g e 42

increase and histopathology shows some foci of o 10%


granuloma what is best diagnosis: o 0%
 Melkerson rosenthal syndrome 841. Commonest cause of graft versus host disease
832. Male with enlarged lips and fissured tongue all are  Bone marrow transplant
treatment except o Blood transfusion
o Corticosteroid o Kidney transplant
o Dapsone 842. 6 y child erythema and scaling with p.p
 Retinoid hyperkeratosis and interphalangeal hyperkeratotic
o Thalidomide papules the best treatment
833. Which of the following is the best treatment option  Narrow band UVB
for frostbite? o Retinoids
 Removal from the environment , immersion in water at o MTX
40-42 C and pain control + 843. Treatment of recurrent Erythema multiform
o Removal from the environment , immersion in water at o Thalidomid
30-35 C o Dapson
o Immersion in water at 45-50 and pain control  Anti-HSV
o Removal from the environment and pain control 844. Erythema multiform mostly commonly associated
834. Frostbite treatment implies with:-
o Removal from site, immersion in water 45 _ 50  HSV
, pain control o HCV
o Removal from site, immersion in water 30 _ 35 o EBV
, pain control o HPV
 Removal from site, immersion in water 42 _ 45 845. Recognized causes of erythema multiform include all
, pain control EXCEPT
o Removal from site, pain control o Herpes simplex infection
835. Ulcer on leg stump due to:- o Mycoplasma pneumonia
o Diabetic o Sulphonamides therapy
o Vascular pressure o SLE
o Friction of arm prosthesis  Chronic liver disease
 All of the above 846. Transepidermal elimination which investigate not
836. A child with depressed skin lesion on buttocks due needed
to past injections of unknown drug best management o Blood glucose
o Topical steroids o Creatinin
o Topical calcineurin inhibitors  Cortisone
 No treatment o HIV
837. Varicose veins maybe complicated by:- 847. Lesion of Kyrle's disease:-
o Hyperpigmentation  Follicular
o Ulceration o Interfollicular
o Painful o Acrosyringium
o Thrombophlebitis 848. The most common misdiagnosis of follicular
 All of above mucinosis is:-
838. 47 years old patient presented with an ulcer on the o Sarcoidosis
big toe, foot & lower leg are cold & arterial pulse is not o DLE
felt, the diagnosis is:- o Leprosy
o Diabetic ulcer o Discoid eczema
o Stasis ulcer  MF
 Arterial ulcer 849. Alopecia mucinosis
o Venous ulcer.  Presents a variable clinical appearance
o Traumatic ulcer o Usually progresses to lymphoma
839. Treatment of choice in Pyoderma faciale is:- o Exhibits metachromasia with Alcian blue stain
o Tetracycline o Is more likely to be serious in children than in adults
o Prednisolone o Is confined to the scalp
o Isotretenoin 850. Which of the following statements about bromoderma
 Combination of prednisolone & Isotretenoin is untrue
o Doxycycline o It is common manifestation of chronic bromide
840. Oral lesion in chronic GVHD:- poisoning
o 100% o It is unrelated to blood bromide levels
 80%  It may appear only after discontinuation of the
o 50% offending drug
| P a g e 43

o It may present multiple types of lesions o Gunclon


o All of the above o Aminglicoside
851. Treatment of cholastasis  Sulphanomide
o Antihistamin 861. Calciphylaxis the setting is
o Cryotherapy o Lung team dialism
o Topical steroid  Hyperparathyroidism
 Opioid antagonists o Intoxication
852. Jordans’ anomaly (lipid droplets in leuckocyte ) are o Endothelial vascular lesion
found in 862. Glucagoma disease of
o Eccrine glands  Alpha cell of pancreas
 White blood cells o Betacell ofpancreas
o Red blood cells 863. Erytherma polycyclic large maculae, a central
853. Jordan anomaliy clearing, the edge increase many mm daily what is
o Lipid droplets in eccrine colis the treatment
 Lipid droplets in granulocytes o Steriod
o Elevation of muscle enzymes o Antibiotic
o Elevation of hepatic enzymes  Search for associated malignancy
854. The mainstay therapy for necrolytic migratory 864. 54 year old patient present with lesion on skin
erythema is: investigation showed TNF high , which diagnosis
o Chemotherapy +  Malignancy
o Pulse steroids o Flummenent pust.psoriasis
 Excision of the pancreatic tumor o Genral erthromderma
o Nutritional supplements 865. Patient with soft papule , central opening , localized
o Electron beam on the face , what is the treatment(commedons)
855. Multiple annular multicnetric lesion on the back, the o Surgical excision
lesion was migrating by 12cm/day best treatment o Tetracycline
(NME) o Intralisonal steroids
 Removal of underlying pancreatic cancer  Reassurance
o MTX 866. Ttt of nodular prurigo
o PUVA  Puva
o Topical corticosteroid o ….......photo
856. Child with asymptomatic 2mm vesicular lesions o …..........photo
contain milky color liquid on posterolateral tongue. o Antihistamine
The lesions are 2 x 5 mm. Best treatment is 867. Which of the following can use UV:
 Counseling o SCC
o Removal of part of the tongue o Nodular BCC
o Intralesional steroids o Superficial BCC
 Kerato acanthoma
857. The early feature of lipoid proteinosis: 868. Patient with knuckle pads on his finger
 Hoarseness of voice. o Intralesional injections of corticosteroids
858. An old man has rolled on ulcer near the eye, the o Salicylic acid or urea
histopathology showed basiloid cells.best treatment o Trauma main cause
is o Familial cases observed
o Wide excision  All above
o Electrocurettage 869. About smoking all following true exept
o Cryotherapy  Smokers are sun protected
 Moh's surgery o Smoking cause premature aging wrinkles
859. 6 hours old baby suddenly develop palpable purpura o Mutch of wrinkeles from infra red smoke effect
surrounded by erythema at elbow,knee and buttocks o Smoking decrease collagen deposition
and his urine analysis is normal but the lesions are 870. What is of fallowing least probable to cause
tender what is his diagnosis: prematurity of birth in the affected pregnant ladies:
 Acute heamorrhagic pupura of childhood o Anetadema
o Anaphlactoid p o Cutislaxa
o C)idiopathic thrombocytopenic purpura o Psoriasis 1st week of life
It develop without renal changes and tends to affect  Mother scc 20 years ago
extremities and it is painful 871. Antibiotic ci of patch test +ve
860. Patient allergic PAPA ( para amonic benzoic asid )  Hydroxyzine hydrocloride
wich treatment you don’t give 872. Med ‫موضع‬
o Pencillin  ‫أوال الظهر اذا ما وجد حتما البطن ثانيا‬
| P a g e 44

873. Erythematous lesion mild scaling more than 7mm /d o Hypocomplementia


 Advice to search for malignancy 885. Histopathology of parakeratosis shows
o Borrelia bordgraferi  There is nuclear in keratin cell (coronoid layer)
o Top corticosteroids only 886. What not assocaited vasculitis
o Anti histone o Erythema elevatum duntenum
874. Kasabach Merritt syndrome associated with: o Erythema induratum
 Angiosarcoma o Granuloma faciale
 Kaposiform hemangioendothelioma+tufted angioma ‫اكتر‬  Reticular histiocytosis
‫االورام الوعائية المرافقة‬+platet trapping+consumption 887. Heliotrobe erythema accurse with
coagulopathy o LE
875. Common site of affection histiocytosis in children  Dermatomyositis
o Skin 888. Not a vasculitis:
 Skelton o Granuloma faciale
876. Child develop erythematous rash after sun exposure, o Erythema multiform
CBC reveal increased rbcs and porphrins what is  Reticular histiocytosis
your ttt: 889. Defention of perforatin disease (Trans epithelial
 Phlebotomy elimination best described as):
o Strict sun protection  Extrusion of dermal materials through the epidermis
o Steroid with minimal damage
877. Elimination of terbenafine is increased by  Several different perforating diseases have been
o Excreted mainly in sebum not in sweet associated with chornic renal failure including
o Metabolized in liver kyrle,sdiseases
o Excreted into urine 80% &20% into faeces  These are a group of disease in which altered
o Cimitidine increased terbenafine components of skin are eliminated via epidermis
 Phenobarbital oral decreases levels of terbinafine oral 890. Extrusion of dermal content through the epidermis is
by speeding up drug metabolism. called
878. Leatest complication of varicella  Transepidermal elimination
o Encephalitis 891. Erythrema similar to wood groins
o Peneomonia  Erythema gyratum repens
 Cardiac (Pericarditis) 892. Match the following:
o Liver  Syphilis →penicillin g
879. The most itchy of lichen planus is:  Lvg →doxycline
o Atrophic.  Rectal gonorrhea →cefitrioxon
 Hypertrophic  Chancroid →ciprofloxacin
o Annular  Chlamydia urethritis →amoxycillin
880. To assure high sensitivity in diagnosis of sle by 893. Geographic tongue
immunofluorescence you shold take skin biobsy from  Psoriasis
o Sun exposed involved sites 894. Graft host
o Sun hidden involved sites
 Immunocompetent donor cells against tissue of
 Uninvolved sites sun exposed(diagnosis) immucomopetent host
o Uninvolved sites sun unexposed(prognosis) 895. What is not located in the erythema nodosum
881. Which of the following presents with pruritus as a
 Soreness
common feature
o Joint pain
o Trousseau's sign
o Conjunctivitis
 Leser trelat sign
896. SCC discreption:
o Hypertrichosis
 Pearl horns in histology early
882. Clssic kaposi, sarcoma all are true EXCEPT:
o Grade 3 the most undiffentiated according to tumor
o A prepondence of people of jewish ashkenaziandor
Grading
mediterranean descent
o Adnoid type is the worst
 Also documented in northern european countries
897. Clinical and histological picture of squamous cell
o A specific hla marker is not known carcinoma in face what is true regarding this type of
o Male to female ratio of 15-1
tumor?
o Median age of onset 64-66years o Adenoid type is the worst type
883. Which of the following diseases may cause elastic
 Horn pearl is present in well differniated tumor
fibers pathology in eps(elastosis perforans serpiginosa)
o Lesion in ear shows more metastases
o Darier disease
898. Scleroderma can be associated :
 Pseudoxanthoma elasticum
 GIT problems
884. Urticarial vasculitis can develop with low:
o Lung
 Ig E 899. Signs of malignant transformation of melanoma
| P a g e 45

 Asymmetrical shape  C1 esterase inhibitor supplement treat the condition


 Irregular borders 912. Most common complication of azathioprine
 More than one color o Hair fall
 Uneven distribution of color  Leukopenia
 Greater than 6 millimeters in diameter o Pancytopenia
 Evolution 913. Turn over time definition
900. Langerhans cells  Epidermal renewal time in normal is 14(cell
o Play a role in contact dermatitis cycle)+14(epidermal transit
o Reduces of the number in psoriasis time)+14(desquamation)=39-40 day while in ps.=5days
o Disrupted function by ultraviolet only
 All of the above 914. What is the envelope protein of the cornified cells
901. *Kyrle infects  Loricrin
o Grape fruit 915. Langerhan's cell histioplasmosis is usually in:
o Perifollicle o Skin
o Terminal tube o Heartmuscle
o All of the above  Bone
 Basilar layer of the epidermis 916. A 42 years truck driver presented with
902. Antibodies not present in pemphigus vulgaris papulovesicular lesion on the penis since 2 weeks and
o Anti desmoglin after one week suffer from firm, hard, painfull lymph
o Anti desmoplakin node in right side then notice sinuses and discharge. He
 Anti envoplakin received treatment from dermatologist and after 2
903. Autoantibodies in pemphigus vulgaris are directed weeks the case become worsen with generalized
against: lymphadenitis (LGV). What is the best treatment :
o Basement membrane o Penicillin benzathin
 Intercellular substance o Oflaxacin
o Cell nucleus  Tetracycline
o Keratin o Acyclovir
o Cell membrane 917. Most fatal of those is:
904. Lesion basic cardiovascular syphilisis o Bullos pemphgoid
 Aortic o Pemphigus foliaceus
905. Calcinosis, raynaud phenomenon, esophageal  Pemphigus vegetans
dysmotility, sclerodactyly, and telangiectasia ? o Herpes gestations
 Crest syndrome. o Dermatitis herpitiforms
906. Dysplastic nevus syndrome all of following are true 918. Lichenoid drug eruption-
except:  Tetracycline
 Autosomal dominant o Penicillin
907. Patient take steroid for treatment of dermatomyocitis o Erythromycin
he developed muscle pain take 40mg cortisone+15mg o Ciprofloxacillin
mitotrixat since week the myopathy become worst and 919. What is true about protein C?
enzyme of muscle is normal the probable cause  Has antithrombotic effect
 Steroid induced myopathy o Inhibit protein S
o Aging 920. Heterogenous protein c deficiency result in:
o Mitotrixat o Thrombocytopenia
o Worst myopathy o Low ability to coagulate
908. Ossification occurs mostly with  coumarin skin lesion
o Chondoroidsyringoma o act as 10 % as normal
o Syringosystadenoma  Venous thrombosis
 Epithlioma of malhebre(pliomatrixoma) 921. Protein c deficiency
o Trichoepithelioma  Venous thrombosis
909. Hepatic pruritis best treatment: 922. Alternative to corticosteroid when tapering it in
o Topical steroids urticarial patient :
o Antihistaminics  Cycloporin
 Opiode antagonist o Colchicine
910. Axillary papules,itchy since one year (foxfordyce) the o Cyclophosphamide
question is: what is the best treatment: o Methotrexate
 Retinoides 923. Honey crusted lesion in 15y pt :
911. Father admitted to the ICU with shocking three of his  Imptigo
sibling have the similar condition (hereditary
angioaedema): what is true about this case
| P a g e 46

924. 20 years old female treated for depression, suffering  No treatment


from ecchymosis and purpura of the lower leg which o Q switched nd-yag
investigation(Gardener diamond syndrom)? o Alexandrite
 Intradermal injection of autolgous RBC,s 934. SPF 15 definition
o Platelet count  Needs 15 fold more UVB to cause erythema
o Skin biopsy o Needs 15 fold more UVA to cause erythema
o Bleeding and clotting time o Needs 15 fold more UVB and UVA to cause erythema
925. 1 month urethritis after extramarietal sex then vesicles o Needs 15 min of UVB to get erythema
and pustules on side of foot and sole: 935. Old pt ithy myxedematous papules on trunck and
o Palmoplantar pustulosis extremities and alopecia and oedematous tongue will
 Reiter syndrome have:
o Syphilis  Low level of T3 and T4
926. Adjuvant to corricostiroid in PV : 936. Cause of bleeding point when removing scale in
 Cyclophosphamide psoriasis:
o Methotrexate  Thining of epidermis over dermal papillae
o Cyclosporine o Munro microabsess
o Colchicines o test tube shape over rete ridge
o Mycophenolate mofetil or azathioprine o Absence of granularlayer
927. Angioedema in siblings: 937. Patient with certain lesions on investigations revealed
o Give low dose sedative antihistaminic hyphae what is the best treatment?
o All will have high oesinophilia  Septrin and streptomyces
 Give anti c1 estrase in attack o Imidazole derivatives
o All have low bradykinin o Corticosteroids
928. Generalized psoriasis in alcoholic and deactive 938. Pregnant patient will not have striae at all in:
hepatitis B with lab upper normal SGPT and SGOT and o Elastosis perforans serpiginosa
high creatinine TTT :  Ehlers danlos
o Acitritin o Marfan syndrome
o Methotrexate o Cutislaxa
 Prednisolone 939. Cyto skeletal structures
o Cyclosporine  Tonofilament
o Cyclophosphamide o Desmosomes
929. Patient with plaque psoriasis since 10 years and o Odland bodies
developed diffuse erythema & exfolliation with history o Hyalokeratine granules
of alcohol intake since 10 years & +ve HBV and 940. Best ttt for Juvenil Rieter
laboratory tests normal values of liver enzymes  The arthritis was treated with acetaminophen and
&elevated creatinine level, what is the treatment naproxen
 Prednislone 941. The most incriminated drug in EM:
o Cyclosporine o Sulphonamide
o MTX  Penicillin
o Acitretin o Rifampicin
930. The only definite indication for giving systemic 942. What is accompanying viral disease porphyria
corticosteroids in pustularpsoriasis is:  Hepatitis C
o Psoriatic enythroderma with pregnancy o Hepatitis B
 Psoriasis in a patient with alcoholic cirrhosis 943. Acute febrile neutrophilic dermatosis associated with
o Moderate arthritis o Multiple myloma
o Extensive lesions  M.leukemia
931. Which of the following is most commonly seen in o Hepatitis
patients with acanthosis nigricans? 944. Gingival hypertrophy occurs most commonly with
o An underlying internal cancer.  Burkett's lymphoma
o An underlying non-hodgkin 945. The following are clinical features of sweet syndrome
o Internal malignancy. EXCEPT:
o DM. o Erythematous plaques and nodules occainolly with
 Insulin resistance. vesicles and pustules
932. All for TTT of acne EXCEPT: o Predominantly neutrphilic infiltration in the dermis
 Pimozide  Leukocytoclastic vuculitis
o Tetracycline o Preceded by non specific respiratory or git infection
o Tretonine o Associated with haemoproliferative disorder
o Vit A o Lecocytosis>8000
933. Treatment of 3y pt with mongolion spot on his back :
| P a g e 47

946. An elderly man + intense itching urticarial plaque o Steriod


lesions + tense bubbles on the abdomen and nooral o Antibiotic
lesions what is the  Search for associated malignancy
o Desmoglin 1 958. Most common from long term treatment with
o Desmoglin 3 retinoids:
 BPA 180 o Renal
947. Old man had pigmented scaly lesion on the face what o Cirrhosis
is the best topical treatment  Skeletal porosis
 Imiquimod 959. 28 male patient with bluish color in thumb finger
948. The best treatment for dermatitis herpetiformis lesion, with dimpling sign (fibroma), what treatment:
o Steroids  Excision
 Dapsone o Leave and follow up
o Astrin 960. Best area to cure from vitiligo by PUVA:
949. A case of dermatitis herpitiformes what symptom o Hand and foot
improves with gluten free diet and not with dapson  Face
 Intestinal velli atrophy o Perineium
950. Hepatic pruritis best treatment: o Nipple
o Topical steroids 961. Female patient with acne treated with doxycycline
o Antihistaminics 100 for 2 months then develop papulopustular and
 Opiode antagonist nodular lesion (kliebsiella) what is diagnosis:
951. Child 4 months with bruises over face, arms, limbs o Drug eruption from doxy
what is the diagnosis: o Acne conglobate
 Wiskot Aldrich syndrome  Acne gram -ve follicultis
o Down syndrome 962. Reaction type of A.C.D
o Phenylketonuria o 1
o Peutz jagger o 2
952. What true about fluconazole : diflucan o 3
o Decrease carbamazipine  4
o Asother antifungal fromazoles 963. As regard to scabis the burrow contain
 Triazole difluorobenzyl 1,2,4 triazole  10-12 mites
o Not related tor enal o 20-30 mites
o Half life 30 to 37 hours o 30-40 mites
953. Which of the following is not a cutaneous marker of o About100 mites
internal malignancy? 964. *The best TTT of scabies
o Bullous pemphigoid o Application of topical benzyl benzoate 25% daily for 7
o Acanthosis nigricans days
o Dermatomyositis  Scrabing and signle topical use of benzyl benzoate 25%
 Erythema chronicum migrans 965. Hidradenitis suppurativa all true except
954. Multiple annular multicnetric lesion on the back, the  Should be ttt with intralisional antibiotics for 2 w
lesion was migrating by 12cm/day best treatment o It is a chornic inflamatoin of apocrine sweet gland
(NME) 966. * Regarding fox foradyce what is true
 Removal of underlying pancreatic cancer o Female=male
o MTX o Asymptomatic
o PUVA 967. Psoriasis may involve
o Topical corticosteroid  Extr auditary meatus
955. Erythematous lesion a mild scaling growing more 968. Fox fordyce disease is:
than 7mm/d (Erythema gyratum repens) o Increasing with pregnancy
 Advice to search for malignancy  Decreasing with pregnancy
o Borrelia bord graferi o Disorder of eccrine gland
o Top corticosteroids only 969. Chloroquine may produce
o Antihistone  Premement neurotoxic lesion of retina
956. Which of the following associated with internal 970. The following disaeses assocaited with endocrinal
malignancy ? abnormalitis EXPECT
 Erythema gyratum repens o Acanthosis negricans
o Toxice pidermal necrolysis o Viltigo
o Erythema elevatum dinetum o Neurofibroma
957. Erytherma polycyclic large maculae, a central  Sturge weber syndrome
clearing, the edge increase many mm daily what is the 971. Food can cause both allergic contact dermatitis and
treatment (Erythema gyratum repens) contact urticaria may becaused by:
| P a g e 48

o Preseles o Tetracycline
o Onion o Chloropromazine
o L imon o Chloropropamide
 Tomato 984. Scabies infestation , all true except
972. Erthromycin can be incraesed by o Rarely involve head and neck
 Hypoglycemic drugs o 5 % lindane is effective
o Cis 13 retenoic acid o Benzobenzoates is equally effective to 5% lindane
o Insulin  Itching occurs 1week after infestation
973. Erythromycin increases the level of the following 985. Female pt developed lesions on the cheeck & nose
drugs EXCEPT and diagnosed as rosacea. Rx is:
o Opc o Amoxacillin
 Theophylline  Tetracycline
o Chlorpromazine 986. Which of these syndromes don't present with atopic
o Glycburide dermatitis like lesions-
974. Skin erruptive of the penis the shaft only o Hyper ige
o Fixed drug eruption o Wiskot Aldrich
o Contact dermatitis  Chediak Heggashi
 Allergic dermatitis  Ataxia telangiectasia
975. The administration of erythromycin can even the 987. Delayed inflammatory erethema UVB
blood level of:- o 1 to 6hrs
 Theophylline o 6 to 12hrs
o Oral contraceptives  12 to 24hrs
o Hydrochlorothiazide o 18 to 36hrs
o Insulin 988. Patient is using sulphonamide and had allergic contact
o 13-cls retinoic acid dermatitis before and afraid of cross reaction with ttt
976. Psoralin ultravioletray A (PUVA) all of the following o Neomycin
are true except: o Ehylendiamine
o Useful in vitiligo o Hyroxizine
o Contraindicated in SLE  PPD (para-Phenylenediamine)
 Used to treat some childhood intractable dermatosis 989. Patient with painless ulcer on the glans what is the
o Increase the risk of basal and squamous cell cancer best early diagnostic tool
977. Oral syphilis is the form of o VRDL
 Mucous patches o TEPHA
o Erythema  FTA ABS
o Vesicles 990. Dysplastic nevus syndrome all of the followin are true
978. Azathioprine side effect is EXCEPT:
o Alopecia  Autosomaldominant
o Malignancy 991. A 12 yr old female, non pruritic annular eruption in
o Tetralognecity the left foot for 8 months, looks pale and not scaling.
 Hepatoxicity Had no response to 6 wks of miconazole .
979. Prognostic for SLE is o Discoid lupus erythramatosis
 Antidnadoblestand o Erythema nodosum
o Ro-ssa o Tinea corporis
980. Treatment of erythema nodosum leprosum  Granuloma annulare.
o Prdnisone 40-60 mg o Choricummarginatum
 Thalidomamide100-400 mg 992. Most common tumour with skin metastasis occurs
o Clofazimine 200 mg-d with
981. Parameters for type IV skin patient for hair removal o Renal cancer
with nd-yag laser-  Lung cancer
 30 j/cm and with pulse width 50ms . o Prostate cancer
982. Blistering skin rash is a feature of the following o Laryngeal cancer
dermatoses EXCEPT: o Pharyngeal cancer
o Erythema herpiticum 993. Carcinoma cutis occur due to
o Erythema multiforme o Cancer prostat
o Sulphonamide allergy  Cancer stomach
 Erythema nodosum o Canncer liver
983. All of the following cause photosensitivity EXCEPT: 994. Basal layer most dominant compotent
o Lithium  Collagen IV
 Propranolol o Collagen VII
| P a g e 49

o Laminin 332 1005. The presence of TNF alpha indicates:


o Fibronectin  Malignancy
995. Lower lip presistant edema +( Melkersson-Rosenthal 1006. Prevalence of ACD
syndrome) ln what treatment is not needed o Balsam of peru
o Thalidomide  Nickel
 Isotretinion o Neomycin
o Ketotefen o Bacitracin
o Cs 1007. Child born with area of hypo and depigmention
996. Female with acne and is taking metformin for and his parent has poliosis best treatment for
treatment of polycystic ovary what is your choice hypopigmentation areas
o Finisteride o Eximer laser
 Spironlactone  Culture autologous melanocytes
o Acitretin o Puva
997. 32 years old male with active nodulocystic acne and 1008. Hansen syndrome is best investigated by:
skin type 4 physical treatment after medical treatment  Skin test
o Dermabrasion best modality after medical treatment o Kveim test
o Dermabrasion, chemicalpeeling o Histamine test
 Intralesion steroid o Metcholin test
o Ch.peeling dermbrsion and intralesion steroid o Lepramin test
o Dermabrasion scar revision 1009. Hemangioma best described as
998. Pyoderma gangrenosum lesion search for the  Malformations in the capillaries, too. 8mm in the
following except epidermis
 Bact,viral,fungal infection o Benigin non proliferative condition
o Active acute and chronic hepatitis o Veins malformation
o I.B.D o Affection of the veins, capillaries
o Lymphoma 1010. Painful well circumscribed nodule or papule in the
999. In pyoderma gangrenosa investigation is done to thigh which is compressible is mainly:
exclude:  Dermatofibroma
o Bacterial infection o Cholingenoma
 Active liver disease o Keloid
o Gammopathy 1011. Flushing is caused by all of the following
o Diabetes EXCEPT :
1000. Best adjuvant treatment with NB-UVB for plaque o Carcinoid syndrome
psoriasis  Glucagonoma syndrome
o Methotrixate o Mastocytosis
o Cyclosporine o Alcohol
 Acitretine o Menopause
o Hydroxylurea 1012. Febril patient with erythematous, painful plaque in
1001. Patient had lesion on right axilla with discharging the malar area is:
sinuses and bluish red nodules in the neck and left  Sweet syndrome
axilla. Mantous testis non specific.with stain Gm +ve o Painful necrotic vasculitis
rods, your diagnosis o Migratory nodular panniculitis
 Scrofuloderma o Glucagonoma
o Asperigillosis 1013. A 70 -year- old patient presented with a skin lesion
o Blastomycosis in the left thigh for many years.this lesion is black, size
1002. Case of SLE 1st line of TTT 1x1cm. It started to be more pigmented with bleeding.
o Cs You will advice :
 Hydroxychloroqiune o Cryotherapy
o Thalidomide o Incisional biopsy
o Mtx  Wide excision
1003. Fiddler's neck in violin players due to: 1014. Patient with round brown lesion in face+ cough+
 Acne mechanica splenomegaly+ hepatomegaly+X-ray revealed enlarged
o Allergic contact dermatitis hilar lymph nodes what is the possible diagnosis?
o Irritant contact dermatitis o Myeloproliferative lymphoma
1004. Pemphigus like antibodies are found in all except  Sarcidosis
 Thermal burn 1015. Competes with epidermal melanocytes to form
o TEN depigmentation of the skin is:
o Bullous SLE o Albinism
o Erythema multiforme o Vitiligo
| P a g e 51

 Pityrasis versicolour  Pitted keratosis


1016. Patient develops wrinkled skin with putting hands 1033. Commenst cause of angular cheilitis
in water what is the disease?  Mechanical
o Larva migrans 1034. Heamangioma
 Asthma-eczema syndrome  Commonest bengin vasculr neoplasm in childhood
o Sarcidosis 1035. Phrynoderma due to
o Leprosy  Low essential fatty acids + vit A
1017. Which of the following tattoo pigments is 1036. *Pheochromocytoma
associated with phtotoxic reaction o Metabolic disassociated with n.fibromatosis
 Cadium sulfide 1037. A 56 year-old man presents with blue-gray
o Ferricoxide discoloration on his face, ears, and dorsal hands. What
o Zincoxide is the most likely offending agent?
o Charcoalindigo o Minocycline
1018. Alopecia scaring  Amiodarone
 LE/LP o Chloroquine
1019. Immunotherapy o Clofazimine
o Radiotherapy 1038. Pyoderma gangarenos TTT all EXEPT
1020. Painful tumor? Lend an egg o &interferon
 Leomyoma  Surgical procedure
 Eccrinespirademafibroma o Antibiotic
 Neuroma 1039. Sjogren larsson syndrome
 Dermatofibroma  AR
 Angiolipoma  ischthysoform syndrome
 Neurolemmoma  no cardiopathy
 Enomatroma 1040. Steroid tapering
 Glomustumor  No need less than 2week
 Granularcellcelltumer o No need less 20g/day
1021. Acral malignant melanoma 1041. Steroid topical more potent
 More in black people  Clobetazole propionate
1022. Common warts what is the suitable ttt and safe to o Betamethazone dipropronate
pregnancy o Clobetasole diproponate
o Podphyllin 1042. Ps.pustular treated by
 Electrocautery o S .antibiotic
o Surgical excision  Oral retinoid
o Salicylic acid application o Ditlrane
1023. RAST o Emolint oint
 Detect IgE o Methtrexat
1024. Arthropode stings contains 1043. Sycosis barbae which correct
o Biologically active substances/ o Sycosis babae
histamine/ach/kinins/lecithine o Tinea barbea
1025. *Acid phosphatase o Ps.
o In platelets  Folliculitis barbae
1026. Annular erytherma of RHF 1044. Acyclovir low dose is recommended if recurrent
 Occurs in corps HSV rate of
1027. AIDS IP  1-2m
 3m/10y o 2-4m
1028. Ante cubital fossa o 4-6m
 Not affected in psoriasis o Year
 Affected in s.derm 1045. Antimalarial TTT check for ophthalmslogist at
1029. *Balanitis least
o Not feature of bowen o Every 1-2m
o Nourethritis o Every 4-6m
1030. Botryomycosis is chronic granulomatus disease  Every 6-12m
coused by staph or pseudomonas o A very rare side effect is damage to
 Ttt floxacillin/erythromycin/fuciceam 1046. Anchoring filament
1031. Balanoposthitis its most common cause  Anchoring fibril:type VII collagen lamina densa
 DM  Anchoring filament:antigen2 (type XVII collagen)
1032. Gram +ve bacteria (micrococcus sedentarius) lamina lucida
| P a g e 51

1047. 6 year boy, 3-day he has multiple yellow, crusted o Vitamin D


erosions on the cheeks, chin, and upper extremities .the  Vitamine E
rest is normal. Treatment for this condition ? 1059. HLA in Behcet disease
o Oral acyclovir  Hlab51
 Oral cephalexin 1060. The joint most frequently affected in acne
o Topical ketoconazole fulminans is the :
o Topical 2% hydrocortisone o Elbow
1048. Immediate tanning caused by: o Intervertebral
 320-340  Sterno clavicular
o 320-400 o Sacroiliac
o 290-320 1061. Birt-hogg-dube syndrome is most strongly
1049. The wave length of UVA2 is associated with which of the following malignancies?
 320-340 o Basal cell carcinoma
o 340-380  Renal cell carcinoma
o 340-400 o Trichoepithelial carcinoma
o 380-420 o Eccrine syringo fibroadenoma
1050. Wave length of UVB 1062. 70 kg patient will do excisional surgery, maximum
o 320 - 425 nm amount of 1 % lidocaine should not exceed
 290 - 320 nm o 10 ml
o 311 - 312 nm  30 ml
o 400-760nm o 100 ml
1051. Difference between UVA1 and UVB is: o 250 ml
 UVA1 goes deep in dermis than UVB o 1 ml
1052. Site to detect minimal erythematous dose 1063. Drug which can cause SLE
o Face o Tetracycline
 Abdomen o Isoniazid
o Leg o Pencillin
o Neck o Penicillamine the most
o Arm&dorsum of the hand  All of the above
1053. Which of the following medications would not 1064. Red tattoo which laeser
induce ochronosis ? o Q switch alexandrite
o Cream containing hydroquinone o Q switch rupy
o Antimalarials  Nd-yag 532nm
 Antihypertensives (Beta blockers) o Pulsday
o Products containing resorcinol and phenol 1065. Scleroderma
1054. Papillon-lefevre and haim-munk syndromes have o Ig E
which of the following symptoms? o Ig A
o Right-ventricular cardiomyopathy o Ig M
o Pseudoainhum  IgG kappa
o Eccrine syringo fibradenoma 1066. Best reliable test for DH diagnosisis
 Periodontitis with tooth loss  Skin biopsy
1055. Antiro antihistone anti DNA  Immunofluorescence
 SLE because of drug 1067. All are used for hair removal,all are true EXCEPT
1056. When metastases to the skin occur from a thyroid  The shorter long of waves more target melanin
malignancy, they are usually due to: o The long duration makes moll heat on skin
o Medullary carcinoma o The presence of edema follicular and redness disappear
 Papillary adenocarcinoma o The presence of pigmentation and hypopigmentation
o Follicular carcinoma relieved after 6 months
o Anaplastic carcinoma 1068. Bacel cell is a single layer naturally and when
1057. Pediculus humanus var.corporis (Body lice) is the diseases are proliferating is
vector in: o 2
o Endemic typhus  3
 Epidemic typhus o 4
o Q fever o 5
o Rickettsial pox 1069. Patient back erythematous lesions on back and
1058. Which supplements decrease hemolysis associated genital area with blasma cells in biobsy
in patients taking dapsone?  Plasma cell balanitis
o Vitamin A o Seyphles
o Vitamin B6 1070. Hirsutism associated with which of the following
| P a g e 52

o Anorexia o Ksbutriaul cream


 Juvenile hypothyroidism 1080. Tacrolimos the best for
o Digoxin toxicity  Atopic
o C/ocitrate ? ? o Eczema
1071. Someone with psoriasis treated with methotrexate o Acne
became cumulative dose of 1680 never being biopsy o Rosacea
what is the procedure for the detection of cirrhosis live 1081. *Error in the pathogenesis of rosy
check the level of liver enzymes has o Damage connective tissue
o The level of liver enzymes o About vessel filtrate
o Ultrasound  Infection helikobakatr
 Prcollagen peptide III (pn111n) o There effective vessels
o CT scan 1082. More types porphyria sensitivity and reaction to
1072. Digeorge syndrome solar radiation
 Thymic aplasia(thymus)  Porphyria cutanea tarda
1073. Mach the following 1083. Fetus is covered by:
o Tinea pedis→t.rubrum o Vellus hair
o Kerion →m.canis  Laugo hair
o Verrucous t.pedis→e.floccosum o Terminal hair
o Bullous t.pedis→t.mentagrophytes o None of the above
o Tinea impricata→t.concentricum 1084. Person has single papule on the back of the hand
o Tinea nigra phaeo -→annellomyceswerneckii without symptoms and there blistered biopsy, showing
1074. Pt. Comes to the ER & his chest x-ray reveals the presence of collagen fibers with myosin and PAS
bilateral hilar LN and he gives u previous history of red test negative, what is the treatment (skin disease focal
indolent lesion on the face, ear, nose and lip and on moccin)
tatooo what ur diagnosis:  Eradicate the lesion surgically
o SLE o Waiting and monitoring
 Sarcoidosis o Lesion injection Balstroi
1075. 36 ys patient with multiple asymptomatic papules 1085. What is the keratin in the follicle of hair
yellowish in color in face neck dorsum of hand and old o 4
scar of tattoo since one year he had cough since the  14
same time and chest x-ray showed mediastinal o 18
lymphadenopathy and clinical hepatosplenomegaly o 1
diagnosis: 1086. Patient has a tough hair on the chin and cheek,
o Lymphoproliferative infilteration what the appropriate classification of the degree of the
 Sarcoidosis situation on the scalp
o Fitzpatrick scale (skin colour)
1076. Which one of the following component causes o Ludwig (androgenc alopecia)
contact dermatitis in children ?  Ferriman gallwey (hirsutism)
o Citricacid 1087. Mycosis fungoides big plaque (satisfyings
o Cinnamon recurrent erythema, papules no frequent relapsing
 Poison ivy dasky erythem cd 30+ve)
o Name of very strange tree which is very unrecognizable  PUVA
1077. Bone marrow transplant immune disease what o Metcrksat
druges don't use o Cyclosporin
o Immunosupresent o Tetracycline
 Retinoids 1088. Genetic angioedema best TTT
1078. Pt. Has asymptomatic papule & plaque on trunk o Anti-histamine
back sole a generalized lymphoadenompathy taken ttt o Steroids
and after 24 hours he become worse what he take  Danazol
 Pencillin o Ephedrine
o Erthromycin 1089. Child has a tumor on the left shoulder, two years
o Clindamycin old, what is the best measure(Scleredema)
o Dapson  Control(reassurance)
1079. Pt.has popul itching under arms worst in summer o X-ray
and there ishyperkeratosis in pathology what is the best o Laser
TTT (fox-fordycedisease) o Surgery
 Ritinoid cream 1090. What's more place to keratoacantoma
o Tacrolimus cream o Forearms
o Pova  Scalp
| P a g e 53

o Legs  I V acyclovir and hospital admission


o Palms 1102. All of the following conditions classically produce
1091. What treatment fastest and least disruptive of the lesions involving the soles and palms except:
photosensitizer keratosis o Erythemamultiforme
o 5 floriorassel  Chickenpox
 Azot((99%cure rate)) o Secondary syphilis
o Electriccoagulation o Hand-foot-and-mouth disease
o Alritinoitdattopical 1103. A pink to red maculopapular rash that starts on the
1092. Anti TNF alfa is used to treat psoriasis in each of face and rapidly spreads to the trunk and extremities
the following EXCEPT and is associated with lymphadenopathy (especially
o Psoriaticery throderma postauricular, suboccipital and posterior cervical) is
o Intractable most characteristic of:
o Pustular  Rubella
 Pregnant o Roseola
o Guttate o Rubeola
1093. Pigmentations like rain drops on the trunk with o Varicella
hyperkeratosis on palm and plantar 1104. All of the following statements regarding
 Arsenic erysipelas are true EXCEPT:
o Adison o It occurs most commonlyon the face and the lower
o PUVA extremities.
1094. *Patient has bromide acne  It is caused by staph. Aureus.
o Be in the form of papules & pustule o It is atype of cellitis.
o There is typical o Exam typically reveals a tender, red plaque with raised
o The most important no comedo and sharply demarcated borders.
1095. Any following medications cause skin orange 1105. Hyperpigmented pityriasis alba is
 Rifadin (rifampin) o It may be related to lichenoid pigmentosus
1096. New born baby see the doctor eczema lesions o Pityriasis alba with pigmented contact dermatitis
found on the face arm and groin. Pustule on areas of the  Variant of classic pityriasis alba showing a strong
body with a cold abcess and bronchitis have ige caliber association with dermatophyte infection
of more than 3000 units what is the diagnosis o Post inflammatory hyperpigmented pityriasis alba
 Hyperimmunoglobulinemia E syndrome 1106. An elderly man has papular lesions = Onyx and
o AD cruston the front faster treatment
1097. *Internal malignancies EXCEPT 1107. A tzanck smear can be useful in making the
o Hypertrichosis diagnosis of all of the following EXCEPT:
o Inflammation of the skin and muscle o Herpes simplex (types1and2)
o Bazaks syndrome o Varicella (chickenpox)
 Halobo  Erythema multiforme
1098. *What is true regarding SLE except o Herpes zoster (shingles)
o Photosensitivity may be one of the manifestations of 1108. A positive nikolsky's sign is typical of skin lesions
o Complement high associated with:
o Alarm regard to renal injury o Toxic shock syndrome
1099. The lesions of erythema nodosum are most o Herpes zoster
commonly located on the:  Toxic epidermal necrolysis
o Arms o Contact dermatitis
o Face
 Lower extremities 1109. The presence of "target lesions"(erythematous
o Trunk plaques with dusky centers and bright red borders) on
1100. Vulvar syndrome (vulvovaginitis) vulvo vaginal the palms and soles is classically associated with:
syndrome caused by candida  Erythema multiforme
The vulvo vaginal- gingival syndrome is occurring in o Secondarysy philis
erosive o Disseminated gonococcal disease
 Lichen planus o Pityriasis rosea
1101. An immunocompromised patient presents with a 1110. Patient went for a holiday on the beach and stayed
painful vesicular rash that has a dermatomal six months and contact dermatitis infection what is the
distribution. The most appropriate management for this reason
patientis: o Nickel
o Oral acyclovir o Acid alheirovlorett
o Analgesics and drying compresses  Compounds and sun screens sensitizer paba para
o Systemic corticosteroids amonicbenzoic acid
| P a g e 54

1111. Grover disease characteristic feature 1120. About hair removal with Alexandride ,Nd :Yag
o Papules ,Ruby what is wrong
 Papulovesicles o Perifollicular edema and erythema are the commenest
o Pustules o Hypo or hyperpigmentation will subside after 6
1112. Malignancy which most metastasize to skin monthes
 Lung tumor o The shorter wave length the more destructed melanin
1113. You have received the computed tomography (CT)  Increase pulse duration will reduce number of hair
scan report on a34-year-old mother of three who had a affected
malignant melanoma removed 3 years ago.Originally, it 1121. Describing a case of acute GVHD ( a patient with
was a Clerk’s level I and the prognosis was excellent. leukemia and do bone marrow transplantation he
The patient came to your office 1 week ago develop skin rash) .which is not indicated in treatment
complaining of chest pain and abdominalpain. A CT  Acitretin
scan of the chest and abdomen revealed metastatic o Cyclosporine
lesions throughout the lungs and the abdomen. She is in o Infleximab
your office, and you have to deliver the bad news of the o Steroid
significant spread of the cancer. The FIRST step in 1122. The lesion most commonly found in patient with
breaking news is to: Kasaba-Merite syndrome is:
 Dliver the news all in one blow and get it over with as o Retiform hemangioendethiloma
quickly as is humanly possible. o Epitheloid hemangioendethiloma
o Fire a „warning shot” that some bad news is coming.  Kaposi hemangioma
o Find out how must the patientknows 1123. Delayed erythema caused by UVB develops with a
o Find out how much the patientwants to know it. peak of after :
o Tell the patient not to worry. o 6-12 hrs
1114. The absence of light below 290nm at the earth  12-24hrs
surface from solar radiation is due to:- atmosphere o 24-48hrs
ozone and other gases 1124. Patient has uriticaria and respiratory distress, takes
Stratosphere corticosteroid, withdraws in other special clinic, and
The stratosphere starts just above the troposphere and now has respiratory distress, urticaria, myalgia,
extends to 50 kilometers (31 miles) high. The ozone abdominal pain and joint pain. He had two brothers
layer, which absorbs and scatters the solar ultraviolet died from this case this,patient must take
radiation, is in this layer. o Steroids again
1115. After using balsam Peru cause allergic contact o Bradykinine
dermatitis what you advice pt to avoid : o Thalidomide
o Chemical dyes  C1 esterase inhibitor concentrate
o Painted metals 1125. The question gives a history with the following P.
o Rubber Foliaceus like lesions plus malar erythema exacerbated
 Fragrances by sun. The most beneficial diagnostic procedure:
1116. Pt with proven allergy to balsam of peru should o Skin biopsy
avoid:  Immunoflorescent studies(DIF)
 Perfumes o Electron microscopy
o Metal plates 1126. Massive periappendigeal lymphocytic infiltrate of
o Rubbery shoes lymphocyte. Diagnosis
o Water proof glue  DLE
1117. Percentage of keratinocytes in epidermis: o Lymphocytoma cutis
 90% (80-85%) o Lymphoma
o 20% 1127. Patient with grouped vesicles and DIF show IgA
o 50% DEPOSITS in epidermis:
o 75% o P. Erythematosus
1118. All true about sweet syndrome EXCEPT o P. Herpetiformis
 Show enhancement if give dapsone  Dermatitis herpetiformis
1119. 8 months child presented by red ,hot ,solitary o Pemphigus-lupus overlap
lesion on the Lt shoulder ,it has smooth surface it's 1128. Most potent antihistamine
present since age 2 months ,most appropriate  Doxepin
management: o Hydroxyzine hydrochloride
o Oral steroids o Hydroxyzine pamoate
 wait and watch 1129. Fox fordyce disease
o interferon  Improve with pregnancy
o Surgical removal
| P a g e 55

1130. Pregnant female developed papulovesicular rash in 1139. Most common cause of calciphylaxis:
face then all body before  Chronic renal failure
delivery the infant develop the same rash after 3 days: o Malignancy
 Chicken pox o Hypercalcemia
o Infectious mononucleosis o Hyperparathyriodism
o Measles 1140. Patients with end stage chronic renal failure (CRF)
1131. Male patient with history of depression has suffering from hyperkeratotic papules on biopsy there
rounded and oval pupuric and ecchymotic patches over was transepidermal elimination of collagen. What the
his thighs, what investigations needed help to diagnose safest treatment:
the case :( Gardener diamond syndrom) o Steroid
o Platlet time o Emollient
o Bleeding and clotting time  NB UVB
 Autologous intradermal injection of RBC's o Allopurinol
1132. Familial mediternian fever manifested by o Isotretinoin
o Purpura 1141. The function of laminin:
o Erythema  Connection
 Erysipelas like o Anchoring
1133. A renal patient with purple colored papules on the o Cell adhesion
chin of the tibia that started to ulcerate centrally which o Transmission
is correct o Immunoregulator
 There is associated hyperparathyroidism 1142. Male patient with painful papulovesicular eruption
o This is mild with low mortality of genitalia then resolved and after that developed
o Ca deposition is seen in large vessels unilateral enlarged inguinal lymph nodes and sinuses
o Can treated with drugs to render calcium and phosphate occurred (LGV). Best treatment:
more soluble o Penicillin benzathin
1134. Oral lichenoid eruption o Oflaxacin
 Early patch test for mercury in most pt  Tetracycline
o Late patch test for gold in most pt o Acyclovir
o Patch test for nickel oxide 1143. Case of old man hospitalized prolong developed
o Removal of amalgam over month with red glossy tong
1135. Pseudoxanthoma elasticum all true except o Zinc
 Gene defect chromosome 16 p 3 o Scurvy
o Peau d, orange most common retinal finding  Vit B2
o Doppler blood pressure to investigate intermittent 1144. Male patient with blue greenish discoloration of
claudication axilla and under clothes and dark urinee asking which
o Similar clinical picture by D penicillamine treatment investigation not needed (Alkaptonuria):
1136. Pseudoxanthoma elasticum-yellowish papules on o X-ray lumbar spine
neck h/p defect in elastic fibers is reported to be o Urine porphyrins
associated with all of the following EXCEPT : o Light microscopy (also skin biopsy)
o Penicillamine intake  Bendict test
o Vitamin A administration 1145. Pigment on ear, sclera, nose, darken diaper area.
o Peadu orang retinal affection Which investigation not needed (Alkaptonuria (:
o Multiparity o Need for skin biopsy
 Defect in the 16q3 gen o Urin analysis porphyrin
1137. A female patient developed a red smooth lesion on o Lumbar space X-ray
top of a scar in the abdomen.  Benedict's test
It was excised and histopathological examination 1146. Sweet syndrome associated with all the following
revealed spindle cells in a except:
storiform pattern. CD 34 was positive. The most  Pregnancy
appropriate about this lesion is: o Rheumatoid arthritis
o CD44 helps to confirm diagnosis o Myeloprolferative disorders
 Simple excision is the treatment of choice o Recurrent herpes simplex
o Radiotherapy does not add in reducing the recurrence 1147. Mass in abdomen, solitary, violaceous, and by
o Metastasis commonly occur to regional lymph nodes biopsy showing spindle cells.(leiomyoma) What is
1138. Calciphylaxis the setting is: WRONG:
o Lung team dialism o Nifedipine improve the symptoms
 Hyperparathyroidism o Actin and desmin
o Intoxication o Surgery is the treatment
o Endothelial vascular lesion  Reassurance
| P a g e 56

1148. 30y old women suffer from painfull gradually o PDL


increase in size erythematous plaque.h/p atypical 1157. The first appearance of leukocytoclastic vasculitis
spindle shape cells & occasionally CD34 +ve wt is due to drug:
true:(DSFP) o 2-3mon
 Simple excision is the treatment of choice o 3-4 mon
o Metastasis commonly occur to regional lymph nodes o 24hr
o Radiotherapy does not add in reducing the recurrenc o 48-72 hr
o Cd44 help to confirm the diagnosis  2-3wk
1149. Patient with painful erythematous subcautanous 1158. Patient driver has asymptomatic papule & plaque
nodule on trunk H/P (dermatofibroma) what is the on trunk back sole a generalized lymphoadenompathy
following statement incorrect: after taken treatment. Which antibiotic he takes.
o Treatment is surgical excision (syphilis)
o Nifdipine is symptomatic relief  Pencillin
o +ve actin & desmin o Erthromycin
 Focal nuclear atypia is suggestive for malignant o Dapson
transformation o Tetracycline
1150. Etretinate not given in a pt with high risk of 1159. Patient with renal transplantation after prongled
o DM use of immunosuppressive appear of skin coloured
o HTN papules on arms & trunk on examination raised border:
 hyperlipidemia o Dissemenated urticaria
1151. HG occurs when  Dissemenated porrokrtatosis
o Parturition o GVHD
 2nd trimester 1160. Patient with tender erythematous nodule on the
o 3rd trimester legs since 3 yrs on histopathology shows epetheloid
o Post partum cells and occasional CD30 +VE (lymphoid lymphoma)
1152. Treatment of hypocomplementemic urticarial o Good rognosis
vasculitis  Presence of CD30+ve denotes malignancy
 Hydroxyzine o Search for tuberculous focus
o Epinephrine o Sensetive for radiotherapy
o Hydroxychloroquine 1161. About sweet syndrome all true except:
o Indonethacin o Vesiculopapular lesions
1153. Mechanism of the common local complication of o Fever >38.5
local anesthetic drugs involves: o Dramatic response to dapson
o Cellular hypersensitivity to vehicle  Dramatic response to steroids
o Cellular hypersensitivity to agent 1162. Face and scalp crusted papules on removal leaves
 Ig hypersensitivity to agent erythematous surface best least invasive medicine to
o Ig hypersensitivity to vehicle remove it (solar keratosis)
o Complication due to wrong injection o Retinoid
1154. A man presented with a nodule on the side of the  Cryotherapy
neck with a fistula, histopathology showed o Surgical
hyperkeratosis and papillomatosis and a creature with o Curretage
upper portion lined with squamous epithelium and o 5FU
lower portion lined with apocrine epithelium diagnosis: 1163. About urticaria what is true:
o Parotid gland tumor o 70-80% associated with food or drugs
o Paget's disease o C1 leukotrin the mediator
 Trichoepithelioma  10-20% associated with elevated antihyroid antibodies
o Tuberclous nodule 1164. Linear verrucous lesion on LL :
1155. All the following side effects of cryosurgery o Incontinentia pigmenti
EXCEPT:- o Ichthyosis hystrix
o Hypopigmentation  ILVEN (inflammatory linear verrucous epidermal
o Hyperpigmentation nevus)
 Photosensitivity 1165. About the atopy all true EXCEPT"
o Parathesia  Associated lens opacity 20-30%
o Scarring
1156. 16y girl want to remove red tattoo on her arm the 1166. All Characteristics by atopy except
most effective o Netherton syndrome
 Double frequency ND-YAG o Job syndrome
o QSWITCHED ruby o Ataxia telegentacsia
o Erbium laser  Chediak hegashi syndrome
| P a g e 57

1167. 3y old with Magnolian spot best ttt o Breast


o Co2 laser o Intestine
o Qswitched ruby 1180. First appearance of hair in fetus:
o Qswitched nd yag o 4th week
o IPL o 9thweek
 Counselling o 11thweek
1168. The skin Langerhans cell what is right...  12th week
 Originated from neural crest. 1181. After systemic intake of sulphonamide pt
o Do not contain tennis racket granules. developed acute eczematoid reaction in site of
o Are 80 cells/mm2. previously allergic due to contact with which substance:
o Are not dendritic. o Neomycin
o Are basal.  Paraphenyldiamine .
1169. Cause of fillagellate hyperpigmentation 1182. The most common neoplasm accompanying
 Bleomycin paraneoplastic pemphigus is:
o Rifampicin  Chronic lymphatcytic leukemia
o Chloroquine o Hodjkins lymphoma
1170. The most important side effect of isotretinoin: o Adenocarcinoma
 Teratogenicity  Non-hodjkins lymphoma‫لو موجودة‬
o Eruptive xanthomas 1183. Patient have urticated wheal in back, (Darier sign)
o Facial erythema which druge avoid it
o Chelitis  Asprin
1171. Stimulation of keratinocyte by: o Corticosteroid
o TNF alfa 1184. Cutaneous manifestation of Darier's disease may
o Interferon alfa be exacerbated by: -
o Interferon Gamma o Dapsoneo
 TGF beta o Betablockers
1172. Dermatofibroma:  Lithium
o Haphazard spindle cells in fascites o Thiazides
o Haphazard collagen bundles o Antimalarial
o Fibroblast in myxoid vascular stroma 1185. Component in basement membrane :
 Mucin between collagen bundle  Collagen
1173. Fluconazole mechanism of action  Laminin
 Lanosterol 14 demethylase o Fibronectin
o Squalene epoxidase 1186. Lesion on medial canthus (description of BCC)
1174. IFN inhibitors in psoriasis least indicated in o The question is: what is the best treatment
o Fulminant pustular psoriasis  Mohs Syrgery
o Sever erythrodermic psoriasis 1187. A 15 years patient with multiple depigmented and
o Psoriasis with pregnancy hypopigmented lesions with areas of hyperpigmentation
 Guttate psoriasis presented to you , The father has an area of
1175. Vulvovaginal gingival syndrome depigmentation with white hair in the scalp. The most
 Lichen planus appropriate line of treatment.you choose is
o Behcet (piebaldism):
1176. SSSS o PUVA
 Staph aureus group 2 subtype71 o NB-UVB
o Sepsis and death  Culture melanocytes
o Identify organism by doing culture from base of lesion 1188. A case of porphyria characterized by
1177. Persistant lip edema with recurrent facial palsy hyperpigmentation and skin fragility. The
Description of milkerson rothensal syndrome .What causativedefective enzyme is
TTT not needed(NSAI,CS, antibiotic,imusupressive)  Uroporphyrinogen decarboxylase
 Clofazimine 1189. 32 old female was treated last year from vitilligo
1178. Male patient with painless ulcer on the shaft of the by puva she now has residual lesion in her face ..best
penis and had sexual intercourse since (9 < ? < 90) day, tretment is:
what is the most confirmatory test: o Puva
 Dark ground microscopy o Excimer
1179. Conversion of testestrone to dihydrotestestrone  Autologus transplantation of melanocytes
ccures in females in" 1190. Hypocomplementemic urticarial vasculitis is
 Sebaceous gland present with
o Ovaries  SLE
o Liver o Wegner granulomatosis
| P a g e 58

o Churg strauss 1203. Old patient with hypo pigmented patches over the
o Systemic sclerosis back with infiltration since 6 months on
o Esinophilic fasciitis treatment…..recently, he developed multiple papules
1191. A case of female patient when exposed to cold and vesicules over face and extremites, What should
she develops cyanosis of finger and when rewarm her you give him?( MF recently developed drug eruption)
fingers return red , is associated with which disease o NSAID
 Raynaud's disease  Steroids
o Rheumatoid arthritis o Pencillin
1192. Pregnant female in the 2nd trimester with o Tetracycline
vesiculobullous lesion and recurrent (HG) treatment o Retinoids
with 1204. Acitritin not used in :
o Systemic steroid o Immunosupression
 Topical steroid  Pregnancy
o Cyclosporine 1205. A cell has a smooth surface and normal looking
1193. Female patient with annular lesion in the leg which cytoplasm is
is asymptomatic , histologically showing pallisading  Merkel cell
granuloma with mucin and negative rheumatoid factor, 1206. Describe a case of mastocytosis ( patient with
(Granuloma annulare ) the treatment is macules and develop urticaria when rubbing ) which
 Reassurance and follow up drugs is contraindicated
o Steroid  Aspirin
1194. A 23 yr old pregnant diabetic female comes with 1207. Drug that lead to orange color in contact lenses of
lake of pus lesions on skin (pustular psoriasis). patients :
Treatment is  Rifampicin
o Methotrexate o Erythromycin
o Azathioprine o Metronidazole
o Retinoids o Tetracycline
 Cyclosporine 1208. Endemic kaosi sarcoma which is true
1195. Erythema nodusm – sarcoidosis - hilar  LN are more affected than classic
lymphadenopathy what is the diadgnosis o Contagious
 Loffegren syndrome o Ulcerative
1196. A case of melanoma in face5x7cm after surgical 1209. Scabies not present in
removal develop dog ear scar, which is not the TTT of  Scalp
this scar: o Wrist
o Tensile removal scar o Axilla
o Remove excess skin o Genitalia
o M plasty 1210. A case of psoriasis more than 35 % of his skin
 T plasty with no improvement with PUVA the next step is
1197. Circular depression over rim of helix, linear  Acitrtin
earlobe creases, facial vascular malformation, 1211. Lepromatous leprosy site of skin biopsy is
macroglossia o lesional site
 Beckwith–Wiedemann syndrome  Perilesional site
1198. Boiling degree of liquid nitrogen o anesthetic site
 −195.79 1212. Girl with erythematous and cold toe finger every
1199. What is pregnancy category X . winter since 7 years is due to
 Isotreinoin  Perniosis
1200. Vogt-Koyanagi-Harada Disease: o Acrocyanosis
 Retinal detachment is the first association o Raynaud's phenomenon
1201. Child complain of rash for the first time in his life 1213. Pt. Cicatrical alopecia follicular surrounded by
over lower lig with no systemic complain…biobsy papules&pustules&biopsy showed neutrophiles&
shows vasculitis (HSP) what is the treatment? eosinophiles infection from one year TTT(folliculitis
 NSAID decalvans)
o Steroids o Hydroxycloroqin
1202. Patient with recurrent oral and genital ulcers and o Greazovolvin
recurrent episodes of conjunctivitis With papules  Refampicin
spreading over trunk and extremites Lab analysis done o Minoxidile
twice with WBCS 18000 TO 20000,over six months on 1214. Chloroquine following is true:
treatment…what is the best option?( behcet disease) st
o 1 choice in psoriatic arthritis
nd
o NSAIDS o 2 choice in rheumatoid arthritis
 Steroids o Used in SLE
| P a g e 59

 May cause retinal damage 1230. Anhidrosis compensated with hyperhidrosis, Tonic
1215. What is best treated with PDT pupil, absent tendon reflexes :
 Superficial basal cell carcinoma  Ross syndrome
o Keratoacanthoma 1231. Patient with painful oral lesion ,skin lesion are
o SCC suprabasal acantholysis, what not in this disease (PV):
o BCC o Dsg3
1216. Reiter's disease (SSSS) TTT is: (β-lactamase- o Dsg1
resistant antibiotic) o Lectin
 NSID  Bpag2
o Sys. Steroid 1232. Stage 3 MM good prognosis when spread to :
o MTX o Liver
o Dapson  Skin (SC)
1217. What is true about protein c: o LN
 It inhibits protein s o Brain
o It activates va, vii- a vitamin K-dependent glycoprotein- 1233. Devil’s punch is
1218. Gottron's sign is present in  Purpura simplex on the thigh of young women
 Dermatomyositis 1234. 10 years old child with fever rash prominent of
1219. Early manifestation of dermatomyositis tongue papillae tachycardia irregular pulse
 Heliotrope erythema o Toxic shock syndrome
 Gottron's sign  Scarlet fever
1220. Male patient has circular bluish discoloration on o Measles
the nail of left thumb which is very painful in cold o Kawasaki
weather and when touch with the tip of the pin cause 1235. Warfarin
pain, the treatment (Glomus Tumor): o Telogen effluvium
 Excisiton 1236. The least disease associated with psoriasis
o Cryo o Atherosclerosis
o Nifidpine o Vitilligo
1221. Not a type of vasculitis o Non skin melanoma
 Multicentric reticulohistiocytosis  Atopic dermatitis
1222. Patient with papular pruritic eruption (Pruritic 1237. TLR toll like recepter ,what is TRUE:
papular eruption of HIV)TTT is: o Probiotic use TLR
 PUVA  TLR used in ttt of acne vulgaris
o Nifedipine 1238. Young patient with papulovesical lesion on nape
o Ivermectin of neck and dorsum of hands appears after sun exposure
o Permethrin during summer and resolve after it:
1223. Drugs for delusional parasitosis o Actinic prorigo
 Pimozide  Polymorphic light eruption.
1224. Defention of perforatin disease o Phytophotodermatitis
 Extrusion of dermal materials through the epidermis o Actinic reticuloid
with minimal damage 1239. Case of psoriasis in children treatment with narrow
1225. All cells are present in epidermis EXCEPT band UV effectiveness increased in combination with
 Mast o Glucocorticoids
o Merkelcell o Cyclsporin
o Mellanocyte cell  Tacrolimos
o Langrhans o Nothing
1226. Keratin in basal layers: 1240. 45 y old man, sudden eruption all over the body
 14&5 with palm and foot involvement. Most likely Dx is:
1227. TTT of pityriasis lichenoides chronica: o Syphilis
o PUVA  Erythema multiforme “most probably”
 Topical tacrolimus o Erythema nodosum
o Oral retinoids o Fixed drug eruption
o Topical retinoid o Pityriasis rosea
1228. Pattern of nuclear antibodies in SLE: 1241. The most painful tumor is
 Spickles  Eccrine spiradema
o Nuclear 1242. IgA antiidodies to tissue trasglutaminase
1229. Patient with hyperpigmented tinea alba it’s due to: diffrentiate this disease from other groups
o Postinflammatory hyperpigmentation after tinea alba o DH
 Dermatophyte infection with tinea alba  Linear IgA disease
o UNEXPLANIED o Bullous SLE
| P a g e 61

o EBA o Nifidipine
1243. The best time for complete improvement of o Cryosurgery
comedones by tretonin is o Sclerotheraoy
o 1-2w 1253. Patient with intense pruritis on genitalia since 4
o 2-4w months, lesions on vulva are glistering. HP showed
o 4-12w atrophy and hyalinization DIAGNOSIS :
 6months o Lichen planus
1244. Lesion on the thight show dimpling on squeezing o Morphea
between thumb what most possible histopathology  Lichen sclerosus et atrophicus
discharging nodules since 5 years in the axilla; inguinal 1254. Enzyme in cell envelope :
and groin  Transglutaminase
o Acne vulgaris  Loricrin
 Acne inversa 1255. Pt with facial furrows and exagerated nasolabial
o Axillary scrofluderma fold. What is TTT?
o Actinomycosis  Botulinum neurotoxin injection
1245. What factor increases the risk of haemangioma? 1256. Sock and gloves caused by:
 Prematurity o Cytomegalovirus
1246. Female patient complain of hair loss since 1 year o Parvo virus
with christmas tree appearance,what the diagnosis:  EBV
 Female pattern hair loss 1257. Young male with mycosis fungoides, prgnosis:
o Alopecia areata  Single patch more common than older
o Telegon eff. o Single patch not respond to electron beam
1247. 23 years old with brown red nodules in the face 1 o Develop sezary syndrome
year ago no nerve affection and no any other lesions in o Bad prognosis than elderly
the hole body, Grenz zone & lymphocyte in 1258. ESRD with hyperkeratotic papules, biopsy
histopathology. What is the diagnosis: showed: transepidermal elimination of collagen, best
o Cutaneous leishmaniasis TTT:
o Verruca vulgaris o Emollient
 Lymphocytoma cutis  Steroids
1248. Patient with nodular single lesion with prominent 1259. AR disorder with photosensitivity, retinitis,
follicular pores without any systemic affection, pigmentation, mental retardation, DNA instability:
histopathologically showing perivascular  Cockayne syndrome
lymphohistiocytic inflammatory infiltrate with Grenz 1260. Patient taking dapsone 100 mg /day he will suffer
zone formation from:
o BCC
 Hemolysis
 Granuloma faciale
o Agranulocytosis
o Sweet syndrome
1261. Haemangioma affecting:
1249. All are associated with acne fulminans EXCEPT :
 Veins and capillaries
 Rapid response to antibiotics
o Male more than female 1262. Commonest malignancy in nail plate:
o Ulcerative lesions
 SCC
o Leukaemic lesions
1263. 45 years old patient with itchy erythematous
o High fever
popular lesions on face, V area of chest for 3 years,
1250. Patient with photosensitivity , malar rash ( red,
severe in summer, improve in winter test to diagnos:
scaly ) developed generalized p foliaceus like lesion ,
(photodermatitis)
best investigation to confirm diagnosis :
o Skin biopsy for H/P  Photo patch test
1264. Birt–Hogg–Dubé syndrome associated with:
 Immunoflurocence staining
 Renal cell carcinoma
o Peripheral eosinophilia
1251. Male with Wilson disease taking penicillamine for  Fibrofolliculomas
2 years developed hyperkeratotic erythematous 1265. Which type of xanthoma not due to lipid
follicular pulgging lesion on back. What is true: abnormalities:
 Transepidermal elimination of collagen o Plane
o Transepidermal elimination of elastin o Tendineous
o Transepidermal elimination of keratin  Xanthoma disseminatum
1252. Nail cyanotic circular area on exposure to cold, o Popular
very painful on touch with tip of pen (Glomus tumor) o Verrociform
how to deal: 1266. A case of decreased lacrimation with increased
 Exision sweating and generalized pruritus .Diagnosis:
 Familial dysautonomia
| P a g e 61

o Sensory neural defect alopecia(myxedema), dystrophic nail and macroglossia,


1267. Patient type 4 skin did a phenol peeling and didn't what is true:
use sunscreen , will suffer from: o There is angioid streaks in the rettina
o Persistent erythema o Serum and urinary electrophorisis are mandatory
 Post inflammatory hyperpigmentation  There is low T3&T4
o Post inflammatory hypopigmentation 1278. Lichenification means all are true EXCEPT:
o Acneform drug eruptions o Hyperpigmentation
1268. Oral and genital ulcer, bilateral conjunctivitis,  Acantholysis
leukocytosis, perphral eosinophilia, biopsy showed +ve o Decrease sweating
mutation gene TTT ( leukemia AML) : o Increased scrat-marking
 Imatinib o Increased thick
o systemic steroids 1279. Latest systemic oral medicine for scabies is
o malphan  Ivermectin
1269. Grisofulvin not used in these cases EXCEPT: o Benzyl benzoate
o Hypersensitivity o Carbaryl
o Hepatic o Gamma benzene hexachloride
o Renal 1280. Which of the following is found least frequently in
 Pulmonary dysfunction patients with TEN (Toxic Epidermal Necro-lysis)?
o Porpheria o Anemia (erythrocytic)
1270. Patient with bilateral flaccid axillary bullae. o Lymphopenia
 Pemphigus vulgaris o Neutropenia
1271. Most common cause of systemic photosensitivity  Eosinophilia
 Sulphonyl urea 1281. Cytoskeleton include all of the following
o Sulphone EXCEPT:
o Dyphenylhydramin o Tonofilament
1272. A young person is having comedones and papulo- o Actin
pustular acne over face and trunk and back. How will o Tubulin
you manage the patient?  Filaggrin
o Topical retinoic acid 1282. A 19-year old male presents with several
 Oral doxycycline + topical retinoic acid comedones, papules and pustules on face and trunk.
o Topical clindamycin The appropriate drug of choice for the patient would be
o Oral retinoic acid o Topical retinoic acid
1273. Bexarotene, an oral retinoid is indicated in  Topical retinoic acid + Oral doxycycline
o Severe acne vulgaris o Topical clindamycin
o Pustular psoriasis o Topical azithromycin
 Mycosis fungoides 1283. After oral injection of 8-methyloxypsoralen, how
o Lichen planus long an interval should be allowed before UVA
1274. Newborn baby presented with eczematous rash, radiation is given in PUVA treatment?
recurrent abscess on the face, arm and groin. Pustule on o 1 hour
areas of the body with a cold Abscess and bronchitis,  2 hours
IgE serum > 3000 units, low platelet level, high o 3 hours
eosinophilic. 1000% o 4 hours
 Wiskott-Aldrich syndrome 1284. Earliest feature of tuberous sclerosis is ?
o Job syndrome  Ash leaf spot
o Netherton syndrome o Shagreen patch
o All of the above o Facial angiofibroma
o None of the above o Gingival fibroma
1275. Spongiosis due to: 1285. A paient suffer from flushing which becomes
o Intracellular edema wrose with alchol,dyspnea and attacks of diarrhea, CT
 Intercellular edema scan shows affection of the liver, which investgation
o Appotosis of cell should be done for diagnosis(carcinoid syndrome) :
1276. halo navus in hp o Serum dopamine
infiltrate dicloses navus o ANCA
lymphocte in blood o Serum sertonin
if halo nevus disappeared clinically in histologic nests  Urinary 5-hydroxy indole acetic acid
of nevus cells will remain 1286. What is normal flora of the skin:
1277. Patient suffers from waxy papules and plaques o Staphylococcus aureus
over the trunk, spontanous brusing,  Staphylococcus epidermidis
o Micricone
| P a g e 62

o Corynebacterium  HHV6
1287. Photo irritated test can be read after: o Parvovirus
 24 1300. Pregnant female 8th month with pigmented lesion
o 36h on face (melasma) the TRUE is
o 48h o Recurrence decrease with subsequent pregnancy
o 72h  May affect maxillary, forehead and mandible
1288. What is the most common types of melanoma? o Affect 10% of female
 Superficial spreading melanoma o Not occur with contraceptive pills
o Acral lentiginous melanoma o More common in men than in women
o Mucosal melanoma
o Nodular melanoma
o Polypoid melanoma
1289. The drug with the least side effects for the
treatment of SLE is:
o NSAIDs
o Methotrexate
 Corticosteroid 1301. diagnosis:
o Hydroxychloroquin o PV
1290. Drugs cause acne form eruptions are: o T.bedies
o Lithium o pompholyx
o Hydantion  indian sandals dermatitis
o Steroid
o Androgen
o Contraceptive pills
 All of the above
1291. Regarding targetoid hemosiderotic hemangioma,
what is true:
o Multiple vascular lesions on legs of eldery people 1302. photo patient complaining of this recurrent rash
o Closely related to HIV with labial affection what is the diagnosis:
 HP shows dilated vascular channels surrounded by  Erythema multiform
hemosidren &extravasated RBC's
o May be mistaken by lymphatoid papulosis
1292. Diabetes mellitus is more common with:
o Erythro coproporphyrin
o Erythro pro porphyria
o Congenital erythropoietic porphyria
o Hemosidrosis 1303. Picture of patient with two patches of hair less area
 Hemochromatosis in the scalp. Best treatment is
1293. keratin 19  Intralesional steroids
o Basal cell of mucous membrane o Minoxidil
 Bulge of hair follicle
1294. Child 2 months, papules and plaques heal by milia
& scarring, nail dystrophy,erosion on palat. What is the
pattern of staining:
 Bp 230,anti k14, collagen IV at base
o Bp 230,anti k14, collagen IV at roof
o Bp 230 at roof ,anti k14, collagen IV at floor
o Bp 230,anti k14, at roof collagen IV at floor
1295. The best ttt for child with atopic dermatitis is 1304. Photo of Alopecia Aerate
 Topical tacrolmius in children the safe treatment
1296. In measles, infective prodroma is followed by o Intralesional steroid
 Exanthema on fifth day o UVB
1297. Ephelides (Freckles) on face o Systemic steroid
 Q-switched ndyag 532  Topical steroid
o Co2 laser 1305. picture of alopecia areata, expected nail lesion wih
1298. Red lunula in SLE this disease is:
 Related to periungual erythema or chilblain lupus  pitting
o Land mark in SLE o onycholysis
1299. Exanthema subitum (roseola infantum) o onychomycosis
| P a g e 63

1306. Child with alopecia  Excision biopsy


with accompanying graphic: o Colonoscopy
o Alopacia avenata o Reassurance
o Androgentic alopecia o Antifunal
o Traction alopecia
 Trichetillomania

1313. (picture) nail with redish swelling under the nail


bed
 Glomus tumour
1307. Figure of child with hair very tighten and oval area
o Subungual fibroma
with no hair TTT:
o S.C.C
o Oral antifungal
o Melanoma
 Advice change the habit of
tightening hair
o Antipsychotic

1314. Figure : small vesicles on hand and forearm of


baby the histopathology shows deposition of igA at
dermoepidermal with subepiderma bullae with
eosinophils and neutrophils what is diagnosis:
 Ch. Bullous dermatosis of childhood
1308. Diagnosis
 Kiolonycia
o Onychorrhexis
1309. Cause of Kiolonycia
 iron defficencyanemia

1315. Child with vesicles in genials; chest; diagnosis


 IgA dermatosis

1310. Picture of patient with


two fingers with destroyed nail.
Best treatment is
 Intralesional steroids
1311. Arrow pointing to the side.
what is it:
o Onychomadesis (shedding at beginning) 1316. Picture for baby with crust redness face with
 Onycholysis (separation at the tip )4 ‫فى عدم وجود‬ history of atopy, what is true:
o Koilonychia (spoon nails) o History of conjunictivities not
 Onychodystrophy(nail destruction in LP) run in families in this condition
 More prone to staph infections
o Not inherdit condation
o Washing with soap will
decrease condition

1312. 65 year male with black longitudinal


with blurred edges line in index finger,
periungual show black colour,
what you advise: 1317. Patient clinical pic. And pathology (perforating
dermatosis) what investigations not needed:
| P a g e 64

o Fasting blood glucos 1322. Viltigo (picture) what type of laser used
o HIV in treatment of this condition
o Kidney function test  Eximre
 Serum cholesterol o Alexandrite
o PDL
o Nd - yag

1318. Clinical and histological picture of squamous cell


carcinoma in face what is true regarding this type of
tumor?
1323. A 8-years child presented with leukotrichia ,
o Adenoid type is the worst type
forehead and upper eyelid hypopigmented macule.
 Horn pearl is present in well differniated tumor
Best topical treatment is
o Lesion in ear shows more metastases
(vitiligo ):
o Corticosteroid
 Topical tacrolimus
o Retinoid
o Topical PUVA

1319. Histology of 5 y nodule near the nose 1cm with


rolled edge : 1324. Figure face lesion on chin
 BCC o HSV
o SCC o Lupus vulgaris
o Melanoma  Tooth abscesses dental sinus

1320. A picture of an old man has nodule near the eye,


since 1.5 year ago now start to ulcerate what is probable
diagnosis: 1325. Figure of thickened toe nails -ve fungal with skin
 BCC colered papules in forehead neck chest and progressive
o SCC also father and sister has the same what we must
o Eroded Seborrheic keratosis invsestigate:
o Malignant melanoma (DNA-biobsy-KOH for exclude onychomycosis-rarely
eye )
o Chest
o ?
1326. Figure of thickened toe nails -ve fungal with skin
colered papules in forehead neck chest and progressive
also father and sister has the same:
1321. Scourge of the nose 2.5cm 3 weeks ago with a  Pachyonychia congenita
picture of the anatomy patients o Nail psoriasis
diagnosis o Dermatophyte onychomycosis
o Saprophyte dermatomycosis
 Keratoacantoma
| P a g e 65

1327. Figure of pin point skin colored papules under both


eyelids diagnosis: 1332. adult patient with connective tissue navus
 Syringoma (shagreen patch) with history of counvulsion since
o Follicular palioma childhood, what is the diagnosis:
 Tuberous sclerosis

1328. Pt. Serpiginous papules cocentric 5x4 on dorsal of


hand (feet) not itching. In histopathology 1333. Fibrofolliculomas,
granulomatous tissue & mucin. trichodiscomas &
Which ttt acrocondrons are characteristic of
o Steroid o Vorner syndrome
o Dapson o Kindler syndrome
 Reasurance o Kasabach-Meritt syndrome
o Topical steroid o Louis-Bar syndrome
 Birt-Hogg Dube syndrome

1329. Picture of erythematous patch with vesicles,


female present with face, axilla, nape of neck, -ve
immunofluorescence, histopath acantholysis without
dyskeratosis ,what is diagnosis: 1334. Picture of volacious patch
 Darier on medial side of feet after
o Hailley hailley antibiotic for URT :
o Paraneoplastic pemphigus o TEN
o Pemphigus folacoeus  Fixed drug eruption
o EM
o Exanthum drug eruption

1330. Purple papules on the face of baby remission after:


 Blue muffin baby
1335. Case description (heterosexuals+ red or violaceous
+ yellow to green "halo,")
 HTV-selected kaposi sarcoma

1331. Picture of child with multiple small brownish


papules in his face with history of mental retardation 1336. Sores and color smear gram sno image gram-
and epilepsy(Tuberous sclerosis) negative bacilli what is the appropriate treatment
 Angiofibroma (chancroid)
o Neurofibromas  Siprofloxacin 2
o Molluscum o Penicillin
o Acne  Azethromycin 1
| P a g e 66

1337. Figure of multiple painful genital ulcers and after 1342. 33Y old man suffer from many years from like this
few days developed enlarged inguinal mass choose the lesion on the back.neck & groin erythematous plaque
right answer: bulla & erosin –ve direct immuonoflurosence
o LGV o Pemphegus valgaris
o Granulomainguinale o Pemphegus folicaous
 Chancroid o Darier disease
o Chancre  Haily-haily disease

1338. Pt can do which treatment for wart (figure penis)


by himself:
 Imiqimod 1343. 5 y eodema of lips and gingival biopsy give
o Podiphilin epitheliod granuloma doesn'respond to antituberculous
o Cryo (Picture of mouth &teeth,lip hypertrophy +facial
o Electrocautery palsy),biopsy showed granuloma
o TB
o Leprosy
 Milkerson rothenthal syndrome

1339. Photo of scrotum brownish papules


(angiokeratoma) what is the best treatment:
 Liquid nitrogen
1344. picture of old female has red face…a case of breast
cancer since 2 years…recently develop red face is it?
o cut LE
o chilblan LE
o Lupus timdus
 dermatomyositis
1340. Recurrent penile eruption
 HS
o Chancre
o Chancroid
o Lgv
1345. A case of hyperpigmentation in the left cheek and
present in the left sclera (nevus of Ota )is treated by
 QS Nd:Yag
o PDL
o IPL
o Excimer

1341. A picture of a pregnant patient with plaque


psoriasis what treatment
o Acetretin
o PUVA
o Cyclosporin
o Systemic steroid
 Tar 1346. Picture of chin (in my opinion it was normal)
Patient complain of this pigmentation with lat side of
sclera have dark pigmentation
What to do to help him
 Q switched ND YAG
o Long pulsed ruby
o Intense pulsed light
| P a g e 67

1347. Itchy scalp , 1353. Photo of tongue ( black tongue): due to :


cervical LN on hair  Antibiotic
 Lice o Antihypertensive
1348. Photo of hair with lice o Antiviral
TTT: o Hypoglycemic
 Permithrin
o Antifungal

1349. Photo of a man from the back


with erythematous patch 1354. Picture of ear auricle with two papules in it (gouty
( multiple patches) tophi),investigation needed
what is the TTT:  Uric acid in urine
 PUVA o Xray
o Steroid o ECG
o Cyclosporine

1355. Diagnosis;
 Becker nevus

1350. Papules on both knees


what drug cause it
(sarcoidosis and EN)
 Tetracycline
o MTX 1356. becker's navus discription on back
o Isotretonin
 Sulphonamides

1351. Nevus target like (and other description)


 Cockade nevus
1352. Child with central papule 1357. Picture of hyperpigmentation in all the body , palm
and concentric layers of crease , hypopigmentation and weakness, the diagnosis:
hyperpigmentation diagnosis is  Addison
o Meyerson naevus
o Sutton nevus (halo navus)
 Cockade nevus
| P a g e 68

1358. picture of infant, inhertence of ectodermal


dysplasia 1363. Picture of kerion, what is the treatment:
 XL-R  Systemic greisofulvin
o Topical antifungal
o Antibiotic
o Incision &driange

1359. Picture of erythemtous rash


on sun exposed neck ,
shoulder & chest after 3 days
of taking thiazide 1364. A patient suffers from papluvesicular lesions on face
 Phototoxic reaction with sesonal varaitions (increase in summer), with biobsy
o Photoallergic reaction there is clear fluid & this hisopathological picture
o FDE (Hydrocystoma), diagnosis:
o Polymorphus light eruption  Uniocular cyst lined by flat cells
o Tadpole

1360. femal patient with lesion in axilla, as fissures, pits,


plaques and scarring. the diagnosis;
 acne inversa 1365. Most common cause of dermatitis is
 Nickel
o Lead
o Paraphenylendemine
o Ethanol

1361. Pic of buttocks with scally


erythematous rash in
old female,
what is the diagnosis: 1366. Venous lake on lip, laser used in treatment
 mycosis fungoid  long pulse nd yag
o pytriasis rosea
o taniea cruris
o pustular psoraisis

1367. Hepatic patient showing all nails with proximal


white and distal parts. the patient is positive hepatitis-B
virus but inactive. Which of the following disease can lead
to the same picture (terry nail):
1362. Whitish nail in old patient what is the diagnosis:  Congestive heart failure
 Onchmycosis, o Retinoid therapy
o psoraiasis o Hypothyroidism
o lichen planus o Lichen planus
| P a g e 69

1368. Incotenia pigmenti 1374. Lichen Planopilaris


 XLD • Presents commonly with
alopecic patches with
perifollicular erythema;
± tenderness or pruritus
• Histology: lichenoid
lymphocytes around affected
follicles with vacuolar
degeneration within affected
1369. Diagnosis infundibula, perifollicular fi brosis
 molluscum • Treatment: antimalarial, topical/intralesional
corticosteroid
1375. Hereditary Hemorrhagic Telangiectasia
(Osler–Weber–Rendu)
• AD, HHT1 and HHT2
• Multiple mucocutaneous
and GI telangiectasias:
1370. ??? epistaxis, telangiectasis
 Eczyma herpeticum (skin/mucosa),
GI bleeding, pulmonary arteriovenous malformations

1376. Erythematosus minor dose is the dose of UVB


which leads to erythema after application on the
terminal part
 True
1371. Patient with anaesthetic macule behind the ear for 5 1377. These are no documented studies that some foods,
years with enlargement of the greater auricular chocolate, jellyfish can share in the production of acne:-
nerve. Best treatment regimen will be (with picture)  True
 Dapsone 100mg daily + rifampicin 600mg monthly for o False
6 months 1378. Acne vulgaris can occur in normal sized sebaceous
o Dapsone 100mg and clofazimine 50mg daily + gland with normal sebum secretion
rifamicin 600mg and clofazimine 300mg monthly for o True
12 month  False
1379. Most of acne patients don't seek medical
Treatment
 True
o False
1380. Small multiple papules are characteristic in steroid
acne
 True
o False
1381. Benzyl peroxide is powerful antibacterial as it
1372. Photo Eman him glasses and hand with lesions
bacterial count & transformation of triglycerides—
nerve involvement with weakness and loss of o True
sensation and the nerve become more thikness is
 False
began
1382. As a group sebaceous gland not in size & no in
o TT
sebum production in patients of acne than those without
o BT
acne:-
o B
o True
 BL
 False
o LL
1383. Apocrine glands in human anogenital regions are
1373. The peri-ungual fibromas of tuberous sclerosis are
small and atrophic and not produce smell:-
 Found in adult patients o True
 False
1384. Sebum amount affect acne:-
 True
o False
1385. Acne may extend as far as buttocks
| P a g e 71

 True o True
o False  False
1386. Acne in an 12 month old infant usually indicates 1401. Polyester clothes reflect more UVR so it is used
endocrinal abnormalities more than cotton clothes in hot climate
o True o True
 False  False
1387. In normal persons apocrine sweat glands have no 1402. The darker the skin complexion in blacks, the
odor:- highest the prevalence of acral pigmented lesions:-
 True  True
o False o False
1388. Acne vulgaris predispose to Rosacea 1403. A major advantage of intralesional steroid
o True injection is that it can be done without incising or
 False draining the lesion, avoiding the possibility of scar
1389. Rosacea is effectively treated by metronidazole formation:-
 True  True
o False o False
1390. Rhinophyma is a precancerous condition 1404. Lichen planus exhibits the isomorphic (Koebner)
o True phenomenon:-
 False  True
1391. With oral methotrexate treatment of psoriasis liver o False
biopsy is not indicated in stable liver:- 1405. Lichen straiatus is self-limiting leaving linear
o True hypopigmentation without known cause-
 False  True
1392. Immediate tanning occurs due to UVA while o False
delayed tanning occurs due to UVB:- 1406. Macule is only change in color
 True  True
o False o False
1393. Conversion of testosterone to the active form 1407. Pustule is a primary skin lesion
dihydrotestosterone occurs in the target organs  True
(prostrate, vas deferens, seminal vesicles & epididymis o False
under effect of 5:-alpha:-reductase enzyme) 1408. Vesicle is a pus containing epidermal cyst less than
 True 0.5 mm
o False o True
1394. Scleroderma 1st manifestation is swelling of  False
hands & feet 1409. Nodule differs from papule in depth & size
o True  True
 False o False
1395. The optimum time for culturing candida is 24 1410. Crust is usually honey colored
hours while for dermatophytes is 2 weeks  True
o False o False
 True 1411. Wheel is the characteristic lesion in urticaria
1396. The difference between side effects of etretinate o True
& isotretinoin is that isotretinoin cause facial dryness &  False
redness of lips 1412. Wheel occurs only in urticaria
 True  True
o False o False
1397. PUVA is the treatment of choice in PLE:- 1413. Fissure is a breach in the skin without loss of
o True substance
 False  True
1398. 70-80% of total life dose of solar radiation is o False
received before the age of 20 years 1414. Lichenification is a feature of chronic contact
o True dermatitis
 False  True
1399. MED is the minimal dose of UVR that induce o False
Erythema & is measured by mj/cm or j/m 1415. Tinea versicolor is caused by epidermophyton
 True o True
o False  False
1400. Telangiectasia are the hallmark of perioral 1416. Girisofulvin is effective treatment in candidiasis
dermatitis:- o True
| P a g e 71

 False o True
1417. Infantile atopic dermatitis is mostly in the nape  False
o True 1434. Antimicrobial agents are the commonest drugs
 False cause drug reaction
1418. In acute dermatitis there is a Spongiosis  True
 True o False
o False 1435. Exanthematous reaction usually appears a week
1419. Adult atopic dermatitis is mostly in the extensor after taking the drug
surfaces  True
o True o False
 False 1436. In case of drug taken for 1st time, reaction will
1420. Chronicity & family history are minor criteria in occur in the 1st 7 days
atopic dermatitis  True
o True o False
 False 1437. Unpredictable drug reaction is dose independent
1421. Chronic eczema is characterized by oozing  True
vesiculation o False
o True 1438. Intolerance is the inability of the individual to
 False tolerate therapeutic doses
1422. In atopic dermatitis, there is in cellular immunity  True
 True o False
o False 1439. Isomorphic phenomenon is another term for
1423. Unilateral hyperhidrosis on face & neck you must keener's phenomenon
see intra-thoracic.  True
 True o False
o False 1440. Hidardentis suppurativa is the disease of apocrine
1424. Pityriasis rosea is contagious gland
o True  True
 False o False
1425. Straie gravidarum is present on top of papules of 1441. Heliotrope rash & gottron's papules are the
L.P characteristic cutaneous lesions in dermatomyositis
o True  True
 False o False
1426. Acantholysis is a loss of the coherence between 1442. Herpes zoster is the effect of Iry infection with
epidermal cells varicella zoster virus
 True o True
o False  False
1427. The first lesion of Pityriasis rosea is the herald 1443. L.P lesion in oral cavity is violaceous in color
patch o True
 True  False
o False 1444. L.P lesions in oral cavity are pruritic
1428. SLE may associated with peripheral neuropathy o True
 True  False
o False 1445. Nail involvement percentage in L.P is 10-15%
1429. Dermatomyositis have malignant association in  True
children o False
o True 1446. Glans penis is the most affected area of genital
 False tract in L.P
1430. En coup de saber is the linear variety of morphea  True
 True o False
o False 1447. Flexural psoriasis is called inverse psoriasis
1431. B. Blockers may exacerbate psoriasis  True
 True o False
o False 1448. The commonest nail change in psoriasis is
1432. Fixed drug reaction may occur in mucous onycholysis
membranes o True
 True  False
o False 1449. Anogenital involvement in psoriasis is common
1433. Fixed drug eruption is type hypersensitivity o True
| P a g e 72

 False o True
1450. Volar psoriasis is caused by staphylococcus  False
o True 1467. In eczema systemic steroids are always can be
 False used
1451. Psoriasis is extremely pruritic o True
o True  False
 False 1468. In Seborrehic dermatitis, palms & soles may be
1452. Pustular psoriasis is caused by staphylococcus involved
aureus o True
o True  False
 False 1469. The ulcerative form of impetigo is called ecthyma
1453. Psoriasis of scalp cause scarring alopecia  True
o True o False
 False 1470. Candidiasis is common with humeral
1454. Alopecia areata may have nail change immunodeficiency
 True o True
o False  False
1455. Steroids usually used in treatment of acne 1471. Candidiasis is common with cellular
o True immunodeficiency
 False  True
1456. In dermatomyositis there is distal symmetrical o False
muscle weakness 1472. Ramsey-Hunt syndrome affects facial nerve,
o True auditory nerve, but not trigeminal
 False  True
1457. In systemic sclerosis there is nail changes o False
o True 1473. Tinea versicolor may be transmitted by long
 False contact
1458. Iry lesions in herpes simplex are smaller & less o True
painful than recurrent attacks  False
o True 1474. Psychosis barbae causes scarring alopecia
 False  True
1459. Hidardentis suppurativa is treated by 7 days o False
antibiotics 1475. Drowsiness caused by antihistamines is considered
o True a type of drug reactions
 False  True
1460. RAST is the best test for allergic dermatitis o False
o True 1476. In acute allergic contact dermatitis, lesion usually
 False is not definite
1461. Isotretenoin is used in treatment in comedogenic o True
acne  False
o True 1477. Tzank smear is used to diagnose bullous
 False pemphigoid
1462. In vitiligo there is no change in sensation of the o True
skin  False
 True 1478. Steroids play no role in treatment of bullous
o False pemphigoid
1463. Antibiotics are proved to be used in Seborrehic o True
dermatitis  False
o True 1479. H.G may occur with the 1st menstrual cycle
 False o True
1464. In Seborrehic dermatitis, the preauricular greasy  False
scales is the 1st symptom 1480. H.G occurs in the 3rd trimester
o True o True
 False  False
1465. Purpura is the commonest unwanted drug reaction 1481. Gluten free diet in iga dermatosis improves the
in the skin patient condition:-
o True o True
 False  False
1466. Pruritis is almost always due to allergy
| P a g e 73

1482. Iga deposits in mucosa of DH not affected with o True


treatment:-  False
 True 1494. Pemphigus vegetans is usually localized in
o False intertrigenous areas
1483. Best reliable diagnosis of DH is DIF of non  True
lesional skin o False
o True 1495. Pemphigus vulgaris may result in
 False hyperpigmentation
1484. Dapsone is the drug of choice D.H  True
 True o False
o False 1496. The type of pemphigus that have racial factor is
1485. Result in involved skin as well as uninvolved skin pemphigus vulgaris
in D.H  True
 True o False
o False 1497. Bullous pemphigoid is common at age 65-80
1486. Gluten free diet is rapidly restore D.H  True
)‫ شهر‬12-5( ‫بعد سنة‬ o False
o True 1498. In pemphigus vulgaris prednisolone is given at a
 False dose of 100-200 mg/day
 True
1487. D.H like psoriasis, tend to occur in extensor o False
surfaces 1499. Iry lesion in pemphigus vulgaris is a flaccid bullae
 True on an erythematous base
o False  True
o False
1488. D.H is more common in male 1500. Steroids are contraindicated in herpes gestation
 True o True
o False  False
1501. Vitiligo, malignant melanoma carry good
1489. D.H is common in old age prognosis
o True o True
 False  False

1490. D.H. is pruritic


 True
o False

1491. Jugenal biopsy shows characteristic findings with


D. H
 True
o False
1492. In Cicatricial pemphigoid, iga is present against
lamina lucida
o True
 False
1493. Oral lesions are common in bullous pemphigoid
1502. Penetration of the skin by UV light is:- greater for longer wavelengths than for shorter wavelengths.
1503. Delayed UVB tanning appear after:- 72 hours.
1504. Immediate UVA tanning appear after few min and fade within 6-8 hours
1505. UAV1 range:- 340-400 nm.
1506. UVA2 range:- 320-340 nm.
1507. UVB range:- 290-320nm.
1508. Drugs cause acne form eruptions are:- Lithium, Hydantion, Steroid, Androgen, Contraceptive pills.
1509. Retin A in acne treatment give approving after:- 6 to 8 weeks.
1510. Retinoid not given to patient with:- hyperlipidemia.
1511. Gram-ve folliculitis is classified as different from acne vulgaris mainly because of:-its flora and the absence of
comidones.
1512. Eumycetoma (IP 20y ) ( fungi ) – keticonozale, Itraconazole
| P a g e 74

1513. Actinomyretoma ( IP 5y ) ( Bacteria ) - streptomycin + dapson


-streptomycin + TMP-SMZ
- dapsone + TMP-SMZ
1514. Erythema anular centrifugum – 2-5 mm/day
1515. Erythema gyratum of repens – 1 cm/day
1516. Pateint take a drug at what time leukocytoclastic vasculitis appear:- 1-3 week
1517. Common complication of cryotherapy:- hyperpigmentation but serious complication is scar
1518. In acne vulgaris, the precursor of large inflammation is:- the white head.
1519. Difference of hyper immunoglobulin ige and atopy:- high level of ige> 50,000.
1520. Which test done to diagnosis of drug eruption:-skin test. While Allergy Patch test
1521. The effectiveness of Methotrexate in psoriasis is apparently due to its effect on what cellular phase:- S.
1522. Tuberous xanthoma and tenderous xanthoma defect in:-LDL receptor
1523. AIDS related Kaposi sarcoma sites:- retro-auricular.
1524. Classical Kaposi Sarcoma occur in:- Jewish race and Mediterranean people.
1525. Ichthyosis vulgaris (AD) defect in:- filaggrin
1526. Punch out ulcer on toe and also on bony prominence with no pulsation, no hair and painful:- pyoderma
gangrenosuus
1527. Halo nervous accompanied with:- Vitiligo and present with:- nevocellular nevus.
1528. In most cases, halo nevi are:- compound nevi.
1529. Desmosome Complex is:- coherence between epidermal cells.
1530. The following drugs decrease the serum level of itraconzol:- phenytoin, rifampicin.
1531. Normal sebum content:- Waxester + Squalene + Cholesterol Esters (Wsce) ‫ويسكي‬
1532. Staph is sensitive to all except:- vancomycin.
1533. The least effect of this antibiotic on the soft tissue infection:- doxycyclin.
1534. Testosterone is converted to dihydrotestosterone in:- target organs.
1535. Sever pruritus specially in:-mycosis fungoidis.
1536. In hereditary angioedema the following is right:- AD, serum C4 is reduced, hereditary deficiency of C1
estrase.
1537. Which sensation is lost first:-
Temperature then light touch then pain then deep touch.
1538. The best treatment of pyogenic granuloma is:- surgical cautary.
1539. Causative virus of oral hairy lecoplakia:- E.B. virus.
1540. Antimalarial ophthalmic examination every- 1-3m
1541. White nails due to:- liver disease.
1542. Causes of molluscum contagiosum:- pox virus.
1543. Lichen nitidus Histopathology:- granulema
1544. Porokeratosis Histopathology:- coronoid lamellae
1545. Kawasaki treatment:- aspirin 100mg/kg +IV Ig
1546. What is the most potent steroid:- clobetasone propionate.
1547. The principal route of excretion of methotrexate is:- renal.
1548. Orally administered methotrexate is best absorbed:-on an empty stomach,
1549. Napkin rash sparing folds are due to:- diaper dermatitis.
1550. Phenol toxicity on:- cardiac, renal.
1551. The most common site for keratoacanthoma is:- sun exposed area (Elderly)
1552. The best reliable test for diagnosis of DH is:- DIF.
1553. Porphyria cutanea tarda has a relation with:- hepatitis C.
1554. Most common fungal infection all over the world:- tinea pedis.
1555. Stress induced urticaria:- adrenaline.
1556. On exposure to heat :- Cholinergic urticaria
1557. Lichenification cause:- hypohidorisis.
1558. Carcinoma cutis occur due to:- cancer stomach.
1559. Leishmaniasis the etiology:- leishmania tropica.
1560. The absence of light below 290nm at the earth surface from solar radiation is due to:- atmosphere ozone and
other gases.
| P a g e 75

1561. The epidermal turn over rate on the scalp in dandruff is:- increased.
1562. Cell envelope of corneacyte – loricrin
1563. Langerhans cell probably functions as:- contact allergen trap.
1564. The most important source of infection for neonatal candidiasis is:- the mothers vagina.
1565. The following organisms does not commonly colonize healthy skin:- candida albicans.
1566. Highest carrier rates of staph pyogenes are found in:- infancy.
1567. The pathogenesis of nail pits in psoriasis is:- punctuate parakeratosis.
1568. Which of the following is not characteristic of puepura hyperglobulinemia (waldenstrom syndrome):- occurs
predominantly in middle age men.
1569. Most common superficial infection in the world:- Ptyriasis versicolor.
1570. Acyclovir is given as prophylactic in recurrent Herpes simplex:- 200mg ∕ 5 time or 400mg ∕ 3 time
1571. The mortality rate of untreated steven-johnson syndrome is:- 5-15%. ‫ فى الكتاب‬10-20%
1572. E.M. associated with T.E.N. mortality rate :- 15-30%. ‫ فى الكتاب‬30%
1573. Beau's lines in nails have a counterpart in hair known as:- poh's marks.
1574. Kyrle's disease affect :- Hair follicle and skin between hair follicle.
1575. The moat commonly identified aetiologic factor in squamous cell carcinoma in nail bed is:- X radiation.
1576. The photosensitizing action spectrum of the furocoumarines is:- 300-360.
1577. Turnover time of sebaceous gland cells as determined in radioactive thymidine update studies is about:- 7 days.
1578. The association of diabetes mellitus, hepatic cirrhosis, cardiac failure and cutaneous hyperpigmentation is
characteristic of:- hemochromatosis.
1579. In addition to white leaf shaped macules early diagnostic sign should be sought for diagnosis of epiloia?:-
poliosis.
1580. Lyme arthritis is associated with:- erythema chronicum migrains.
1581. Pyoderma gangrenosum, especially if superficial and bullous should be cause one for:- leukemia.
1582. Characteristic features of jop syndrome do not include:- pseudomonas abscess.
1583. Susceptibility to tinea imbricate to be related mainly to:- inheritance.
1584. Wegners granulomatosis is generally most effectively treated with:- cytotoxic drugs.
1585. Topical therapy with salicylic acid preparations:- is very unlikely to result in dangerously elevated serum
salicyliate levels in adults.
1586. Phototherapy of neonatal hyperbilirubinaemia is accompanied with light having a wave length of:- 400-500 nm.
1587. Female pregnant has tens bulae and vesicles started from umbilicus to all abdomen which recure every
pregnance best treatment is (HG):- Corticosteroid
1588. Female pregnant has tense bullas and vesicles started on the stria sparing the umbilicus best treatment:-
antihistaminics and tropical steroid
1589. Female with Hirsutism the scale use to measure its degree:- Gallwy Scale
1590. 20 nail dystrophy best treatment:- intralesional steroid
1591. Best treatment of hypocomplemenic urtecarial vasculitis:- sedative antihistaminic
1592. Anchoning filament presnt in:- lamina lucida
1593. The commonest side effect of hydroxyurea therapy as used for psoriasis:- anemia.
1594. A side effect of chloroquine therapy, peculiar to blonde and red hair patients is:- bleaching hair.
1595. By electron microscopy, human apocrine secretion is:- apocrine and merocrine.
1596. Thermoregulatory vasomotor activity occurs in:- arterovenous anastomoses. Metarterioles, arterioles.
1597. Cerumem consists:- sebum and apocrine secretion.
1598. The characteristic follicular eruption seen in patients on lithium carbonate is:- hyperkeratotic, extensor.
1599. In the treatment of mycoaia fungoides with topical nitrogen mustard;- delayed hypersensitivity commonly
develops, desensitization can usually be achieved if necessary.
1600. Thiazide cause:- Lichenoid drug eruption.
1601. Site of diabetic neuropathy:- Soles.
1602. In tuberous sclerosis the lesion are:- angiofibroma
1603. Epidermal cleft opened and the cells die:- Cytolysis.
1604. Which type of porphirya causing severe photosensitive:- Erythropoitic protoporphyria (EPP).
1605. Drugs induce SLE: Hydralazine, Isoniazid, Procainamide
1606. All are right about Pemphigus folliacious except:- Mucosal lesion.
1607. Lichenoid reaction occur with:- color film
| P a g e 76

1608. % of eye affection in:- Atopic Dermatitis- 10%


1609. Dose of 8MOP in patent 70 kg before PUVA:- 70 X 0.4 :- 28 mg.
1610. The usual site to detecting minimal erythematous lesion:- Abdomen.
1611. Drug cause pellagra like symptoms:- INH
1612. Ulcer of big toe with cold limb and intermittent clouducation is:- Arterial ulcer.
1613. In LGV:- bubos, inflammatory adenitis.
1614. Wiscott-Aldrich syndrome + chronic granulomatous disease is XLR while hyper ige syndrome AD
1615. Hereditary angioedema all false except:- AD
1616. Fox for dyce disease is disorder in:- Eccrine gland
1617. Fox for dyce spots is disorder in:- Sebasceous gland
1618. Lamina fibroreticularis=lamina densa

Match
1619. The diseases with the enzymes in which there are defects:-
 Phenylketonuria phenylalanine hydroxylase,
 Albinism tyrosnase,
 Ochronosis homogentistic acid oxidase,
 Homocystinuria cystathione synthetase.

1620. The drugs with the appropriate characteristics:-


Cyclophosphamide alkylating agent, potent immunisuppresant effect,
Azathioprine purine analoge, potent anti inflammatory effect,
Methotrexate potent immunisuppresant effect,
Cytosine arabinoside potent immunisuppresant effect,
Azaribine none

1621. Match the primary tumors with the relative frequencies of cutaneous metastasis in women:-
Carcinoma of breast, A
Carcinoma of colon, B
Melanoma C
Carcinoma of the ovary. D

1622. Match the ligual changes to disease-


a) Cleft or lobate tongue -oral facial digital syndrome.
b) Unilateral macroglossia -nurofibromatosis.
c) Edematous macroglossia -superior vena cava syndrome.
d) Scrotal tongue -mikerson Rosenthal syndrome.
e) Macroglossia associated with gingival hemorrhage -multiple myeloma.
f) Congenital absence of paillae -family dysautonomia.
g) Hyperextensibility of the tongue -ehlers danlos syndrome.

1623. Match the following:-


Vit C………….(50mg. Gingivitis + scurvy + bleeding)
Vit A………….(5000 I.U. night blindness + pitots spots + ker)
Vit B12………..(20 mg nicotinic acid . Megaloblastic anemia)
Vit B2…………(Riboflavin 2 mg. Angular stomitits)
Niacin………..( B6. Pellagra with 3D diarrhea, dementia, dermatitis)

1624. Match the following:-


Chancroid 1-5 days Haemophilus ducreyi Amoxicilline + clavulanic acid
LGV 3-30 days Chlamudia (L1 , L2 , L3) Ciprofloxacin
Syphilis 9-90 days Treponema pallidum Penicillin G
| P a g e 77

Rectal Gonorrhea 2-5 days Neisseria gonorrhoeae Ceftrixone


Chlamydia urithritis 2-3 weeks Chlamedia trachomatis Doxacycline
Granuloma inguinal 8-80 days Donovani bodies Doxacycline or TMP-SMZ or
Ciprofloxacin
1625. Painfull tumors LEND AN EGG

-L Leiomyoma -A Angiolipoma -E Endometrioma


-E Eccrine spiradenoma -N Neurolemmoma -G Glomus
-N Neuroma -G Granula cell
-D dermalefibroma

1626. T. Capitis M. Canis, M. Audouini, M. Gypseum (small ectothrix) (Gray patch)


M. Canis, M. Audouini, M. Gypseum (small ectothrix) (Kerion)
T. Tonsurans, T. Violaceum (Endothrix) (Black dot)
T. Schoenleinii (Favic) (Favus – Adult)
T. Pedis T. Rubrum & occasionally E. Floccosum (Hyperkeratotic - moccasin)
T. Mentagrophytes & occasionally E. Floccosum (Vesiculobullous)
T. Mentagrophytes (Ulcerative)
T. Mentagrophytes (Macerated toe web)
Onychomycosis T. Rubrum (DLSO)
T. Mentagrophytes (SWO)
T. Rubrum, T. Mentagrophytes (PSO) 90% with AIDS
T. Rubrum, T. Mentagrophytes (EO)
T. Imbricala T. Concentricum

Here are some points mentioned in previous exams


1. Lamina lucida: weakest BMZ layer.
2. Herpes zoster: thoracic is much more important than all other types.
3. Herpes zoster: Nose tip is supplied by a branch of ophthalmic (nasociliary)
4. Herpes simplex: Recurrence may be a cause of erythema multiform.
5. Herpes simplex: Recurrence 6 times/year necessitates prophylactic ttt.
6. HIV: The most common bacterial infection (aureus)(superficial folliculitis)
7. HIV: the most common visceral affection: GIT
8. HIV: the most common occurs mostly in GIT, lungs, NOT in penis.
9. HPV: Serotypes 16 & 18- C accuminatum- cancer cervix
10. Epidermodysplasia verrociformis: ↓↓CMI (normal humoral)
11. Hand-foot-mouth disease: • Coxsaxkie
12. HPV6: roseola infantum
13. Parvovirus B19: Erythema infectiosum
14. T pedis: most common fungal infection.
15. T. Capitis: adults T. Tonsurans (most common)
16. T. Capitis: children: T tonsurans, M canis (most common)
17. T. Nigra Palmaris; Phaeonnellomyces (Exophiola) werneckii
18. Piedra, black: Pieraia hortae
19. Piedra,white: Trichosporon beigelii
20. Aspergillosis: neutropenia & systemic steroids (most important risk factors)
21. Aspergillosis:(culture features)
22. Mycetoma: dapsone + streptomycin ttt of choice
23. TB; lupus vulgaris, commonest cutaneous variant
24. TB; lupus vulgaris, apple jelly nodules (on diascopy,characteristic)
25. TB;scrofuloderma:lateral neck (most common site)
26. TB, BCG: a live attenuated vaccine
27. Leprosy: skin biopsy (best diagnostic test)
28. Leprosy: early nerve affection (in TT)
| P a g e 78

29. Leprosy: Fernandez test +ve 24-48h


30. Leprosy: Mitsuda test +ve 3-6 weeks
31. Scabie:Spares mid-back
32. Sarcoidosis in black, HLA-B8 (highest incidence)
33. Sarcoidosis: Thyroid, cruptococccosis, lymphoma, lung cancer, granuloma annulare (associations)
34. Sarcoidosis; kveim test (6 weeks): brucella, lymphoma, Crohn\s –false positive
35. Syphilis; early congenital; early manifestation appear during:- 2nd to 10th weeks (6th weeks).
36. Syphilis;transmission to the fetus occurs in:- benging of the 2nd trimester.
37. Syphilis; early congenital; early manifestation ; rhinitis-nasal snuffles
38. Syphilis; early congenital; late manifestation ; condylome lata
39. Syphilis; secondary; palms-soles (most common site of rash)
40. Syphilis; mucous patches (most common presentation)
41. Syphilis; Painless ulcer
42. Syphilis; FTA-abs test 1 to turn positive
43. Syphilis; mixed chancre; T. Pallidum +H ducrei
44. Gonorrhea; cervix (most common site in females)
45. LGV: pubo
46. Atopic dermatitis: pruritus (the cardinal manifestation)
47. Allergic contact dematits: Nickel (most common agent in +)
48. Allergic contact dermatitis: Rubber (common affect feet)
49. Allergic contact dermatitis: Neomycin (most common antibiotic)
50. Allergic contact dermatitis: Plants (most common agent in children)
51. Allergic contact dermatitis: Cinnamon oil(most common agent in chewing gum)
52. Allergic contact dermatitis: Phenylene diamine (most common agent in color developer, hair dye)
53. Allergic contact dermatitis: Painters, cementmprick,shoe makers (most common affected)
54. Allergic contact dermatitis: Prick test-contact urticaria within 15-20 minutes (if possible)
55. Allergic contact dermatitis: ttt sensitizers e.g. Neomycin (most common agent in leg ulcer)
56. Melanocyte number ↓↓with age
57. Peutz-jegher syndrome-jejumum ileum (commonly affected sites)
58. Mongolian spots :sacrum (most common site)
59. Neurofibromatosis:6 CAM 5mm, prepubertal, major criterion
60. Vitiligo: face (best ttt response)
61. Hypopigmentation :Pityriasis versicolor (tyrosine, competitive inhibition)
62. Hypopigmentation: cryotherapy (commonest complication)
63. Solar radiation: vitamin D synthesis, psychological well-being, healthy skin
64. UVA: striking earth surface 150 folds UVB
65. UVA: maximum erythemogenically 9-1mm
66. UVA: quick tanning (oxidizes preformed melanin)
67. UVA: wrinkles (fibroblast injury, mild dermis)
68. PUVA: psoriasis, atopic eczema, pityriasis rosea, pruritus with hemodialysis (uses)
69. PUVA: sunburn i.e erytherma, pain commonest side effect
70. UVB (acute)erythema,12-24 h
71. UVB-delayed tanning,72 h new melanin formation
72. UVB (chronic)aging, cancer
73. Malignant melanoma:50 years old (peak incidence)
74. Malignant melanoma in ♀ mostly on:- lower limb.
75. Malignant melanoma in ♂ mostly on:- trunk.
76. Malignant melanoma occur in:- giant nevus.
77. Malignant melanomas of the oral mucosa are located on the:- maxillary mucosa.
78. Malignant melanoma related to sun exposed:-lentigo malignant.
79. Malignant melanoma all are right except:- hair in hairy nevous.
80. Malignant melanoma (Acral) is:- Central European and Asian.
81. Malignant melanoma: brain site affected
82. Malignant melanoma: giant congenital nevus(4-10% time risk)
| P a g e 79

83. Malignant melanoma: pigment in a nevus (a sign of malignancy)


84. Malignancy melanoma: hair growth in a nevus (NOT a sign of malignancy)
85. Lentigo simplex/café au lait patches :clinical differential is NOT possible
86. Keratoacanthoma: sun exposed site (most commonly)
87. Squamous cell carcinoma: old scar least metastic risk
88. Squamous cell carcinoma modified skin e.g. Glans (highest metastatic risk)
89. Squamous cell carcinoma: immunosuppressed 18 folds higher risk
90. Squamous cell carcinoma 7th decade peak incidence
91. Squamous cell carcinoma the condition is most likely to evanuate: ertythroplasia of Queyrat,
92. Squamous cell carcinoma a cutaneous viral infection which may evanuate: epidermodysplasia verruciformis.
93. Pilomatrixoma: most common tumor to ossificate
94. Hodjkin’s pruritus, ichthyosis, hyperpigmentation (common skin manifestations)
95. Histtiocytosis x: Bone (the commonest organ affected)
96. Liposarcoma:(the most common malignant tumor)
97. Lung cancer: cutaneous metasteses (most common cause)
98. Painful tumors: BLEND AN EGG.
99. Psoriasis: Epidermal cell turnover 3-4 days (Normal 26-28 days)
100. Psoriasis:HLA-B13,17
101. Psoriatic arthropathy affect:- 5% of psoriatic patient.
102. Psoriasis: pustuar (Dapsone ttt of choice)
103. Psoriasis: nail pitting (most common sign)
104. Psoriasis: pin-point hemorrhage (due to suprappaillary thinning)
105. Psoriasis: Acanthosis of rete ridges is NOT present
106. Psoriasis: Granular cell layer NOT present
107. Psoriasis: Hyperkeratosis NOT present
108. Psoriasis: Immune complexes are NOT present
109. Psoriatic arthropathy affect:- 5% of psoriatic patient.
110. Lichen planus: Hepatitis C is correlated
111. Lichen planus: Hpertrophic (most itch)
112. Lichen planus: Lichenification no sweating
113. Lichen planus; nail longitudinal lines
114. Lichen planus,mucosal,parakeratosis(absent granular layer)
115. Lichen planus: Subepidermal igm(DIF)
116. Lichen planus: Levamisole, Methotrexate (ttt of choice)
117. Pemphigus vulgaris: Nickolisky’s sign:lack of epidermal coherence
118. Pemphigus vulgaris: Nickolisky’s sign: positive in
119. Pemphigus vulgaris: Worst prognosis in bullous disease
120. PCT: Liver disease
121. Pseudo PCT: kidney disease
122. CBCD “chronic bullous disease of childhood” HLA-B8
123. CBCD: granular iga (both normal and affected skin)
124. DH: Spontaneous remission (10%)
125. DH: Dapsone (ttt of choice)
126. DH: Sulphapyridine 1-1.5g/d (in dapsone intolerance)
127. HG:2nd trimester (onset)
128. HG: C3, igg (vs. PUPPP)
129. HG: morbidity/motality(vs.PUPPP)
130. BP: igg,C3 (immunoreactants)
131. BP: incidence of underlying cancer NOT ↑↑
132. SCPD (subcorneal pustular dermatosis) best treatment dapsone
133. EM (erythema multiform):H.S, orf, suiphonamides (major causes)
134. EM: deposits around superficial blood vessels
135. Darrier: onychorrhexis
136. Behcet’s disease: young + (peak incidence)
| P a g e 81

137. Behcet’s disease Topical steroids (NOT effective ttt)


138. Behcet’s disease: Urethritis (NOT a manifestation, vs Reiter’s)
139. Reiter’s syndrome: HLA-B27
140. Pathergy test: pyoderma gangrenosum + sweet syndrome + Behcet’s
141. Acute febrile neutrophilic dermatosis: Myeloproliferative(major cause)
142. PG (pyoderma gangrenosum):Psoriasiform lesion NOT present
143. SLE:DIF high sensitivity (areas NOT intensely treated)
144. SLE:Skin lesions: Livedo(patient at risk)
145. SLE: Kidney functions (best prognostic factor)
146. SLE: ndna(active with severe nephritis)
147. DLE: lupus band test NOT positive (positive in normal sun-expposed)
148. DLE: DIF high sensitivity (areas NOT intensely treated)
149. DLE: DIF high specificity (areas NOT sun-exposed)
150. MCTD: (Mixed connective tissue disease):specified nuclear igg.
151. SS:(systemic sclerosis)GIT(most affected organ)
152. SS: igg (predominant Ig)
153. Sjorgen-Larsson: Anterior congenital cataract is NOT manifestation
154. Inflammatory bowel disease; skin affection: PG, SCPD,EED
155. Crohn’s disease :oral lesions (the more common ones)
156. Pityriasis lichenoids chronica ttt: erythromycin/tetracycline
157. Tongue, Black Hairy: bacteria, antibiotic
158. Tongue; Smooth, sprue, pellagra (beefy red)
159. Tongue, Geographic: psoriasis(pustular),Seborrhea, Reiter’s
160. Tongue; strawberry: measles
161. Tongue; scrotal: congenital/acquired (Melkerson-Rosenthal)
162. Alopecia areata: Hair re-growth potential in all cases
163. Ichthyosis; X-linked: ↓↓steroid sulphatase
164. Xerosis: the most common geriatric dermatosis
165. Xerosis: the most common cause of pruritus
166. Heriditary-Haemorrhagic telangiectasis: autosomal dominant AD
167. Acanthosis nigricans: adenocarcinoma of the stomach
168. Acanthosis nigricans: pruritus (NOT in erythema gyratum repens)
169. Acanthosis nigricans:insulin resistance
170. Palmar eryherma in pregnancy, LE chronic liver, lung, thyroid disease
171. Spider telangclectaisa; chronic liver, children
172. Howel-Evans syndrome: Palmoplantar keratoderma+ cancer esophagus
173. Erythroderma: lymphoma-leukemia (25%of cases)
174. Azthiprine: T-cell
175. Cyclophosphamide: B-cell
176. Allylamines; Disrupt ergosterol synthesis earlier (squalene epoxidase)
177. Fluconozale: Disrupt ergosterol synthesis: later (primarily: lanosterol 14-a demethylase;a cytochrome
45dependent)(secondarily:sterol A22-desaturase;a cytochrome 450-dependent)
178. Itraconazole: same as fluconazole(both are triazoles)
179. Itraconazole: Demethylase, a cytochrome P-450-dependent enzyme
180. Fluoconazole: Demethylase, a cyctochrome P-450-dependent enzyme
181. Itraconazole. Terbinafine: reach the nail plate through nail bed (faster)
182. Griseofulvin, ketoconazole: reach the nail through nail matrix 18m
183. Aspirin: Urticarial
184. Azathioprine: malignancy
185. Bleomycin: Black nails:
186. Cyclophosphamide . Myelosuppression. Haemorrhagic cystitis, azoospermia
187. Cyclosprines: Nephrotoxicity
188. Cytotoxics; anagen effluvium
189. Dapson hemolytic anemia, agranulocytosis
| P a g e 81

190. Erythromycin: Hetotoxicity,Steven-johnson


191. Etretinate: hair thinning
192. Griseofulvin: photosensetivities SLE exacerbation
193. Ichthyotic: Triparanol
194. Isotretinoin: dryness(worst:teratogenicity)(contraindication: ↑↑lipids
195. Itraconazole: Hepatitis, cholestatic jaundice
196. Ketoconazole: nephrotoxicity
197. Lichenoid: Gold,color developers
198. Lindane: neurotoxicity
199. Lithium: Acne steroid, Darrier Methotrexate: Myelosuppression,hepato(dose-dependent)NOT nephro
200. Minoxidil: ECG changes, ↑↑ hair thickness and weight
201. PABA:NOT with sulfoamides
202. Penicellamine: Anetoderma, Atrophoderma, Morphea, Pemphigus.
203. Penicillin: Morbilliform (commonest rash:NOT FADE)
204. Photosensitivity:Diphenyhydantoin
205. Photoxicity:Tetracyclines,frusemide,sulphonamides,thiazides,phenothiazine,benoxaprofen,naproxen,griseofulvi
n.
206. Pigmentation:chlorpromazine
207. Psoralens: Occular damage
208. Quinones: Retinal toxicity
209. Red yellow tattoo: Cadmium sulphide
210. Steroids: Acneiform
211. Teracycline: Onycholysis:
212. Thalidomide: Teratogenesis
213. Topical anesthetics: Allergic contact dermatitis

Dr.Ashraf Abo
✂ ✂ ✂Suture removal
---------------------------
✂Head 5-7 days
✂Face 3-5 days
✂Eyelid & eyebrow 3-5 days
✂Trunk 5-7 days
✂Extremities 7-10 days
✂Surface of joint 10-14 days
✂Hand 7 days
✂Sole of foot 7-10 days

�derma ....note.. �
☀Most common / Drug of choice mcqs ☀
1) Most common organism causing tinea- Trichophyton rubrum
2) Most common organism causing tinea capitis- Trichophyton violaceum
3) Most common cranial nerve involved in Hansens- facial
4) Most common nerve taken for nerve biopsy in Hansens- radial cutaneous (upper limb), sural (lower
limb)5
) Most common cause of mononeuritis multiplex - Hansen (India), DM (world)
6) Most common cause of ENL- LL> BL7) Most common cause of a negative Slit skin smear in Hansen-
neural leprosy
8) Cause of Type 1 reaction- BB> BT> BL
9) DOC for type 1 and type 2 reaction- steroids
10) DOC for chronic, recurrent ENL- thalidomide
11) Most common side effect of dapsone- hemolytic anemia
| P a g e 82

12) Most common side effect of clofazimine- pigmentation


13) Most common cause of inverted saucer lesion- borderline leprosy
14) Most common cause of leonine facies- LL
15) Earliest sensation lost- temperature
16) Most common Hansen- Borderline Tuberculoid
17) Commonest site for Fixed drug eruption (FDE)- lips
18) DOC for tinea - terbinafine
19) DOC for tinea capitis- griseofulvin
20) Most common type of onychomycosis – Distal and lateral onychomycosis (In HIV, the most common
type is proximal subungual onychomycosis and superficial whiteonychomycosis)
21) DOC for sporotrichosis- itraconazole> potassium iodide
22) Most common cause of reactive arthritis- Chlamydia> Shigella
23) Most common Psoriatic arthritis- oligoarticular, asymmetric.
24) DOC for psoriatic arthritis- Methotrexate
25) DOC for arthritis mutilans- etanercept
26) DOC for guttate ps- antibiotics
27) DOC for erythrodermic psoriasis- Methotrexate
28) DOC for pustular psoarisis- Acitretin
29) DOC for early mycosis fungoides- Electron beam therapy > Phototherapy
30) Most common type of pemphigus- pemphigus vulgaris
31) Rarest type of pemphigus- pemphigus vegetans
32) DOC for Dermatiis herpetiformis- Dapsone
33) Most classical joint involved in Psoriatic Arthritis- DIP
34) Most common cause of non bullous impetigo- strepto> staph
35) Most common cause of bullous impetigo- staph
36) Most common underlying disease in kaposis varicelliform eruption - atopic dermatitis
37) Most common site of adult atopic dermatitis- ante cubital fossa
38) Most common site of pediatric atopic dermatitis- cheek
39) Most common cause of cumulative Irritant contact dermatitis- detergents, Wet work
40) Most commonest cause of Allergic contact dermatitis-nickel
41) Most common cause of air borne contact dermatitis- parthenium
42) DOC for air borne contact dermatitis - azathioprine
43) Most common layer of epidermis for lamellar body presence- granular layer
44) Most common layer for synthesis of vitamin D ( Also same answer for presence of langerhans cells)-
spinous layer
45) Thickest layer of epidermis- corneum
46) Thinnest layer of epidermis- granular
47) Most common cause of acute paronychia- staph
48) Most common cause of chronic paronychia-candida
49) Most common syphilis transmitted by sexual route- primary
50) Most common syphilis transmitted from infected mother- secondary
51) Most infectious lesion in syphilis- mucous patches
52) Most sensitive test in syphilis- Enzyme Immunoassay> TPPA > FTA-abs
53) Most specific test in syphilis- TPPA> TPHA
54) DOC for chancroid- azithro
55) DOC for LGV and donovanosis- doxy
56) DOC for syphilis in pregnancy- penicillin
57) DOC for urethral discharge and cervical discharge (syndromic management)- azithro+ cefixime
58) DOC for vaginal disch (syndromic management)- fluconazole + secnidazole/metro/tinidazole
59) DOC for bubo (syndromic management)- azithro+ doxy
60) DOC for genital ulcer (syndromic management)- if vesicle - acyclovir, if not azithro+ benzathine
penicillin
61) DOC for neurosyphilis- crystalline aqueous penicillin
62) DOC for penicillin allergy in syphilis- doxy
| P a g e 83

63) DOC for penicillin allergy in syphilis in pregnancy- desensitization


64) DOC for penicillin allergy in neurosyphilis- desensitization
65) DOC for Impetigo herpetiformis- steroids
66) Investigation of choice in primary syphilis- dark ground illumination
67) Most common cutaneous TB- lupus vulgaris (In children, it is Scrofuloderma)
68) Test of choice for lupus vulgaris- biopsy
69) Most common organism for p versicolor now in India- Malassezia globosa
70) Most common internal organ inv in leprosy- testis
71) Organ never inv in leprosy- uterus> CNS72) Sensation never lost in hansens- propioception, vibration
73) DOC for Post herpetic neuralgia- Gabapentin
74) Most characteristic of LP on histopathology- basal cell degeneration
75) Best time to read patch test- 4 days
76) Commonest drug for FDE- sulphonamides
77) Commonest cause of Erythema Multiforme- HSV
78) Commonest cause of SJS/TEN- drugs (NSAIDS, anti epileptic, sulphonamides, penicillin)
79) DOC for scabies- 5% permethrin
80) DOC for scabies in pregnancy- 5% permethrin
81) DOC for nodular scabies- permethrin+ steroids
82) DOC for nerve abscess- I and D
83) DOC for nodulocystic acne- oral isotretinoin
84) DOC for hormonal acne- OCP with drosperinone+ estrogens
85) DOC for pediculosis corporis- disinfection of clothes
86) DOC for head louse- 1% permethrin
87) DOC for norwegian scabies- ivermectin
88) Most common shape of burrow in scabies- S-shaped
89) Most common and earliest manifestation of tuberous sclerosis- ash leaf macule> adenoma sebaceum
90) Earliest manifestation of congenital syphilis- snuffles
91) Best blood test for congenital syphilis- FTA-ABS igm
92) Most common site for morphoea- limbs
93) Most common cause for salt and pepper skin pigmentation- scleroderma
94) Most common cause of acanthosis nigricans- obesity
95) Most severe form of psoriasis- Von zumbusch
96) Most common melanoma- superficial spreading melanoma
97) Poorest prognosis in melanoma- nodular
98) Most common type of BCC- noduloulcerative
99) Most common cause of hypopigmented, scaly patches on cheek of children- Pityriasis alba
100) Most common cause of hypopigmented, nonscaly, atrophic patches on cheek of endemic area
children- indeterminate hansens
101) Investigation of choice for neurosyphilis- CSF-VDRL
102) Most common type of oral LP- reticulate / white lacy pattern
103) DOC for localised alopecia areata- intralesional steroids
104) Most effective drug in alopecia areata- contact sensitizers
105) commonest autoimmune association in vitiligo- thyroid
106) Commonest agent for leucoderma- paratertiary butyl phenol (PTBP)
107) Commonest agent for hair dye allergic contact dermatitis- paraphenylene diamine (PPD)
108) Commonest agent for footwear allergic contact dermatitis- Mercaptobenzothiazole (MBT)
109) Investigation of choice for Air borne contact dermatitis- photo patch test
110) Commonest extra genital site for primary chancre- lips
111) Commonest cause of recurrent blisters on genitals (healing with hyperpigmentation- FDE) ( if not
then herpes genitalis)
112) Commonest vitiligo- Vitiligo vulgaris
113) Most common cause of erythema nodosum- Strepococcus.
114) Most common cause of patchy alopecia- Alopecia areata
115) DOC for rosacea- Metronidazole (topical), Doxy (Oral)
| P a g e 84

116) Commonest site for primary syphilis chancre- Coronal sulcus


117) First test to become positive in primary syphilis- FTA-Abs
118) Characteristic nail change in LP- Pterygium
119) Commonest cause of apple jelly nodules- Lupus vulgaris
120) Commonest cause of hypopigmented, minimally scaly macules and patches on chest and back of
young adults- P. Versicolor
=================================================
Polymyositis V/S Dermatomyositis V/S Inclusion Body Myositis
=================================================
►Affects Males > Females: Inclusion Body Myositis, In contrast to the other 2.
►Associated with Parasites (protozoa, cestodes, nematodes), tropical and bacterial myositis::
Polymyositis
►Associated with malignancy:: Dermatomyositis (15%)
►Familial association:: Inclusion Body Myositis(in some cases)
►Systemic autoimmune diseases:: Frequently with Polymyositis and less frequently with the other 2.
►Not associated with Drugs:: Inclusion Body Myositis. Drugs include penicillamine (dermatomyositis and
polymyositis), zidovudine (polymyositis), and contaminated tryptophan (dermatomyositis-like illness).
►In contrast to polymyositis and dermatomyositis, where facial muscles are typically spared (Dalakas
2010a; 2010b), mild facial weakness is very common in inclusion-body myositis
►Rash or calcinosis associated with:: Dermatomyositis
►Pattern of weakness
•Proximal muscles > Distal Muscles: In case of Polymyositis and Dermatomyositis
•Distal muscles > Proximal Muscles: Inclusion Body Myositis
►Responds to IVIG:: Dermatomyositis
►Muscle biopsy findings::
•"Primary" inflammation with the CD8/MHC-I complex and no vacuoles::Polymyositis
•Perifascicular, perimysial, or perivascular infiltrates, perifascicular atrophy::Dermatomyositis
•Primary inflammation with CD8/MHC-I complex; vacuolated fibers with -amyloid deposits; cytochrome
oxygenase–negative fibers; signs of chronic myopathy:: Inclusion Body Myositis
ORANGES IN MEDICINE
*orange placenta---- syphilis
*orange tonsils --- Tangiers disease
*peau'd orange appearance--- ca breast
*orange discolouration of urine-- dehydration
drugs--rifampicin, furazolidone, sulfosalazine, Rifabutin,Entacapone,Clofazimine
*orange discolouration of skin--carotenodermia
*orange/saffron coloured amniotic fluid-- post maturity
*Orange skin cornea is seen in Mustard Gas poisoning used in biological warfare...
*orange cylinder - - Cyclopropane
� � � � FRUITS IN MEDICINE � � �
�Strawberry tongue- kawasaki disease,scarlet fever,tss
�Strawberry gums- wegener's granulomatosis
�Strawberry hemangioma- capillary hemangioma
�Strawberry vagina in trichomoniasis.
�Strawberry skin n nasal mucosa- sarcoidosis
�Strawberry nasal mass- rhinosporidiosis
�Strawberry shaped skull - edward syn
�Strawberry gallbladder -cholesterolosis
�STRAWBERRY HAMENGIOMAS IN SKIN
�STRAWBERY VAGINA-TRICHOMONIASIS
�STRAWBERRY RECTOSIGMOID -INFECTION OF SPIROCHETE
�STRAWBERRY SKIN- sarcoidosis
�Strawberry sigmoid-b.vincenti
| P a g e 85

�Strawberry nevus - capillary haemanigoma.


�BLUEBERRY: Rubella, CMV
(congenital disease)
�MULBERRY: Tuberous sclerosis
(mulberry tumor), syphilis
(mulberry molar,neonatal finding)
�RASPBERRY TUMOR: Umbilical
adenoma
�CRANBERRY JUICE - prevents UTI
�Berry aneurysms - saccular aneurysm of cerebral artery.
�Berry ligament- lateral thyrohyoid ligament.
�Blueberrymuffin baby - congential CMV.
INFECTION,toxoplasmosis,rubella
�Bunch of grapes sign- hydatiform mole.
�Mulberry calculus- urinary calculus.
�Mulberry spot- abdmonial eruption in typhus fever.
�Mulberry molar - congenital syphilis.
�Mulberry hypertrophy - Rhinitis medicamentosa.
�Apple peel appearance : Intestinal
atresia
�Apple jelly nodules: Lupus vulgaris
�Apple core deformity- in
carcinoma colon (double contrast
barium enema finding)
�Apple peal appearance- ileal atresia
�Apple sauce appearance- meconeum illeus
�Apple core sign - synovial chondramatosis of femur
�Peau'd orange skin:Breast cancer
�Lemon on stick appearance :
cushing's syndrome
�Banana nd � lemon sign.. Both r usg finding of arnold chiari type 2 malformations (sp a/s with sipna
bifida) lemon sign due to indentation in frontal bone. Banana fr contour of cerebellum
�Banana fracture- paget disease.
�Pear shaped bladder-pelvic haematoma, bilateral
lymphocoele.
� Cherry red spot - cherry red spot myoclonus syndrome.
�Cherry angioma - senile haemanigoma.
�CHERRY RED SPOTS-central retinal artery occlusion,berlins oedema,tay sachs,niemanns picks,
gauchers
Most common cancer
A) MALE
1) Incidence---Lung cancer
2)Mortality---Lung cancer
B)FEMALE
1)Incidence----Breast ca
2)Mortality----Breast cancer
C) OVERALL
● Most common incidence and mortality both---Lung cancer
• “Revised Ghent criteria ”
---------------------------------------------
It is currently used for clinical diagnosis of Marfan syndrome.
| P a g e 86

It considers :
a) Family history
b) Cardinal clinical signs in the absence of family history
c) Presence or absence of Fibrillin mutation
Disorders affecting the tongue :
· Furred Tongue Poor dental hygiene/Excessive smoking/Topical or systemic antibiotic
· Atrophy of the Tongue Anemia: iron deficiency/Ariboflavinosis/Pellagra
· Scrotal Tongue :Hypothyroidosis/Edema/Symphiliticgumma
· Bifid Tongue : Oral facial digital syndrome/Cleft palate/Widely spaced teeth
· Geographical Tongue -/Psoriasis/Atopic dermatitis/Seb. Dermatitis
· Median rhomboid glossitis - develop mental defect/C h. Oral candidiasis
· Glossodynia : burning tongue/psychological disorder

Cytochrome P450 inducers (↓metabolism) Cytochrome P450 inhibitors (↑metabolism)


 Phenobarbitone ↓↓  Fluconazol ↑↑
 Phenytoin  Itraconazol
 Rifampin  Ketoconazol
 Grisofulvin  Cimitidine
 Cortisol  Erythromycin
 Carbamazepine (Tegratol)  Miconazol
 Clarithromycin

Drugs ↓ Cyclosporine A blood level Drugs ↑ Cyclosporine A blood level


 Phenytoin ↓↓  Fluconazol ↑↑
 Rifampin  Itraconazol
 Carbamazepine (Tegratol)  Ketoconazol
 Methylprednisolone
 Allopurine
 Erythromycin
 Clarithromycin

Mycoplasma Lymphogranuloma venereum Granuloma inguinale Chancroid


In ♂ 1ry stage Serpiginous granuloma Genital ulcer
 Urethritis  Ulcer  (painless ulcer)  (painful ulcer)
 Epididymitis  Urithritis
In ♀  Proctitis Pseudobubos Genital discharge
 Bacterial vaginosis  (painless LN)
 Pelvic inflammatory disease 2ry stage
In Both  Lymphadenitis Scarring and mutilation Genital LN
 Reiter syndrome  Inguinal bubos,  (painful LN)
In Infants  Groove's sing
 Pneumonia
 Fever 3ry stage
 Respiratory distress syndrome  Genital elephantiasis
 Anorectal strictures & fistulas

FANA Diagnosis
1- Peripheral SLE
2- Homogeneous Drug
3- Nucleolar Scleroderma
4- Speckled MCTD
| P a g e 87

AD AR XLR XLD
1. Neurofibromatosis 7. Xeroderma pigmentosum 12. X-linked ichthyosis 18. Incontinentia
2. Ichthyosis vulgaris 8. Pseudoxanthoma 13. Dyskeratosis congenital pigmenti
3. PPK 9. Phenylketonuria 14. Ehlers-Danlos syndrome 19. Focal dermal
4. Darier's disease 10. NBIE 15. Weskot-Aldrich's syndrome hypoplasia
5. C1 esterase angioedema 11. Acrodermatitis 16. Chronic granulomatous 20. CHILD syndrome
6. Peutz Jaghers enteropathica disease
17. Fabry disease

DNA virus RNA virus


 HPV  Measles (Paramyxovirus)
 Hhvs  Rubella (Togavirus)
 HHV1 (Herpes simplex)  Coxsaxkie (Hand-Foot-Mouth disease)
 HHV2 (Herpes simplex)  Hepatitis A
 HHV3 (Herpes zoster)  Hepatitis C
 HHV4 (EBV) (infectious mononucleosis)  Hepatitis D
 HHV5 Cytomegalo virus  Hepatitis E
 HHV6 Roseola Infantum  AIDS
 HHV7
 HHV8 Kaposi sarcoma
 Pox virus
 Vaccinia (monkey pox, Cow pox, Small pox)
 Orf (milker nodule
 Molluscum contagiosum
 Hepatitis B

You might also like